Free Essay

Writing Workout

In:

Submitted By zeeman786
Words 55651
Pages 223
SAT
WRITING
ESSENTIALS

SAT
WRITING
ESSENTIALS

®

NEW

YORK

Copyright © 2006 LearningExpress
All rights reserved under International and Pan-American Copyright Conventions.
Published in the United States by LearningExpress, LLC, New York.
Library of Congress Cataloging-in-Publication Data:
Starkey, Lauren B., 1962–
SAT writing essentials / Lauren Starkey.
p. cm.
ISBN 1-57685-532-5
1. English language—Composition and exercises—Examinations—Study guides. 2. SAT
(Educational test)—Study guides. I. Title.
LB1631.5.S785 2006
378.1'662—dc22
2005027520
Printed in the United States of America
987654321
ISBN 1-57685-532-5
For more information or to place an order, contact LearningExpress at:
55 Broadway
8th Floor
New York, NY 10006
Or visit us at: www.learnatest.com About the Author

Lauren Starkey is a writer and editor who specializes in educational and reference works. Her thirteen years of experience include eight years on the editorial staff of the Oxford English Dictionary. The author of more than ten volumes, Lauren lives in Essex, Vermont, with her husband and three children.

v

Contents

CHAPTER 1

1

Old versus New

1

Strategies for Test Taking

2

Scoring

4

SAT Study Timetable
CHAPTER 2

Getting to Know the Writing Section of the New SAT

5
11

Identifying Sentence Errors

12

Improving Sentences

32

Improving Paragraphs
CHAPTER 3

The Multiple-Choice Section

45

The Essay

55

Strategies for Timed Essays

56

Understanding the Prompts

58

The Art of Persuasion

59

Anatomy of an Essay

59

Planning Your Essay

65

Drafting Your Essay

68

Essay Writing Workshop

69

vii

– CONTENTS –

CHAPTER 4

Practice Test 1

75

CHAPTER 5

Practice Test 2

103

CHAPTER 6

Practice Test 3

133

viii

SAT
WRITING
ESSENTIALS

C H A P T E R

1

Getting to Know the Writing
Section of the
New SAT
For over 80 years, high school juniors and seniors have faced the SAT on their paths to college. During that time, the test has undergone some changes. However, the new SAT, offered for the first time in March
2005, represents the most significant change in the history of the test.
What does that mean for the more than two million students who take the test each year? They’ll miss more of the Saturday on which they take it: The old SAT was three hours long, and the new one is almost four. Instead of two sections, the test now includes three, and the top score is 2,400 instead of 1,600. But of even greater importance are the changes within those sections. Let’s look more closely at what today’s students will encounter with the new SAT.

Old versus New
Minor changes have been made to the Math and Verbal sections. Math topics have been expanded to include exponential growth, absolute value, and functional notation. Familiar topics, such as linear functions, manipulations with exponents, and properties of tangent lines, are given greater emphasis. Skills such as estimation and number sense will be tested in new formats. The Verbal section is now known as Critical Reading, and has added short reading passages while eliminating analogies.
The biggest change to the new SAT is the addition of a Writing section; however, all of the material in this section isn’t entirely new. The Writing section has three parts; the first two are multiple choice, and the last is essay writing. You’ll have 35 minutes to complete the multiple-choice section, which is broken down into 25- and 10minute parts. It contains the same structure and content as the “old” SAT II Writing Test (which was optional,

1

– GETTING TO KNOW THE WRITING SECTION OF THE NEW SAT –

gent preparation or coaching is a combination of three critical components:

and has now been eliminated), and includes 49 questions designed to measure your knowledge of basic grammar and usage rules as well as general writing and revising strategies. The questions consist of three types: identifying sentence errors, improving sentences, and improving paragraphs. Preceding the multiplechoice section is the essay, for which you are given a prompt to which you have 25 minutes to respond.
Here’s an overview of each section:








1. studying the material that will be presented
2. studying the test itself
3. practicing by taking mock tests
For most students, working through the second and third components makes the most difference on test scores. You’ve already learned the math, grammar, and critical reading skills that are tested on the SAT.
While you might need a refresher on some of those skills, what’s even more important is understanding the test itself. In this book, we’ll review misplaced modifiers, but we’ll also reveal how they’re used on the test, and how you can spot them more easily. When you are very familiar with the test’s format through study and practice, your performance will improve.

Essay. The essay will always be the first section on the SAT. You’ll get a prompt, which will either be one quote, two quotes, or a sentence that you must complete. Then, there is an assignment that explains what you need to do. You might have to agree or disagree with a quote, develop your point of view about an issue related to a quote, or explain the choice you made in the sentence completion. Identifying Sentence Errors. In each question is one sentence with four words or phrases underlined. You need to determine which underlined portion, if any, contains an error.
Improving Sentences. Each question contains five versions of a sentence—you choose the one that is most clear and correct.
Improving Paragraphs. Only about 10% of the questions in the writing section are this type, which is good news. They are the most timeconsuming, with five or six questions relating to a passage of about 200 words. The questions can involve organization of paragraphs, sentence order, word choice, and grammar issues.

Strategies for Test Taking
One of the factors cited in the coachability argument is the fact that there are methods of approaching the SAT that work much better than others. For example, when you know that it only makes sense to guess when you can eliminate one or more multiple-choice answers, you are much more likely to get a better score. Likewise, be aware that there are easy questions, which come first, and harder questions, which appear at the end of the test. It makes sense to answer the easiest first because the computer scoring your test does not discriminate—each right answer, whether to a difficult or simple question, counts for just one point. Your objective is to get as many right as possible within the allotted time. Hard questions may take a couple of minutes to think through, while during the same time, you could have answered three easier questions.

Coachability

The SAT, including the new Writing section, is often referred to as a coachable test. That means you can improve performance through study and practice, whether with this book, software, or a course. In fact, many companies in the test-preparation business tout a hundred- or more point gain for their students. Dili-

2

– GETTING TO KNOW THE WRITING SECTION OF THE NEW SAT –

Skip questions you don’t know how to answer.
You can leave questions blank and still get a good score. It doesn’t make sense to spend a lot of time on a really difficult question if you can skip to others that could be easier for you. If you have extra time, you can go back to the tough ones and try again.

Determining the Level of
Difficulty

How do the writers of the SAT determine the level of difficulty of each question? Before the question is included in the actual test, it’s put into an experimental or “equating” section. If you haven’t heard about this section before, here’s the scoop: Every test contains one of these sections, and it doesn’t count toward your score. However, since there’s no way to know which section it is, apply yourself equally to all of the material on the test; don’t waste time trying to identify the experimental one.
Once an experimental test section has been given, the Educational Testing Service (ETS) looks at the results. If most test takers get a question right, it’s determined to be easy, and if most get it wrong, it’s hard. The questions in each section, then, are organized from easiest to hardest. If there are fifteen sentence errors questions, five will be easy, five will be average, and five will be hard.
More specific strategies for each section will be given in Chapters 2 and 3, but here are a few more general pointers:

Read carefully.
Moving too fast can hurt your score. Multiple-choice questions, especially the last few, can be subtle. If you miss a word, or otherwise read the question incorrectly, you’ll probably get it wrong. Essays that don’t directly address the topic get a zero (the lowest score) no matter how well written they are.
Use your test booklet.
During the test, your booklet may be used to flag questions you’ve skipped (you may have time to get back to them), underline or circle key words in a question, and/or eliminate choices you know are wrong. Go ahead and mark up your booklet—once you’re done with it, it’s headed to a paper shredder.
Be aware of the time.
When time is called, you must put down your pencil and close your book. Keep track so you aren’t caught off guard; taking practice tests with a timer will help you familiarize yourself with the number of questions and their difficulty in relation to the clock.
Remember that if you finish a section early, you can go back and try those you skipped, or check your answers (only in that section).

Study the directions before taking the test.
Following the directions exactly is critical. Why spend valuable time during the test poring over them? The College Board, which administers the
SAT, reveals the directions word-for-word on their website (www.collegeboard.com). Study and understand them ahead of time, and you’ll have more time to spend answering questions and scoring points.

3

Snacking on Test Day
The new SAT is 30 minutes longer than its previous incarnation, so the breakfast you ate before taking the test won’t give you enough energy to get through it. It’s a great idea to bring foods that give you long-lasting energy rather than sugary snacks that temporarily elevate your blood sugar. Think nuts, dried fruits, and cheese (not strong-smelling), as well as bottled water, to improve stamina and concentration.

Scoring

Come prepared.

The Writing section is scored in two ways: Multiplechoice questions are scored by a machine, and the essay is scored by two graders. The machine simply reads the marks you made with your number two pencil. It gives you one point for every correct answer, deducts a quarter of a point for every incorrect answer, and gives you zero points for questions left blank.

Bring with you:








your admission ticket sharp, number two pencils (at least two) a good eraser identification with photo (such as a driver’s license, a school- or government-issued ID card, or a valid passport) a watch (if it has an alarm, turn it off) snacks, including water: These must be in sealed containers within a book bag and can only be consumed out of the testing room during breaks.

Should You Guess?

Every multiple-choice question has five possible answers, meaning that if you have no idea which is correct, a guess will give you a 20% chance of getting it right. But if you guess wrong, you’ll lose one-quarter of a point. In other words, for every five questions you answer with random guessing, you’ll probably get one right. But you’ll lose a point for the four you got wrong.
That means random guessing is a waste of time. If you can’t eliminate even one answer, skip the question.
But what if you can eliminate one or more answer choices? If you are positive one answer is wrong, you now have a 25% chance of getting it right, up from 20% if you couldn’t eliminate any answers. That means for every four questions you answer this way, one will be right. Subtract the three-quarters of a point you lose for the three wrong answers, and you are ahead a quarter of a point. If you can positively eliminate two answers, and are guessing between the remaining three, you’ll be ahead even more. Therefore, if you can narrow down the answers by even just one, it makes sense to guess.

Leave at home:





blank paper, notes, books, and dictionaries highlighters, pens, and colored pencils portable listening or recording devices cell phones and pagers

4

Address the Topic
It can’t be stressed enough how important it is to clearly address the topic. You can write an incredible essay filled with unique insights, mature diction, and outstanding organization and development.
But if it doesn’t address the topic, it will receive a zero.

SAT Study Timetable

The Essay

Scorers of the essay are high school and college teachers who use a scale of 1–6; their two scores are combined to reach an essay score of 2–12. (Note, however, that an essay written off-topic, no matter how good, will receive a zero.) If the two scores vary by more than a point, a third reader scores it.
Essay scorers are trained to use a holistic approach, meaning they consider the essay as a whole, rather than word-by-word. Big issues, such as organization and structure, count more than little ones, such as an errant spelling mistake or extraneous comma.
That means essays receiving a twelve may have a couple of mechanics errors.
Specifically, scorers look for three things:

Whether you’re reading this book six weeks or six months before you take the SAT, the steps in your timetable remain the same.
Now

Take a practice test, such as the one in Chapter 4. Score your test and analyze the results. For each incorrect response, ask yourself:




1. development of a point of view in response to the topic
2. strong supporting examples and details
3. skillful use of language



Don’t get put off by the third requirement. Scorers know you have just 25 minutes to write your essay, so they don’t expect perfect grammar and punctuation
(although it certainly won’t hurt!). They will look for word choices that reflect a strong vocabulary (avoid clichés and slang), variety in sentence structure, and logical development of ideas. We’ll go into greater detail about essay specifics in Chapter 3.

Was there something you needed to know that you didn’t know? Make a list of the topics you need to review and devote extra time to studying them. Did you misunderstand the question? What about the question confused or tricked you?
Did you make a careless mistake? Careless mistakes include transference errors (marking the wrong oval on the answer sheet) and simple misreading, such as mistaking one word for another.

When you’ve finished your analysis, use it to make a list of your strengths and weakness. You’ll see which specific skills need reviewing, and which test-taking skills need improving. Then, get out your calendar.
How much time can you realistically devote each day and each week to your SAT preparations? Estimate how long you can spend on each of the four question types. 5

– GETTING TO KNOW THE WRITING SECTION OF THE NEW SAT –

you’ll get a chance to try some practice questions. If you haven’t improved since your first practice test, you’ll need a more thorough review of the issues that tripped you up. Goof-Proof Grammar (LearningExpress, 2002) not only covers grammar, but also usage and punctuation. It’s a great resource, because each short section is followed by a quiz that helps you retain what you’ve learned. Proceed through Chapter 2 in the same manner, one question type at a time.
Before you begin work on the Essay in Chapter 3, take your second practice test. You may choose to leave out the essay until your study of Chapter 3 is complete.
Score your test and analyze the results. Create a new list of strengths and weaknesses—you should notice a longer strength list this time!
Use the same techniques for Chapter 3, studying the material presented, and practicing with the Essay
Writing Workshop section. There, you’ll get to write thesis statements and introductory hooks for a number of prompts, and be able to read and score two complete essays written from the same prompt. Be sure to study the explanations of why each essay received the score it did.

Tomorrow until the Week before
Test Day

Use this book in stages as you study. There are four types of questions, including the essay. Plan on essay study and practice to take at least twice as long as one of the multiple-choice-question sections. Schedule the four stages into the time you have remaining, planning to complete study one week before your test date.
It makes sense to study Sentence Errors questions first, followed by Improving Sentences and
Improving Paragraphs. Why? The skills you need to tackle Sentence Errors may also be tested in Improving
Sentences questions. But Improving Sentences questions will also test for additional skills. Those additional skills, plus the ones you reviewed for Sentence
Errors, plus more additional skills, will be tested in
Improving Paragraphs. Each type of question, in other words, calls for a deeper understanding of the writing process, from grammar and usage to organization and development of ideas.
Therefore, in Chapter 2, you’ll find Identifying
Sentence Errors first. You’ll review the most common grammar and usage issues these questions test for, and

6

Study Suggestions
Your goal is to retain all of the material you study, and there are many different techniques to help you accomplish it. But some techniques are more effective than others. For example, taking practice tests is helpful ONLY if you carefully review your answers and learn why you missed certain questions. The best way to study the material in this book is to get active; instead of being a passive reader, interact with what you read by asking questions, taking notes, marking up passages, and making connections.


Ask Questions. The more difficult the passage you’re reading, the more crucial it is that you ask questions such as: What is this passage about? What is the main idea, or topic? What is the author’s point of view or purpose in writing this? What is the meaning of this word in this sentence? What does “it” refer to in this sentence? What is its antecedent? Is this sentence part of the main idea, or is it a detail?



Take Notes. Think about and respond to what you’re reading. Write the answers to the questions listed above. Record your reactions to the text, such as why you agree or disagree with the author’s point of view, or why you like or dislike his or her writing style. If you come across an unfamiliar word, look it up and record the definition (the act of writing it will help you remember it).



Mark It Up. Assuming this book belongs to you, highlight and underline when you read. When you see a main idea, mark it. If there’s an unfamiliar word or a word used in an unfamiliar context, mark it. The trick, though, is to be selective. If you’re marking too much of the passage, you need to practice finding where the author states his or her main idea.



Make Connections. Relate new material to what you already know. For example, if you’re trying to learn the word demographic, you may know that dem-ocracy refers to government by the people, while graphic refers to information, written or drawn. Then, you can remember that demographic has to do with information about people.
Making connections is one of the things that differentiates remembering from memorizing. In the short

run, it may seem easier to just memorize a word or a fact; but unless you understand what you’re learning—unless you have connected it to what you already know—you’re likely to forget it again. Then, you will have wasted your study time and not improved your test score.

During the week, locate your test admission ticket and put it with your personal identification. Make sure you know where you’re taking the test. If it’s an unfamiliar place, drive there so you will know how much time you’ll need to arrive punctually, park, and walk from parking to the building where you will take the
SAT. This “trial run” will help you avoid a last minute rush to the test, which would only increase anxiety.

One Week before the Test

Saturday morning, one week before you take the SAT, is a good time for your final practice test. Then, use your next few days to wrap up any loose ends. Reread your notes on test-taking tips and techniques. If you made vocabulary flash cards, look at a few each day. Log onto www.collegeboard.com and reread the official directions for each part of each section. They should be very familiar to you at this point.

7

– GETTING TO KNOW THE WRITING SECTION OF THE NEW SAT –

tions) to make sure you are transposing correctly. Look at the question number, and then check your answer sheet to see that you are marking the oval by that question number.
If you find yourself getting anxious during the test, remember to breathe. You have worked hard to prepare for this day. You are ready.

The Day Before

It’s the day before the SAT. Here are some dos and don’ts: DO:
■ relax!
■ find something amusing to do the night before— watch a good movie, have dinner with a friend, read a good book.
■ get some light exercise.
■ get together everything you need for the test: admission ticket, ID, number two pencils, watch, bottle of water, and snacks (see the box on page 4 for some guidelines).
■ go to bed early. Get a good night’s sleep.

Commit to Memor y
These are the most important points to remember from Chapter 1:


DON’T:
■ study; you’ve prepared, now relax.
■ party; keep it low key.
■ eat anything unusual or adventurous—save it!
■ try any unusual or adventurous activity—save it!
■ allow yourself to get into an emotional exchange with anyone; postpone any such discussion so you can focus on the exam.



Test Day









On the day of the test, get up early enough to allow yourself extra time to get ready. Set your alarm and have a back-up system in case it doesn’t go off. Ask a family member or friend to make sure you are up. Eat a light, healthy breakfast, even if you usually don’t eat in the morning. If you normally have coffee, don’t overdo it.
Too much caffeine can interfere with concentration.
Give yourself plenty of time to get to the test site and avoid a last-minute rush. Plan to get to the test room ten to fifteen minutes early. Once the exam begins, keep an eye on the time.
Remember not to spend too long on questions you don’t understand. Mark them (in your test booklet, not your answer sheet) so you can come back if there’s time. Check periodically (every five to ten ques-









8

The new SAT has three sections instead of two; the top score is 2,400 rather than 1,600.
The test is now three hours and 45 minutes long.
The Writing section is new; it’s comprised of the essay (25 minutes) and two multiple-choice groups (25 and 10 minutes).
Multiple-choice questions are: Identifying Sentence Errors, Improving Sentences, and Improving Paragraphs.
The SAT is a coachable test, meaning study and practice can improve your score.
Multiple-choice questions are presented in order of difficulty, with the easiest questions first.
Do all of the easiest Identifying Sentence Errors and Improving Sentences questions first. Then, complete the harder questions of those types.
Finally, tackle the Improving Paragraphs questions. Study the directions for each question type; you’ll save many minutes during test time if you don’t have to read them (official directions are at www.collegeboard.com). If you can’t eliminate at least one answer choice, skip the question.
If you can eliminate one or more answer choices, guess. – GETTING TO KNOW THE WRITING SECTION OF THE NEW SAT –







Use your test booklet: Mark off answer choices you know are wrong, circle questions you’ve skipped in case there’s time to come back to them, and take notes for your essay.
You’re not expected to turn in a final draft essay; aim for a “polished rough draft.”
Graders give your writing a total score of 2–12 based on a holistic reading that takes into account







9

development of a point of view in response to the topic, supporting examples and details, and use of language. Take a practice test before you begin studying to identify strengths and weaknesses.
Create a study schedule, and use this book to work through each type of question.
Ease up on studying the week before the test.

C H A P T E R

2

The MultipleChoice Section

A

s with the SAT as a whole, most of the questions in the Writing section are multiple choice. That means for each question, the correct answer is provided for you—but so are four incorrect ones.
The ETS is masterful at creating tricky distracters—wrong answers designed to look like possible correct answers. It’s your job to weed out the wrong answers and select the right one. While there are specific strategies for finding the right answer for each type of multiple-choice question, there are some basic rules that work for all of them:
■ Read the question carefully. Be sure you know exactly what is being asked. In particular, look for tricky wording such as, “All of the following are true about the author’s purpose EXCEPT.” Train yourself to notice any word in the question that is in all capital letters. Such a word will often completely change the meaning of the question.
■ Rule out incorrect answers. You may only be able to eliminate one or two incorrect answers, but every elimination increases the probability that you will choose the correct answer. Mark up your test booklet, putting a line through each answer as it is eliminated to avoid confusion.

11

– THE MULTIPLE-CHOICE SECTION –





Identifying Sentence Errors

Remember the rule for guessing. If you can’t eliminate one answer, skip it. If you can eliminate one or more, guess, and move on.
Beware of distracter techniques. While distracters, or traps designed to lure you away from the right answer, are more prevalent in the Critical Reading and Math sections of the SAT, they do appear in a more subtle form in the Writing section. You won’t find those absolute words, such as always, never, all, and none. But you will encounter, especially with more difficult questions, answers that at first glance appear correct.
For the easier questions, the answer probably is right, but for the difficult ones, think twice. We’ll get more specific about distracters as we explore each type of multiple-choice question in depth.

These multiple-choice questions are designed to test your knowledge of grammar and usage. Let’s take a closer look at how they are structured, the types of errors you’re most likely to encounter, and how best to approach Identifying Sentence Errors. At the end of this section, we’ve included ten practice questions (answers are at the end of the chapter).
Question Structure

Each sentence has four possible errors, underlined and marked a–d. There is also a choice e for “no error.” No sentence contains more than one error. It is your task to find the error, or choose e if the sentence is correct.
You are not asked to identify, explain, or correct it. All you must do is locate it.
Here’s a sample:

These are the three types of multiple-choice questions you’ll encounter in the Writing section:






Those old Atari video games in your closet are on the a wish list of the Computer Museum of America, in

Identifying Sentence Errors
Each sentence has four underlined words or phrases, and a fifth choice, “no error.” You need to determine which underlined portion, if any, contains an error in grammar or usage. If the sentence is correct, you will choose answer e, “no error.” Approximately 25% of the multiple-choice writing questions are this type.
Improving Sentences
You’ll be given five versions of a sentence, and have to choose the one that is most clear and correct. Approximately 65% of the questions in the
Writing section are Improving Sentences.
Improving Paragraphs
These questions concern a passage of approximately 200 words. They ask about how to improve the passage on many levels, from largescale organizational issues to word choice and grammar. Approximately 10% of the questions are this type.

San Diego, California, and they hope you will donate b c it to their holdings. No error. d e
What’s wrong? The plural verb are is correct; it expresses the action of the plural noun games. There’s also nothing wrong with San Diego, California and holdings. The plural pronoun they, however, is incorrect. It replaces the noun Computer Museum of America, which is singular.
Errors You’re Likely to See

There are many possible grammar and usage errors, and of course, every type is fair game. However, most of the questions will contain just a handful of common errors. In this section, we’ll review the eight you’re most likely to encounter. When you know what to look for, you’ll find sentence errors more easily.

12

Grammar and Usage
The grammar and usage issues you’ll most often encounter are:


agreement



pronoun case



consistency



idiom



parallel structure



word choice



verb form



adjectives and adverbs

Here’s an example:

We began this chapter with Identifying Sentence
Errors for one important reason: Many of the grammar and usage issues prevalent in these questions will also come up in Improving Sentences and Improving
Paragraphs questions. If the review that follows isn’t enough for you to fully understand each possible error, study those you still find confusing at greater length with a grammar book such as Goof-Proof Grammar
(LearningExpress, 2002).

“Eat, drink, and be merry,” is a label associated with a b
Greek philosopher Epicurus, but like most catchy slogans, they simplify what is actually a rich and c d complex message. No error. e Agreement
Expect at least one of your Improving Sentence Errors questions to be about agreement, meaning the balance of sentence elements such as subjects and verbs and pronouns and antecedents. (Recall that an antecedent is the noun that a pronoun replaces.) To agree, singular subjects require singular verbs, and plural subjects require plural verbs. Likewise, singular nouns can be replaced only by singular pronouns, and plural nouns require plural pronouns.
To make this type of question tricky, you’ll often find a “filler” phrase between the subject and verb or noun and pronoun. The intention is to distract you; the more space between the subject and verb or noun and pronoun, the more difficult it can be to determine agreement. Notice how the phrase like most catchy slogans can mislead you. If you assume slogans is the subject, then the pronoun they and the verb simplify seem correct—they agree with the plural subject. But subjects are never in prepositional phrases, so slogans can’t be the subject of the verb simplify. Look again at the sentence. What is simplifying? Not the slogans, but the label “Eat, drink, and be merry”—a singular noun.
Thus, the pronoun must be it and the verb must be simplifies to agree with the subject, so choice c contains the error and is therefore the correct answer.

13

– THE MULTIPLE-CHOICE SECTION –

Consistency
Just as sentences must be balanced, they must also be consistent. And like errors in agreement, errors in consistency involve verb and pronoun usage. For example, if a sentence begins in the past tense, it must stay in the past tense. Pronouns need to be consistent in person and number. A shift from the singular I to the plural we, for example, can leave the reader wondering just who is doing what in the sentence.
Here are two examples of errors in consistency from the practice tests:

need to follow the same grammatical pattern. If you’re writing about your friend’s favorite leisure activities, you wouldn’t say, “Juan loves skating and to read.” It sounds awkward because the items in the pair aren’t the same grammatically. The first is a participle (skating), and the second is an infinitive (to read). Even if the names of the verb tenses aren’t familiar to you, you can see that one of the two needs to be changed in order to maintain parallel structure. You could change skating to to skate, or to read to reading. Either way, you’ll get parallel structure.

Keeping your room uncluttered is easy when you a make it a habit to spend ten minutes a day just b c putting things back where they belonged. No error. d e

Here, again, are examples from the practice tests:
One of the best ways to prepare for a career in a journalism is to become an informed citizen by b reading a variety of newspapers, watching

Think twice before sending potentially computera clogging e-mail attachments such as pictures and b videos; if the recipient is low on disk space, or uses a c dial-up service to get their e-mail, he or she won’t d appreciate the gesture. No error. e documentaries and televised news programs, and c you should read books about world leaders, politics, d and grassroots movements. No error. e This sentence offers a list of three things one can do to become an informed citizen: reading, watching, and you should read. Notice how the third item does not follow the same grammatical pattern as the first. It should begin with a verb in participial form, reading.
You should is unnecessary and should be eliminated.

The first sentence is in the present tense as evidenced by the present participles keeping and putting, and the present make. But it ends with the past tense belonged.
To be consistent, that last verb needs to be changed to the present tense belong. In the second sentence, the author correctly uses the singular pronoun he or she to replace the singular noun recipient. But she then incorrectly uses the plural pronoun their to refer to the same antecedent.
The use of his or her would correct the error.

The more I read about deep sea fishing, the more a b it makes me want to get out there and try it. No error. c d e Parallel Structure
Parallel structure involves pairs and lists of words and phrases. Both items in a pair, and all items in a list

In this example, the phrases after the words the more I read are not grammatical equivalents. It makes me should be matched with I want to.
14

– THE MULTIPLE-CHOICE SECTION –

We can divide these irregular verbs into three categories: Verb Form
Verbs are the “meat” of a sentence—they express what the subject is doing, thinking, or feeling. Correct verb form is essential to sentence clarity, and you can expect to find at least one question with a verb form issue.
Here are some of the common verb errors found on the SAT.


irregular verbs with the same past and past participle forms irregular verbs with three distinct forms irregular verbs with the same present and past participle forms

Incorrectly conjugated irregular verbs. About
150 English verbs are irregular; that is, they do not follow the standard rules for changing tense.
PRESENT

The following table lists the most common irregular verbs.

PAST

PAST PARTICIPLE

SAME PAST AND PAST PARTICIPLE FORMS:

bite

bit

bit

dig

dug

dug

bleed

bled

bled

hear

heard

heard

hold

held

held

light

lit

lit

meet

met

met

pay

paid

paid

say

said

said

sell

sold

sold

tell

told

told

shine

shone

shone

shoot

shot

shot

sit

sat

sat

spin

spun

spun

spit

spat

spat

swear

swore

swore

tear

tore

tore

creep

crept

crept

15

– THE MULTIPLE-CHOICE SECTION –

PRESENT

PAST

PAST PARTICIPLE

deal

dealt

dealt

keep

kept

kept

kneel

knelt

knelt

leave

left

left

mean

meant

meant

send

sent

sent

sleep

slept

slept

spend

spent

spent

bring

brought

brought

buy

bought

bought

catch

caught

caught

fight

fought

fought

teach

taught

taught

think

thought

thought

feed

fed

fed

flee

fled

fled

find

found

found

grind

ground

ground

begin

began

begun

ring

rang

rung

sing

sang

sung

spring

sprang

sprung

swim

swam

swum

do

did

done

go

went

gone

am

was

been

is

was

been

THREE DISTINCT FORMS:

16

– THE MULTIPLE-CHOICE SECTION –

PRESENT

PAST

PAST PARTICIPLE

see

saw

seen

drink

drank

drunk

shrink

shrank

shrunk

sink

sank

sunk

stink

stank

stunk

swear

swore

sworn

tear

tore

torn

wear

wore

worn

blow

blew

blown

draw

drew

drawn

fly

flew

flown

grow

grew

grown

know

knew

known

throw

threw

thrown

drive

drove

driven

strive

strove

striven

choose

chose

chosen

rise

rose

risen

break

broke

broken

speak

spoke

spoken

fall

fell

fallen

shake

shook

shaken

take

took

taken

forget

forgot

forgotten

get

got

gotten

give

gave

given

forgive

forgave

forgiven

forsake

forsook

forsaken

17

– THE MULTIPLE-CHOICE SECTION –

PRESENT

PAST

PAST PARTICIPLE

hide

hid

hidden

ride

rode

ridden

write

wrote

written

freeze

froze

frozen

steal

stole

stolen

SAME PRESENT AND PAST PARTICIPLE FORMS:

come

came

come

overcome

overcame

overcome

run

ran

run

In English, as in many other languages, the essential verb to be is highly irregular:

SUBJECT

PRESENT

PAST

PAST PARTICIPLE

I

am

was

have been

you

are

were

have been

he, she, it

is

was

has been

we

are

were

have been

they

are

were

have been

Lay and lie are commonly confused. To lay means to place something down, and to lie means to recline.
Obviously in this sentence, the latter verb is required; however, to make things even more confusing, the correct past tense form of to lie is lay.

Here’s an example of an irregular verb question:
Eliza laid down on her bed to rest while the rest of a b c her family enjoyed the appetizing meal prepared by d her grandmother. No error. e ■

18

Incorrect tense. If there’s an error in tense, the sentence will provide enough context for you to determine the tense the verb(s) should be in. For example: – THE MULTIPLE-CHOICE SECTION –

Pronoun Case
Personal pronouns have two main forms: the subjective and objective cases. This simply means that we use one form when the pronoun is acting as a subject and another form when the pronoun is acting as an object.
Expect to see a couple of sentence errors involving confusion of subjective and objective cases.

From 1947 to 1956, thousands of scrolls and fragments of ancient manuscripts has been found in a caves on the shore of the Dead Sea, including early b c copies of biblical books in Hebrew and Aramaic. d No error. e PERSONAL PRONOUNS
SUBJECTIVE
CASE

I



you

he, she, it

him, her, it

we

us

they

them

who

Missing subjunctive. Most verbs are in the indicative mood, meaning that they simply indicate an action, thought, or feeling. The subjunctive mood is used to express something that is wished for or that is untrue. It is formed with the past tense or past perfect tense (using the helping verb were). But we often forget to use it, both in speech and in writing. When a sentence starts with if, I wish, or It would have been, it’s probably in the subjunctive mood.

me

you

Has been is the present perfect form of the verb is.
However, the first phrase, From 1947 to 1956, tells us that the action took place in the past. This sentence requires the simple past tense, were.

OBJECTIVE
CASE

whom

Here’s an example that uses a distracter:
The difference between you and me is that you get a b your best work done in the morning while I c perform better in the evening. No error. d e

It would have been nice if you brought more money a b with you because these tickets are very expensive. c d
No error. e You and me is not the subject of the sentence (difference is), but rather the object of the preposition between. Even if it might sound wrong, me, the objective form of I, is correct.
Another common sentence error involves a pronoun following the word than. Because the than +
(pro)noun construction requires a verb (even if that verb is not articulated), you must use the subjective form of the pronoun: I am taller than he [is].

The clause it would have been tells us that the money isn’t there, it’s just wished for, so the verb needs to be subjunctive: if you had brought.

19

– THE MULTIPLE-CHOICE SECTION –

Idiom
Idioms are expressions peculiar to a particular language, whose meanings cannot be discerned by defining them word for word. What downward movement, for example, happens when one “falls in love”? On what is one perched on when “sitting pretty”? There are thousands of English idioms, most of which are very familiar to you, even though you may not have known they were idioms. The two most common errors you’re likely to encounter are those involving prepositional pairs (e.g., take care of, according to) and the use of infinitives and gerunds (e.g., want to meet, practice swimming). Since idioms are typically learned through conversation, you’ll probably be able to hear idiom errors in the Identifying Sentence Errors multiple-choice questions. Listen carefully to each sentence as you read it, and identify the error.

You’ll probably find at least one multiple-choice question that tests your ability to differentiate between who and whom. Who is the subjective form, and whom is the objective. If you’re unclear about which to use, substitute the words he and him for who or whom. If he is correct, you need who (both subjective case) and if him is correct, you need whom (both objective case).
Here’s an example:
The physical and psychological unrest of the working a class was explored often in the plays of Arthur Miller, b for who the subject of the American Dream, and its c achievability for ordinary Americans, never got stale. d No error. e 1. This year’s model is different than last year’s.
2. She has difficulty in the Advanced Placement
History class.
3. The color choices are typical for that artist.

Who is the object of the preposition for, but it is in the subjective case. Correct it by changing it to the objective form whom. If you were unclear about whether this was the error, you could have recast the sentence to try he or him in place of who:

These errors should have sounded wrong to you:
(1) different than should be different from, (2) difficulty in should be difficulty with, (3) typical for should be typical of. If the subject of prepositions is confusing, you’ll need to do some memorizing. Idioms are idiosyncratic—there are no easy rules for remembering them! Following is a list of idiomatic preposition uses that often appear on the SAT.

The subject of the American Dream never got stale for (he/him).
Obviously, him is correct.

20

– THE MULTIPLE-CHOICE SECTION –

correspond with a person (meaning to communicate) differ from something independent of someone or something interested in a subject interfere in someone’s business interfere with an activity similar to something stand by or with someone stand for a cause stand on an issue succeed in an endeavor wait at a place wait by the phone wait for someone wait in the snowstorm wait on a customer work with me

agree on an amendment agree to do something agree with someone argue about or for a proposal argue with a person apologize for an error approve of a change bored with small talk compare to (when you are showing the likes of two things or putting them in same category) compare with (when highlighting similarities or differences) concerned about or with an issue contrast with (when noting differences) correspond to or with something (meaning to relate) Here are the errors you should have heard: (1) practicing to swim should be practicing swimming, (2) pretending being should be pretending to be, and (3) resents to be should be resents being. As with prepositions, if you’re confused about when to use gerunds and infinitives, you’ll need to do some memorizing.

The other type of idiom error you’re likely to see is the improper use of infinitives (to + verb: to water) and gerunds (verb + ing: watering). Some verbs must take one or the other, and a small handful take both.
Your ear will probably hear this type of error. Listen for the non-idiomatic usages in the following sentences:
1. I spend two hours each day practicing to swim.
2. We had fun pretending being rock stars.
3. My father resents to be asked for money all the time. VERBS THAT
TAKE INFINITIVES

VERBS THAT
TAKE GERUNDS

VERBS THAT TAKE
EITHER INFINITIVES
OR GERUNDS

afford

admit

attempt

agree

adore

begin

aim

appreciate

bother

ask

avoid

cannot bear

appear

consider

cannot stand

21

– THE MULTIPLE-CHOICE SECTION –

VERBS THAT
TAKE INFINITIVES

VERBS THAT
TAKE GERUNDS

VERBS THAT TAKE
EITHER INFINITIVES
OR GERUNDS

be determined

deny

cease

beg

detest

continue

care

discuss

hate

claim

dislike

hesitate

decide

enjoy

intend

expect

escape

like

have

finish

love

hope

imagine

prefer

learn

keep

start

manage

mind

mean

miss

need

postpone

offer

practice

plan

put off

pretend

quit

promise

recall

refuse

recollect

say

resent

tend

resist

try

risk

wait

suggest

want

tolerate

wish

understand

22

– THE MULTIPLE-CHOICE SECTION –

There are two categories of words that are most problematic: confused words (homonyms that sound the same but have different meanings and spellings) and misused words (pairs so often used incorrectly the errors sound acceptable to most people). Read through the lists of some of the most frequently tested words, noting any you’re not sure of.

Word Choice
Many students breathed a sigh of relief when the College Board announced that it was dropping Analogy questions from the SAT. These questions appraised vocabulary, and were thought to be among the hardest on the test. However, they’ve been replaced by a handful of Identifying Sentence Errors questions involving word choice. Here’s where you’ll need to show you know the difference between affect and effect, whether or weather, and fewer and less.

CONFUSED WORDS
WORD

DEFINITION

a lot (noun)

many

allot (verb)

to give or share in arbitrary amounts

accept (verb)

to recognize

except (prep.)

excluding

access (noun, verb)

means of approaching; to approach

excess (noun, adj.)

extra

addition (noun)

increase

edition (noun)

an issue of a book or newspaper

advice (noun)

a recommended opinion

advise (verb)

to give advice; inform

affect (verb)

to influence

effect (noun)

result

effect (verb)

to bring about

all ready (adj.)

completely prepared

already (adv.)

by or before a specified or implied time

all together (adj.)

in a group; in unison

altogether (adv.)

completely or thoroughly

23

– THE MULTIPLE-CHOICE SECTION –

CONFUSED WORDS
WORD

DEFINITION

allude (verb)

to refer to something not specifically mentioned

elude (verb)

to escape notice or detection

ascent (noun)

the act of climbing or rising

assent (verb)

to agree or accept a proposal or opinion

assure (verb)

to make certain (assure someone)

ensure (verb)

to make certain

insure (verb)

to secure from harm; to secure life or property in case of loss

beside (adj.)

next to

besides (adv.)

in addition to

bibliography (noun)

list of writings

biography (noun)

a life story

capital (noun)

money invested; a town or city where the government sits

capitol (noun)

a government building

choose (verb)

to select

chose (verb)

the past tense of choose

cite (verb)

to acknowledge; to quote as a reference

sight (noun)

the ability to see; vision

site (noun)

a place or location

complement (noun)

match

compliment (noun, verb)

praise; to give praise

consul (noun)

an official appointed by the government to live in a foreign city and attend to the interests of the official’s country

council (noun)

a group of people called together to provide advice

counsel (noun, verb)

advice; to give advice

continual (adj.)

taking place in close succession

continuous (adj.)

without break or let up

24

– THE MULTIPLE-CHOICE SECTION –

CONFUSED WORDS
WORD

DEFINITION

cooperation (noun)

assistance; help

corporation (noun)

type of business organization

decent (adj.)

well mannered

descent (noun)

decline; fall

dissent (noun)

disagreement

desert (noun)

arid, sandy region

dessert (noun)

sweet served after a meal

disburse (verb)

to pay

disperse (verb)

to spread out

disinterested (adj.)

impartial; no strong opinion either way

uninterested (adj.)

don’t care

elicit (verb)

to stir up

illicit (adj.)

illegal

envelop (verb)

to surround; to cover completely

envelope (noun)

flat paper container for letters or other documents

farther (adv.)

beyond

further (adj.)

additional

flack (noun, verb)

press agent (noun); to act as a press agent (verb)

flak (noun)

criticism

forth (adv.)

forward; onward

fourth (adj.)

next in number after the third

hear (verb)

to perceive by the ear

here (adv.)

in this or at this place

hoard (verb)

to collect and keep

horde (noun)

a huge crowd

imply (verb)

to hint or suggest

infer (verb)

to assume; to deduce

25

– THE MULTIPLE-CHOICE SECTION –

CONFUSED WORDS
WORD

DEFINITION

loose (adj.)

not restrained; not fastened

lose (verb)

to fail to win; to be deprived of

loath (adj.)

reluctant

loathe (verb)

to feel hatred for

medal (noun)

a badge of honor

meddle (verb)

to interfere

metal (noun)

a mineral substance

passed (verb)

the past tense of past

past (adj.)

finished; gone by

personal (adj.)

individual

personnel (noun)

employees

principal (adj.)

main

principal (noun)

person in charge

principle (noun)

standard

quiet (adj.)

still; calm

quit (verb)

to stop; to discontinue

quite (adv.)

very; fairly; positively

stationary (adj.)

not moving

stationery (noun)

writing paper

taught (verb)

the past tense of teach

taut (adj.)

tight

than (conj., prep.)

in contrast to

then (adv.)

next

their (pronoun)

belonging to them

there (adv.)

in a place

they’re

contraction for they are

26

– THE MULTIPLE-CHOICE SECTION –

CONFUSED WORDS
WORD

DEFINITION

to (prep.)

in the direction of

too (adv.)

also; excessively

two (adj.)

the number after one

weather (noun, verb)

atmospheric conditions; to last or ride out

whether (conj.)

if it be the case; in either case

who (pronoun)

substitute for he, she, or they

whom (pronoun)

substitute for him, her, or them

your (pronoun)

belonging to you

you’re

contraction for you are

WORD

WHEN TO USE IT

allude

used when a reference is made indirectly or covertly

refer

used when something is named or otherwise mentioned directly

amount

used when you cannot count the items to which you are referring, and

MISUSED WORDS

when referring to singular nouns number used when you can count the items to which you are referring, and when referring to plural nouns

anxious

nervous

eager

enthusiastic, or looking forward to something

among

used when comparing or referring to three or more people or things

between

used for two people or things

bring

moving something toward the speaker

take

moving something away from the speaker
Hint: Remember, you bring to, and you take away.

can

used to state ability

may

used to state permission

27

– THE MULTIPLE-CHOICE SECTION –

MISUSED WORDS
WORD

WHEN TO USE IT

each other

when referring to two people or things

one another

referring to three or more people or things

e.g.

an abbreviation for the Latin exempli gratia, meaning free example or for example

i.e.

an abbreviation for the Latin id est, meaning it is or that is

feel bad

used when talking about physical ailments

feel badly

used when talking about emotional distress

fewer

when you can count the items

less

when you cannot count the items

good

an adjective, which describes a person, place, or thing

well

an adverb, which describes an action or verb

its

belonging to it

it’s

contraction of it is
Hint: Unlike most possessives, it doesn’t have an apostrophe.

lay

the action of placing or putting an item somewhere; a transitive verb, meaning something you do to something else

lie

to recline or be placed (a lack of action); an intransitive verb, meaning it does not act on anything or anyone else

more

used to compare one thing to another
Hint: One of the two can be a collective noun, such as the ballplayers or the Americans.

most

used to compare one thing to more than one other thing

supposably

capable of being supposed

supposedly

believed to be the case

that

a pronoun that introduces a restrictive (or essential) clause

which

a pronoun that introduces a non-restrictive (or unessential) clause
Hint: Imagine a parenthetical by the way following the word which.
“The book, which (by the way) Joanne prefers, is her first novel,” is incorrect. Therefore, it should read, “The book that Joanne prefers is her first novel.” “Lou’s pants, which (by the way) are black, are made of leather,” is correct.

28

– THE MULTIPLE-CHOICE SECTION –

Adjectives and Adverbs
Because adjectives and adverbs serve similar functions
—they both modify or describe—they are often confused and therefore make good candidates for SAT questions. Remember that adjectives modify nouns or pronouns while adverbs modify verbs, adjectives, and other adverbs. Let’s look at a simple sentence:

Here are some examples of word choice errors from the practice tests:
1. Many people believe that the end justifies the means, so weather their intentions are good or a b c bad is irrelevant—the result is the only thing

The dog barked. that matters. No error. d e

We can modify or describe the noun dog by adding an adjective such as brown, skinny, or annoying.
We can also describe the action of the dog, the past tense verb barked, by adding an adverb. Loudly, softly, and incessantly would all work. Notice that all three adverbs end in -ly; it’s the most common ending for these modifiers.
Here are some examples of sentences that confuse adjectives and adverbs:

2. If you’re bothered by pesky telemarketers a b constantly interrupting you, register with the c National Do No Call Registry to add you’re home d and/or cell phone number to their list. No error. e 3. Former actor John Robert Powers has had a

1. The game Monopoly, originally called

huge affect on our culture; after opening the a b modeling agency in 1923, he began the first

The Landlord’s Game, was based on the a economic theories of Henry George, who

practice of selling everything from magazines c to food items to vacation destinations with d the help of beautiful people. No error. e proposed a single federally tax based on land b ownership; he believed this tax would weaken the ability to form monopolies, encourage c equal opportunity, and narrow the gap between

Did you spot the errors? In sentence 1, weather is used incorrectly. It is a noun referring to atmospheric conditions, and should be replaced by the conjunction whether. In sentence 2, you’re, a contraction of the words you are, should be replaced by the possessive pronoun your. In sentence 3, affect and effect are confused. Affect is a verb meaning “to influence.” The correct word is effect, a noun referring to result of the influence. rich and poor. No error. d e

29

– THE MULTIPLE-CHOICE SECTION –

superlative (–est) is for comparisons among three or more things. Whenever you find a sentence that includes a comparison, check to see how many items are being compared. For example:

2.The Italian master Artemisia Gentileschi painted large-scale historical and religious scenes, which a were often violently, in an era when women b artists were consigned to portrait painting and c imitative poses. No error. d e

Of the two cross-country trips I’ve taken, I liked the one to Santa Fe best.
Best might not jump out at first glance as an error; it doesn’t necessarily sound wrong. But there are only two things being compared, and that means the comparative better is correct.

3. Historians agree that the combat mission was a a b failure because the soldiers were inadequate c d armed. No error. e Strategies for Identifying
Sentence Errors

1. Listen to the sentence. If it sounds wrong to you, and the error isn’t immediately apparent, think about how you’d fix it. What change would you make to improve it?
2. This question type is presented in order of difficulty. Although each question is worth just one point, the first few will be easier than the last few.
If Identifying Sentence Errors appear first in your
25-minute section, don’t spend too much time on the difficult ones; it makes more sense to move ahead to the Improving Sentences section, where you’ll again encounter the easiest questions first.
3. Don’t look for punctuation or spelling mistakes; they’re not being tested in these questions.
4. Be wary of difficult questions (those appearing last). They’re usually complicated. If an answer seems obvious, it could be a trick.
5. If you’ve eliminated a choice or two, and still can’t answer confidently, guess, and move on.

Since you know you’re looking for adjective/ adverb errors, they should have been easy to find. In the first sentence, the adverb federally (note the -ly ending) is used to modify the noun tax, when the adjective federal is required. Sentence 2 also uses an adverb (violently) to describe a noun (scenes). In sentence 3, the verb armed is described using the adjective inadequate.
Change the adjective to the adverb inadequately to correct the error.
Some of the most confusing adjectives and adverbs are included on the list of misused words on page 23. Note the differences between good and well, feel bad and feel badly, and more and most. There’s a good chance you’ll see them on the test.
Another frequently tested issue with adjectives and adverbs is comparisons. Recall that the comparative form (–er) is used to compare two things. The

Tip
About 20%, or one in five, of sentences will be error free. If the sentence looks and sounds right to you, choice e (no error) is probably correct.

30

– THE MULTIPLE-CHOICE SECTION –

6. Trinny relented and finally gave us the recipe for a b that amazing dessert she made last Saturday, and c we can’t hardly wait to try it. No error. d e

Practice Identifying Sentence
Errors

Directions: Determine which underlined portion, if any, contains an error in grammar or usage. If the sentence is correct, choose answer e, “No error.” Answers appear at the end of this chapter.

7. No matter how careful the students handle the a b c beakers, every year a few get broken. No error. d e

1. Sheila knew it was important to do well, so for a weeks before the exam, she fretted, worried, b c and was feeling anxiety. No error. d e

8. In contrast to the high-profile company CEO, a b the local business owner did not receive an hono-

2. Since the weather has improved, there are less a b people interested in whiling away the hours in c the dark of the movie theater. No error. d e

rarium for speaking at the monthly meeting of c the area business leaders’ society. No error. d e
9. After three days of heavy rain, as predicted the a b river overflowed its banks, and the water creeped c d up to our front door. No error. e 3. After he got on the train, he realizes the report a he needed to work on was still sitting on his desk b c back at the office. No error. d e

10. He’s doing well following the court hearing that a b determined whether he should face any criminal c d charges. No error. e 4. Only one of the students have finished the book, a b and it has been three weeks since it was assigned! c d
No error. e 5. Before boarding the cross-town bus, everyone a b c must have his or her ticket. No error. d e

31

– THE MULTIPLE-CHOICE SECTION –

Improving Sentences

inal version is the best (the most clear, concise, and correct) version of the sentence. Answer choices b–e will offer different versions of the underlined portion of the sentence. Your task is to determine which choice offers the best version of the sentence.
Some choices will correct or improve the original problem, if there is one. Some will continue to make the same mistake and/or introduce new ones. Only one choice will be both grammatically correct and the most clear and concise way to express the idea.
If it sounds like Improving Sentences questions are a little more complex than Identifying Sentence
Errors, you’re right. Instead of focusing on individual words or phrases to determine the error, you need to look at larger structural and stylistic issues within the sentence to determine the correct answer. Finding that answer requires two distinct steps: (1) determining what, if anything, is wrong with the underlined portion of the sentence and (2) deciding which answer choice fixes that mistake and does not introduce a new mistake. Improving Sentences questions cover a wide range of issues, including grammar and usage, sentence structure and organization, and logic and style. You’ll be given a sentence with a portion underlined, and have to determine whether that portion contains an error. If it does, you must choose the best one of four versions of the sentence. To do so, you must consider not only what is correct but also what is the most clear and effective way to express an idea.
We’ll first look at how these questions are structured, then review the writing issues you’re most likely to encounter. Before you try your hand at ten Improving Sentences questions, you’ll learn the best strategies for approaching them.
Question Structure

In each Improving Sentences question, part or all of the sentence will be underlined. Choice a will repeat the original underlined text. Approximately one in five times, choice a is the correct answer because the orig-

32

The Top Seven Errors in Improving Sentences
The errors in Improving Sentences cover a wide range of writing issues, including grammar and usage, sentence structure and organization, and logic and style. Fortunately, the ETS likes to focus on a handful of specific mistakes. You can expect to see these seven kinds of errors—some of them many times—on test day:
1. improper coordination or subordination of ideas
2. incorrect sentence boundaries
3. faulty comparisons
4. misplaced modifiers
5. wordiness
6. incorrect use of the passive voice
7. incorrect punctuation
The eight most common errors are covered in the Sentence Errors section!

There are two distinct ideas here: (1) It was snowing and (2) I wore my boots. But the relationship between these ideas isn’t correctly expressed by the coordinating conjunction but, which expresses contrast. Instead, it needs a conjunction that shows the second idea is a result of the first:

Errors You’re Likely to See

Improper Coordination or Subordination of Ideas
Within sentences, clauses (groups of words containing both a subject and verb) are often connected by coordination (two independent ideas) or subordination
(one idea depends upon the other):

It was snowing, so I wore my boots.
OR
Because it was snowing, I wore my boots.

Coordination: We are going to dinner and then we are going to a movie.
Subordination:After we go to dinner, we are going to a movie.
Before we go to a movie, we are going to dinner.

Here’s another example:
Henry tried to read War and Peace in the original Russian, and it was too difficult.
What’s the relationship between the two ideas?
Contrast. And expresses addition. Here are three corrected versions:

One of the favorite issues in Improving Sentences questions is coordination and subordination, because it tests your ability to see logical relationships between ideas. To tackle these questions, you need to determine how the ideas in the clauses work together. Is one idea in addition to the other? In contrast? Is there a progression in time or sequence? How exactly does one idea relate to the other? Here’s an example:

Henry tried to read War and Peace in the original Russian, but it was too difficult.
Although Henry tried to read War and Peace in the original Russian, it was too difficult.
Henry tried to read War and Peace in the original Russian, however it was too difficult.

It was snowing, but I wore my boots.
33

– THE MULTIPLE-CHOICE SECTION –

tionship, but only one will be correct, clear, and concise.
For your review, here are the most common coordinating and subordinating conjunctions:

Because the same idea can often be expressed with several different conjunctions, be sure you choose the one that fits the 3C’s (correct, clear, and concise).
There might be two versions that express the right relaCOORDINATING CONJUNCTIONS

and

for

but

nor

or

so

yet
SUBORDINATING CONJUNCTIONS

after

rather than

although

since

as

so that

as if

than

as long as

that

as though

though

because

unless

before

until

even if

when

even though

whenever

if

where

if only

whereas

in order that

wherever

now that

while

once

34

– THE MULTIPLE-CHOICE SECTION –

We were hungry and John was tired so we had to stop at the first rest area that we saw.

Incorrect Sentence Boundaries
A complete sentence requires a noun and verb, and expresses a fully developed thought. The two most common mistakes at the sentence level are extremes.
Sentence fragments stop too quickly; they are phrases that are not whole thoughts. Run-on sentences don’t stop soon enough; they include two or more complete clauses or sentences.
Sentence fragments are often missing a subject or verb, and may be dependent clauses. They can also be phrases or parts of other sentences. Fragments are punctuated as sentences, so they can sometimes be difficult to identify. Even though they don’t express complete thoughts, they can be long and appear to be correct. Here are a few examples:

Kim studied hard for the test that’s why he got an A.
Patty took flying lessons every Saturday so she couldn’t go to the picnic and she couldn’t go to the graduation party either but she has already signed up for another group of flying lessons because she likes it so much.
If you suspect a run-on sentence, determine if there are two independent ideas that can stand alone
(just because a sentence is long doesn’t mean it’s a runon). Check the answer choices for one of the following fixes for run-on sentences:

Because she had to stop studying and go to lacrosse practice.

1. Separate the clauses with a period. We are here.
You are not.
2. Connect the clauses with a comma and a coordinating conjunction (and, or, nor, for, but, so, or yet). Make sure the coordinating conjunction expresses the right relationship between the two ideas. We are here, but you are not.
3. Connect the clauses with a semicolon (and possibly a conjunctive adverb such as however, therefore, or otherwise, making sure it expresses the right relationship between the two ideas). We are here; you are not.
4. Make one sentence dependent upon the other by using a subordinating conjunction such as although, because, since, or while. Again, make sure the subordinating conjunction expresses the right relationship between the two ideas.
Although we are here, you are not.

Cried a lot.
When we finished the game after the sun began setting. If you suspect a fragment, look for the version
(choice b, c, d, or e) that expresses a complete thought.
This might require adding a subject or a verb, deleting a subordinating conjunction (because, while), deleting a relative pronoun (who, that, which), or connecting a dependent clause to an independent clause. The fragments above can be corrected as follows:
She had to stop studying and go to lacrosse practice. Sheu Ling cried a lot.
We finished the game after the sun began setting. The context of the sentence will determine the best correction. If the relationship between the clauses needs to be expressed, then the run-on needs a conjunction of some sort. The run-ons above can be corrected as follows:

Run-on sentences are made up of two or more independent clauses or complete sentences placed together into one sentence without proper punctuation. For example:
35

– THE MULTIPLE-CHOICE SECTION –

I’m more interested in the shoe sale at Macy’s than in the white sale at Walmart.
OR
I’m more interested in Macy’s shoe sale than in
Walmart’s.

We were hungry and John was tired, so we had to stop at the first rest area that we saw.
Kim studied hard for the test; that’s why he got an A.
Patty took flying lessons every Saturday so she couldn’t go to the picnic. She couldn’t go to the graduation party either, but she has already signed up for another group of flying lessons because she likes it so much.

Misplaced Modifiers
A modifier is a word, phrase, or clause that describes another part of a sentence. A misplaced modifier is simply in the wrong place in the sentence. The danger of misplaced modifiers is that they confuse meaning:

Faulty Comparisons
Faulty comparisons are errors in sentence logic. They’re often tough to catch because they sound okay; many people speak in faulty comparisons all the time. Here’s an example:

I had to have the cafeteria unlocked meeting with student government this morning.
Did the cafeteria meet with student government?
To say exactly what is meant, the modifying phrase meeting with student government should be moved to the beginning of the sentence.

I’ve seen every painting by Jackson Pollack, and they’re better than any other painter.

Meeting with student government this morning, I had to have the cafeteria unlocked.

You probably understood the sentence to mean that the paintings by Pollack are better than the paintings by any other artist, but that’s not what the sentence says. The author is actually comparing the paintings of
Pollack to the other painters, not their paintings. To correct faulty comparisons, like things must be compared.

Wordiness
Whether it’s the main mistake in the original prompt or a flaw in one or more of the distracters, unnecessary wordiness is a common error in Improving Sentences questions. In general, the more concise, the better (as long as all necessary information is conveyed).
Wordiness has many causes, including:

I’ve seen every painting by Jackson Pollack, and they’re better than any other painter’s.
OR
I’ve seen every painting by Jackson Pollack, and they’re better than paintings by any other artist.




Here’s another example:


I’m more interested in the shoe sale at Macy’s than in Walmart.



This sentence compares the shoe sale to Walmart rather than to a sale at Walmart. Here’s the kind of fix to look for:

36

“clutter” phrases such as “because of the fact that” that, which, and who phrases (turn them into adjectives: “the manual that is helpful” becomes
“the helpful manual”) unnecessary repetition (e.g., “the meeting is at 4
P.M. in the afternoon”—4 P.M. is in the afternoon) inexact phrases (“I am not in agreement” vs.
“I disagree”; “she was very upset” vs. “she was devastated”) – THE MULTIPLE-CHOICE SECTION –

Here are examples of wordiness, with their more concise counterparts (if there is no concise example, the word or phrase is unnecessary): in the near future (soon) it is clear that last but not least (finally) on a daily basis (daily) on account of the fact that (because) particular somewhere in the neighborhood of (about) take action (act) the fact that the majority of (most) the reason why (the reason or why) through the use of (through) with regard to (about or regarding) with the exception of (except for)

a lot of (many or much) all of a sudden (suddenly) along the lines of (like) are able to (can) as a matter of fact as a whole as the case may be at the present time (currently or now) both of these (both) by and large by definition due to the fact that (because) for all intents and purposes in order to (to) in the event that (if)

mathematics is a field of study, so it does not need to be modified with the words field of. Review these lists of repetitive phrases and be ready to spot them more easily in SAT prompts and answer choices.

Some words and phrases don’t need a modifier, because the specific is implied in the general. For instance, the word consensus means general agreement. Therefore, modifying it with the word general is repetitive. Similarly,

RETAIN ONLY THE FIRST WORD

DROP THE MODIFIER (FIRST WORD)

any and all

past memories

first and foremost

final destination

refer back

general consensus

close proximity

various differences

large in size

each individual

often times

basic fundamentals

reason why

true facts

heavy in weight

important essentials

period in time

future plans

round in shape

terrible tragedy

37

– THE MULTIPLE-CHOICE SECTION –

RETAIN ONLY THE FIRST WORD

DROP THE MODIFIER (FIRST WORD)

odd in appearance

end result

mathematics field

final outcome

cheap quality

free gift

honest in character

past history

confused state

totally obvious

modern in design

rarely ever

unusual in nature

unexpected surprise

extreme in degree

sudden crisis

strange type

a. The American Red Cross offers a program called the Learn to Swim Program that begins with a class called
b. The American Red Cross’s Learn to Swim Program begins with a class called
c. The American Red Cross offers a program that is called the Learn to Swim Program that begins d. The American Red Cross is an organization that offers a program that is called the Learn to Swim Program that begins with a class called e. The American Red Cross offers a program called the Learn to Swim Program that begins with a class called

Another common problem that leads to wordiness is the use of unnecessary prepositions. When two or more prepositions are used together, chances are at least one is unnecessary.
I cleaned up under the kitchen cabinets.
She likes all sports except for soccer.
In both of these sentences, there is an unnecessary preposition. Here’s how to correct them:
I cleaned under the kitchen cabinets.
She likes all sports except soccer.
Notice how choices a, c, d and e in the following example all suffer from wordiness. They all use that clauses and repeat words such as program and called:

The American Red Cross offers a program called the Learn to Swim Program that begins with a class called Introduction to Water Skills, then progresses to Fundamental Aquatic Skills.

38

– THE MULTIPLE-CHOICE SECTION –

Incorrect Use of the Passive Voice
You may find one or more prompts or answer choices that use the passive when the active voice is needed. In the passive voice, the subject (most often you) is acted upon. While there are occasions in which it’s correct to use it, most sentences should be in the active voice. Passive constructions tend to be wordy or lack focus. Compare these sentences:
Active:
Passive:



1. There are three ways to use semicolons to separate independent clauses.
First Case: Use a semicolon to separate independent clauses joined without a conjunction.

My friend asked for another helping. Another helping was asked for by my friend.

Active:
Passive:

Example:
Four people worked on the project; only one received credit for it.
Second Case: Use a semicolon to separate independent clauses that contain commas, even if the clauses are joined by a conjunction.

I misplaced my wallet.
My wallet was misplaced by me.

Active:

The administration has selected three finalists for the open position. Three finalists for the open position have been selected by the administration. Passive:

Semicolon Errors
Semicolons (;) are used in two ways: to separate independent clauses and to separate the items in a list when those items contain commas.

Example:
The strays were malnourished, dirty, and ill; but
Liz had a weakness for kittens, so she adopted them all.
Third Case: Use a semicolon to separate independent clauses that are connected with a conjunctive adverb that expresses a relationship between clauses.

Note the simplicity and directness of the first sentence in each pair. The second sentences, written in the passive voice, are clunky and noticeably longer. With very few exceptions, sentences like these should be quickly eliminated; they’re almost always wrong.

Example:
Victoria was frequently tardy; therefore, she received a low grade.
2. Use semicolons to separate items in a series that contain commas to show which sets of items go together. Incorrect Punctuation
Identifying Sentence Errors questions don’t test for it, but Improving Sentences questions do. (The good news is, though, that neither tests for spelling!) Keep in mind when answering these questions that a misplaced or missing comma, an errant apostrophe, or an unnecessary semicolon could be the error you’re looking for.
There are dozens of rules about the many different punctuation marks in the English language. Fortunately, the punctuation errors on the SAT tend to stick to three categories: semicolon, comma, and apostrophe errors. Examples:
The dates for our meetings are Monday, January 10; Tuesday, April 14; Monday, July 7; and
Tuesday, October 11.
She has lived in Omaha, Nebraska; Nutley, New
Jersey; Amherst, Massachusetts; and Pensacola,
Florida.

39

– THE MULTIPLE-CHOICE SECTION –



Here, the purpose of the sentence is to explain who wrote the prescription, so that clause is essential.
The context of the sentence will help you determine whether information is essential and therefore whether commas are needed.

Comma Errors
There are many rules about when to use and when not to use commas. Here are the five comma errors you are most likely to see on the exam:

1. Comma between subject and verb. When a subject is immediately followed by its verb, nothing should come between them:

4. Comma separating two independent clauses.
Known as a comma splice, this error is the incorrect use of a comma to connect two complete sentences. It creates a run-on sentence. To correct a comma splice, you can either:

Mary decided to relax with a good book.
2. No comma after introductory phrase or clause.
Introductory phrases and clauses should be followed by a comma:



By lunchtime, Aidan had already finished his project. ■


After a long day at work, Mary decided to relax with a good book.

replace the comma with a period, forming two sentences replace the comma with a semicolon join the two clauses with a conjunction such as and, because, or so
Comma splice: Our school received an award, we raised the most money for the local charity.
Corrected sentence: Our school received an award. We raised the most money for the local charity. OR
Our school received an award; we raised the most money for the local charity.
OR
Our school received an award because we raised the most money for the local charity.

3. No comma around “interrupters.” Words, phrases, and clauses that “interrupt” the core sentence (and are not essential to the meaning of that core sentence) should be set off by commas:
Mary, a pediatrician, really enjoys her work.
The phrase a pediatrician is an “interrupter”: It’s not essential to the sentence. We could take it out and the sentence would still be a complete, grammatically correct idea. Thus, it needs to be set off with commas.
Here’s another example:
Mary, who always wanted to be a pediatrician, loves her job.
In the following example, the who clause is essential to the sentence and should not be set off with commas: Mary is the one who wrote the prescription.

40

Its vs. It’s
Unlike most possessives, its does not contain an apostrophe. The word it’s is instead a contraction of the words it is. The second i is removed, and replaced by an apostrophe. When revising your writing, say the words it is when you come across it’s or its. If they make sense, you should be using the contraction. If they don’t, you need the possessive form, its, without an apostrophe.



4. Possessive plural nouns already ending in s need only the apostrophe added:

Apostrophe Errors
Apostrophes are used to form contractions, indicate ownership, and form certain plurals. Review these six rules for their use.

The customers’ access codes are confidential.
The students’ grades improved each semester.

1. Apostrophes form contractions by taking the place of a missing letter or number:

The flight attendants’ uniforms were blue and white. We’re going out of town next week.

5. Show possession in the last word when using names of organizations and businesses, in hyphenated words, and in joint ownership:

Don’t write the proposal without the instructions from your boss.
My husband was in the class of ’89.

Brad and Janet’s graduation was three months ago. 2. Add ’s to form the singular possessive, even when the noun ends in s:

I went to visit my great-grandfather’s alma mater. The school’s lunchroom needs to be cleaned.

The Future Farmers of America’s meeting was moved to Monday.

The drummer’s solo received a standing ovation. 6. Use an ’s to form the plurals of letters, figures, and numbers used as words, as well as certain expressions of time and money. The expressions of time and money do not indicate ownership in the usual sense:

Mr. Perkins’s persuasive essay was very convincing. 3. A few plurals that don’t end in s also form the possessive by adding ’s:

She has a hard time pronouncing s’s.

The children’s toys were found in every room of the house.

My street address contains three 5’s.

The line for the women’s restroom was too long.

He packed a week’s worth of clothing.

Men’s shirts come in a variety of neck sizes.

The project was the result of a year’s worth of work. 41

Now What?
But what if you can’t identify the error? Here are three strategies:
1. Let the choices guide you. Scan each version to see what aspect of the original sentence is changed and how. The way the original is rewritten will often reveal the nature of the error in the original prompt.
2. Look for the most commonly tested errors. There are seven kinds of mistakes that pop up most often in Improving Sentences questions. If you are having trouble finding an error, do a quick check for the common errors described in this section.
3. Guess or move on. If you can eliminate at least one choice as obviously wrong, guess. If not, skip the question and move on.

Strategies for Improving
Sentences






Locate the error. Try to determine the error as you read the sentence. What’s wrong with the underlined portion? Is it a run-on sentence, does it have faulty parallelism, or does it have unnecessary wordiness? If you determine there is no error
(remember, approximately 20% of the questions will be error-free), select choice a and move on to the next question.
Eliminate all choices with the original error. If you identify an error, eliminate choice a (don’t even bother reading it; it only repeats the original prompt). Then, eliminate any other choices that make that same mistake.



42

Eliminate all choices that make other errors.
From the remaining choices, eliminate any versions that make a different error, even if they correct the error in the prompt. This includes any versions that are grammatically correct but are unnecessarily wordy, ambiguous, or use unnecessarily complicated sentence structure.
Find the best sentence using the 3C’s. From the choices that remain, select the one that is correct
(no grammar or usage errors or lapses in logic), clear (no ambiguity or tangled sentence structure), and concise (no unnecessary wordiness).

– THE MULTIPLE-CHOICE SECTION –

13. Seated high in the arena, the rock band looked like toy figures to the audience members with the cheapest tickets.
a. Seated high in the arena, the rock band looked like toy figures to the audience members with the cheapest tickets.
b. Being seated high in the arena, the rock band looked like toy figures to the audience members with the cheapest tickets.
c. The rock band looked like toy figures to the audience members with the cheapest tickets, who were seated high in the arena.
d. The rock band looked like toy figures to the audience members with the cheapest tickets, seated high in the arena.
e. Seated high in the arena, to the audience members with the cheapest seats, the rock band looked like toy figures.

Practice Improving Sentences
Questions

Directions: In each of the sentences that follow, part or all of the sentence is underlined. The underlined text may contain an error in sentence construction, grammar, word choice, or punctuation. Choice a repeats the original underlined text. If there is no error in the underlined portion, choose a. If there is an error, select the answer choice that most effectively expresses the meaning of the sentence without any ambiguity or awkwardness. Answers can be found at the end of the chapter. 11. I’m getting forgetful; yesterday, my wallet was misplaced by me.
a. forgetful; yesterday, my wallet was misplaced by me.
b. forgetful; yesterday, my wallet is misplaced by me. c. forgetful; yesterday, I misplaced my wallet.
d. forgetful; yesterday, I was so forgetful that I misplaced my wallet.
e. forgetful. Yesterday, my wallet was misplaced by me.

14. Valerian is the name of two different plants: one of the most popular medicinal herbs in the
United States, whereas in Honduras it is a grass that grows to be three to four feet tall.
a. whereas in Honduras it is a grass that grows to be three to four feet tall.
b. and a grass that grows to be three to four feet tall in Honduras.
c. but in Honduras, it is a grass that grows to be three to four feet tall.
d. in contrast, in Honduras, it is a grass that grows to be three to four feet tall.
e. and in Honduras, it is a plant that is a grass that grows to be three to four feet in height.

12. In 1985, the few families that had a computer used them mainly for word processing and game playing, although most families today have Internet access that allows them to e-mail and surf the web.
a. although most families today have Internet access that allows them to e-mail and surf the web.
b. while most families today have Internet access that allows them to e-mail and surf the web.
c. however, today, most families have Internet access that allows them to e-mail and surf the web.
d. most families today have Internet access that allows them to e-mail and surf the web.
e. although for most of today’s families, there is
Internet access that allows them to e-mail and surf the web.
43

– THE MULTIPLE-CHOICE SECTION –

16. Once known as a fringe treatment, meditation has been scientifically proven to help manage the symptoms of depression.
a. Once known as a fringe treatment, meditation has been scientifically proven to help manage the symptoms of depression.
b. Being known as a fringe treatment, many people who suffer from depression have been able to help manage their symptoms by meditating. c. Meditation is no longer thought of as a fringe treatment; managing the symptoms of depression has been scientifically proven.
d. The symptoms of depression have been scientifically proven to be managed through meditation, once thought of as a fringe treatment.
e. Scientifically proving that meditation, once known as a fringe treatment, can help manage the symptoms of depression.

15. Iridology, the study of the colored part of the eye, known as the iris, to diagnose health problems, had its first medical reference in a German physician’s 1670 book Chiromatica Medica.
a. Iridology, the study of the colored part of the eye, known as the iris, to diagnose health problems, had its first medical reference in a German physician’s 1670 book Chiromatica
Medica.
b. Developing long ago, in 1670, iridology, which is the study of the colored part of the eye, known as the iris, to diagnose health problems.
c. While it had its first medical reference in a German physician’s 1670 book Chiromatica
Medica, iridology is the study of the colored part of the eye, known as the iris, to diagnose health problems.
d. The study of the colored part of the eye, known as the iris, to diagnose health problems, iridology had its first medical reference in a German physician’s 1670 book Chiromatica Medica.
e. The first medical reference to iridology, the study of the iris to diagnose health problems, was in a German physician’s 1670 book Chiromatica Medica.

17. Free verse is poetry without regular form, although sonnets are organized into three or four stanzas followed by a couplet.
a. although sonnets are organized into three or four stanzas followed by a couplet.
b. while sonnets are organized into three or four stanzas followed by a couplet.
c. likewise sonnets are organized into three or four stanzas followed by a couplet.
d. since those that are organized into three or four stanzas followed by a couplet, sonnets.
e. poetry organized three or four stanzas followed by a couplet is called a sonnet.

44

– THE MULTIPLE-CHOICE SECTION –

20. According to a recent survey, 36% of Americans cook vegetarian meals; significantly more than the previous decade.
a. meals; significantly more than the previous decade. b. meals, a significant increase over numbers from the previous decade.
c. meals, which is a significant increase over the previous decade.
d. meals, which, compared to the previous decade, is a significant increase in number.
e. meals, previously in the last decade the numbers were much higher.

18. First-generation Vietnamese-American Monique
Truong blends fact and fiction, history and speculation in The Book of Salt; a novel inspired by a reference to an Indochinese cook in The Alice B.
Toklas Cook Book.
a. The Book of Salt; a novel inspired by a reference to an Indochinese cook in The Alice B.
Toklas Cook Book.
b. The Book of Salt, which was a novel that was inspired by a reference to an Indochinese cook in The Alice B. Toklas Cook Book.
c. her novel The Book of Salt, which was inspired by a reference to an Indochinese cook in The
Alice B. Toklas Cook Book.
d. her novel, The Book of Salt; which was inspired by another book, The Alice B. Toklas Cook
Book, which mentioned an Indochinese cook.
e. her novel The Book of Salt, it was inspired by a reference to an Indochinese cook in The Alice
B. Toklas Cook Book.

Improving Paragraphs
These questions are the toughest, and most timeconsuming, questions you’ll encounter in the Writing section. You’ll be asked about ways in which a rough draft of a short essay can be improved. Don’t worry so much about what the passage says; your job is to choose the best ways to improve how it says it.
The draft will be followed by questions that cover a range of writing issues, from the sentence level (grammar and usage, sentence structure, word choice, etc.), to the paragraph level (paragraph divisions, transitions, paragraph unity), to the essay level (overall organization, development, and support). The questions are designed to measure your ability to identify weaknesses and improve the writing in a text.
This section describes the question format, including the kinds of questions to expect, the errors you’re likely to see in the passages, and strategies to determine the best answer for each question.

19. Held in 1883, President Chester Arthur presided over the ceremony to officially dedicate the
Brooklyn Bridge.
a. Held in 1883, President Chester Arthur presided over the ceremony to officially dedicate the Brooklyn Bridge.
b. Held in 1883, it was President Chester Arthur who presided over the ceremony to officially dedicate the Brooklyn Bridge.
c. The Brooklyn Bridge was officially dedicated in 1883, at a ceremony that was presided over by President Chester Arthur.
d. President Chester Arthur presided over the
1883 ceremony that officially dedicated the
Brooklyn Bridge.
e. The 1883 ceremony, presided over by President Chester Arthur, which officially dedicated the Brooklyn Bridge.

Question Structure

The Improving Paragraphs section begins with a short passage (typically two to four paragraphs long). The passage will most likely be the draft of an essay, but you might also find a letter to the editor, an excerpt from a

45

– THE MULTIPLE-CHOICE SECTION –

a. We were divided by ability into different classes, each of which focused on, during the camp week, improving a set of skills.
b. We were divided into different classes, and by ability we focused on a set of skills so that we could improve on them over the course of the week at camp.
c. We were divided into different classes and our abilities to focus on a set of skills worked to improve those skills during the camp week.
d. Divided into different classes based on ability, we focused on the camp week to improve a set of skills.
e. After we were divided into different classes based on ability, we focused on a set of skills to improve on during the camp week.

memo, or another type of general writing. Because the passage is a draft, it will need improvement on many levels. Following the passage will be a series of questions about how to improve the passage. These questions can be divided into three general categories: revising individual sentences, revising sentence pairs, and revising the big picture. You can expect the five or six
Improving Paragraphs questions to be divided equally among these three question types.
Revising Individual Sentences
These questions refer to a specific sentence within the passage and ask you to determine the most effective revision of that sentence. They are essentially the same as those in Improving Sentences, with the exception that you will often need to consider the context of the passage to determine the correct answer. For example, the alternate versions of the selected sentence may offer different transitions from the previous sentence, but only one will be correct, concise, and the most appropriate way to move from one idea to another.

The correct answer is e, which expresses the ideas of the sentences more clearly and concisely than any of the other choices.
Revising the Big Picture
“Big picture” questions ask about paragraph-level and essay-level issues such as organization and writing strategies. Thus, the format and writing issues can vary greatly. Here’s a sample big picture question:

Revising Sentence Pairs
These questions refer to two sentences within the passage and ask you to determine the most effective revision and/or combination of those sentences. These questions are also similar to Improving Sentences, often focusing on establishing the right relationship (coordination/subordination) and proper boundaries between the two sentences. Here’s an example:

Which of the following is the most logical order of the paragraphs?
a. 1, 2, 3, 4
b. 1, 3, 2, 4
c. 2, 3, 4, 1
d. 4, 3, 2, 1
e. 1, 4, 2, 3

Which of the following is the most effective combination of sentences 10 and 11 (reprinted below)?
(10)Our group was divided by ability into different classes. (11)Each class focused on a set of skills and worked to improve those skills during the camp week. 46

Caution: Question Formats Vary!
Read each question carefully. Unlike Identifying Sentence Errors and Improving Sentences questions, the prompts and answer choices for Improving Paragraphs will vary. All will offer five choices (a–e), but choice a will not always repeat the original text.

lem, comparing two items when only one has been mentioned, ignoring issues of chronology—these are all examples of illogical organization. The most common organizational methods include: order of importance, chronology, cause and effect, and comparison and contrast.
Underlying the organizational pattern is the basic essay structure, assertion-support. That is, an essay has a main idea, which should be stated near the beginning, and the rest of the essay serves to develop and support that idea. The same structure is repeated in each paragraph; there is one main idea, often expressed in a topic sentence, and the rest of the paragraph supports that idea.
A question that asks “Which would be the most effective order of paragraphs?” tells you to look carefully at the organizational pattern. Are the paragraphs out of chronological order? Does the discussion of X interrupt the discussion of Y? Does the paragraph start with specific examples, make a general statement, and then go back to providing more examples? Look for these types of problems when you encounter a question about organization. Errors You’re Likely To See

You’re now familiar with the types of questions you will encounter. But what kinds of issues will those questions most likely deal with? All of the errors that appear in
Identifying Sentence Errors and Improving Sentences are fair game in Improving Paragraphs. Expect to see problems with word choice and sentence-level issues such as adjective/adverb confusion, verb tenses, faulty comparisons, and improper use of the passive voice.
In addition, there will be six new types of questions to expect, including those about effective paragraphing, organization of ideas, transitions, cohesion, development of ideas, and style.
Effective Paragraphing
A paragraph by definition is a group of sentences about one idea. Long paragraphs often contain more than one main idea and should usually be divided to improve readability and unity of ideas. A question about effective paragraphing might be worded as follows:
The author wishes to divide paragraph 2 into two paragraphs. After which sentence should the author begin a new paragraph?

Transitions
Transitions are words, phrases, and sentences that show the relationship between ideas, and lead from one idea to another, such as meanwhile, however, after, or in contrast. A paragraph that needs a stronger transition is a likely candidate for an Improving Paragraphs question. Here are a couple of examples:

This question requires that you look for a turning point in the paragraph—a place where the topic shifts, and a new idea is introduced.
Organization of Ideas
Paragraphs and essays can be organized in countless ways, and many of them are correct. Problems arise, however, when the organization isn’t logical. Explaining a solution to a problem before detailing the prob-

47

– THE MULTIPLE-CHOICE SECTION –

The author’s argument could best be expanded by which of the following statements?

Which of the following phrases should be added to the beginning of sentence 4 to link it to sentence 3?

Which of the following sentences, if added to paragraph 3, would provide the best support for the main idea?

Which of the following sentences, if added to the end of paragraph 1, would most effectively link the paragraph to the rest of the essay?

Which best describes the relationship of sentence 7 to sentence 6?

These questions tell you that you need to look for the relationship between sentences 3 and 4 or paragraphs 1 and 2. Does paragraph 2 offer another example? Does it describe a different point of view? When you understand the relationship, select the sentence that best expresses it.

For the last type of question, you will be asked to choose from a list of answers such as: It is an example, it contradicts the argument, it confirms the claim, it adds information, or it draws a conclusion.
Style
The last type of error you may encounter involves stylistic issues such as word choice, tone, or level of formality. Here are two examples:

Cohesion
As stated earlier, a paragraph is a group of sentences about the same idea. Frequently, a passage will include one or more sentences that stray from the main idea of the paragraph or essay. To improve the cohesion of ideas, off-topic sentences should be deleted or moved to another, more relevant section. Here’s how a question about cohesion might be phrased:

The author wishes to alter the tone of sentence 12.
Which of the following revisions would most suit the overall tone of the essay?
Which of the following offers the most effective revision of sentence 6 (reprinted below)?

The deletion of which sentence would most improve the second paragraph?

(6)I can’t tell you how much I learned by reading
Macbeth.
a. Macbeth really had a big impact on me.
b. I learned tons by reading Macbeth.
c. Macbeth taught me an invaluable lesson about the dangers of ambition.
d. Macbeth is a play that I read that I learned a lot from. e. Reading Macbeth was an extremely knowledgeable experience.

Development of Ideas
An idea is properly developed in three steps: first, it’s introduced with some explanation; second, more details and/or examples are given; third, a conclusion is drawn. In Improving Paragraphs questions, development is tested in a number of ways. An introductory or concluding sentence could be missing, and you’ll be asked to choose one. Or, you may be asked to select the most logical information to add to a paragraph. Other questions will ask you to consider which sentence from a list might best serve to further develop an idea. The key is to look at the logical relationships between ideas and to remember the overall assertion-support structure of essays. You might find prompts like the following: Only choice c really improves the sentence. Why?
Because invaluable lesson about the dangers of ambition is more specific and exact than the other versions—it tells what was learned. Choice c is also written in a more formal tone than the other choices, which is more

48

– THE MULTIPLE-CHOICE SECTION –

4. Remember the 3C’s. For questions that ask you to revise sentences, use the 3C’s as your guide. Choose the version that is correct (no grammar, usage, or logic errors), clear (no ambiguity or confusing sentence structure), and concise (no unnecessary wordiness).
5. Study the most common question topics. This will help you know what to expect and what to look for as you read the passages.
6. Save the big picture questions for last. They’re usually the most time-consuming Improving Paragraphs question type.

suited to a discussion of literature. The other choices all contain slang or other informal idioms.
Strategies for Improving
Paragraphs

Improving Paragraphs are more difficult than Identifying Sentence Errors and Improving Sentences questions; you’re dealing with a passage and three different levels of its composition. But these questions aren’t impossible. They focus only on one step of the writing process, revision. And there are only a few kinds of questions and errors you should expect. The following strategies will help you spend your time on these questions most effectively, helping you score the most points in a reasonable amount of time.

Practice Improving Paragraphs
Questions

Directions: Questions 21–25 are based on the following passage, a first draft of an essay about the evocation of mood and emotion in Edgar Allan Poe’s “The TellTale Heart.” Read the passage and the questions that follow. For each question, choose the answer that will most improve the passage. Some questions ask you to choose the best revision of a particular sentence or pair of sentences. Other questions ask you to consider how to best improve the overall organization of the passage. In each case, the correct answer is the one that most closely conforms to the conventions of formal writing. The answers are at the end of the chapter.

1. Do Improving Paragraphs last. This is your most important strategy for the Writing section’s multiplechoice questions! Improving Paragraphs is the smallest question category on the exam (only about 5 questions) and these questions take the longest to complete—so save them for last. Do Identifying Sentence Errors first and then Improving Sentences to answer the most questions in the least amount of time and earn the greatest number of points.
2. Scan the questions before you read the passage.
The draft contains many more errors than you will be asked about. Reading the questions first can help you focus on the mistakes that you will need to revise and not be distracted by the other weaknesses and errors. 3. Read the questions carefully. The questions will tell you the specific lines to revise and the specific writing issue(s) that need to be addressed. For example, if a question asks, “Which phrase, if added to the beginning of sentence 2, would most improve the essay?” you know you need to determine the relationship between sentences 1 and 2 and then find the best transition. (1)Writers have to be very skillful in word choice in order to evoke emotions. (2)As I explored Edgar
Allan Poe’s works, I became intrigued with the way
Poe carefully chose language and how it elicits specific feelings.
(3)Poe’s “The Tell-Tale Heart” involves a young man who rents a room from an elderly man in a large, dark mansion. (4)Descriptions of this eerie setting help Poe create the mood. (5)He masterfully builds suspense in “The Tell-Tale Heart.” (6)You realize he is capable of anything. (7)For example, the tenant opens his landlord’s bedroom door at night

49

– THE MULTIPLE-CHOICE SECTION –

22. Which of the following revisions is the best way to combine sentences 4 and 5?
(4)Descriptions of this eerie setting help Poe create the mood. (5)He masterfully builds suspense in
“The Tell-Tale Heart.”
a. In “The Tell-Tale Heart,” Poe not only creates the mood by describing this eerie setting he also masterfully builds suspense.
b. Creation of mood is achieved through descriptions of this eerie setting, and then Poe masterfully builds suspense.
c. This eerie setting helps Poe create the mood, and set the stage for the suspense he will masterfully build.
d. Poe not only creates the mood by describing this eerie setting, but also in “The Tell-Tale
Heart,” he masterfully builds suspense.
e. Poe simultaneously creates mood and builds suspense in “The Tell-Tale Heart” by describing this eerie setting.

and stares at his glass eye for hours while in a seething rage.
(8)The police investigate the home after a neighbor reported hearing screams. (9)The tenant invites the police into the room where he did hide the corpse that was dismembered. (10)Poe adds to the suspenseful mood because you wonder if the man will confess to murder. (11)While being interrogated, the man hears a faint heartbeat that grows louder. (12)However, he is the only one who hears it.
(13)His attempt to fool the police while sitting on the corpse fails as he mentally breaks down from the noise inside his mind and confesses.
(14)As Poe creates an eerie, suspenseful tone in his fiction, it shows that authors can lead their readers to feel certain emotions through their writing.

21. In the context of the passage, which of the following is the most effective revision of sentence 9
(reprinted below)?
(9)The tenant invites the police into the room where he did hide the corpse that was dismembered.
a. The tenant invites the police into the room where he hid the dismembered corpse.
b. The tenant did invite the police into the room where he did hide the dismembered corpse.
c. The tenant led the police to the room where he did hide the corpse that was dismembered.
d. The tenant invites the police right near where the dismembered corpse was hiding.
e. In a move that only adds to the eerie and suspenseful tone, the tenant invites the police to come into the room where earlier he hid the dismembered corpse.

23. The revision to sentences 11 and 12 that would most improve the essay is:
(11)While being interrogated, the man hears a faint heartbeat that grows louder. (12)However, he is the only one who hears it.
a. Place sentence 12 before sentence 11.
b. Delete the word however, and connect the sentences with the word and.
c. Connect the sentences with a comma.
d. Delete sentence 12.
e. Leave them as is.

50

– THE MULTIPLE-CHOICE SECTION –

Commit To Memor y

24. In the context of the essay, which of the following revisions of sentence 14 more effectively concludes the essay?
(14)As Poe creates an eerie, suspenseful tone in his fiction, it shows that authors can lead their readers to feel certain emotions through their writing.
a. Poe’s building up of an eerie mood and suspenseful tone is a good example of how authors can lead their readers to feel certain emotions through their writing.
b. The powerful conclusion of “The Tell-Tale
Heart” shows that strong emotions can be felt by readers of fiction when the author, such as
Poe, creates an eerie, suspenseful tone.
c. Poe’s creation of an eerie, suspenseful tone shows that authors can lead their readers.
d. The powerful conclusion of “The Tell-Tale
Heart” is yet another example of Poe’s mastery of language and his ability to evoke emotion.
e. Poe’s masterful creation of mood and tone in
“The Tell-Tale Heart” puts him in a league of other great writers of suspense, such as Sir
Arthur Conan Doyle and Ellery Queen.

Here are the most important lessons from Chapter 2:












25. Which of the following is the most logical order of sentences within paragraph 2?
(3)Poe’s “The Tell-Tale Heart” involves a young man who rents a room from an elderly man in a large, dark mansion. (4)Descriptions of this eerie setting help Poe create the mood. (5)He masterfully builds suspense in “The Tell-Tale Heart.” (6)You realize he is capable of anything. (7)For example, the tenant opens his landlord’s bedroom door at night and stares at his glass eye for hours while in a seething rage.
a. 3, 7, 4, 5, 6
b. 3, 4, 6, 5, 7
c. 5, 4, 6, 7, 3
d. 3, 5, 4, 6, 7
e. 3, 4, 5, 7, 6









51

No matter the order they’re presented in, answer
Writing section multiple-choice questions as follows: Identifying Sentence Errors, Improving Sentences, and Improving Paragraphs.
If you can eliminate one or more responses, but aren’t sure of the answer, guess.
If you’ve spent a few minutes on a question, and still can’t eliminate a response or two, skip it and move on.
About 20% of the Identifying Sentence Errors and
Improving Sentences questions will have no error.
That means for Identifying Sentence Errors questions, you’ll select choice e (“no error”), and for
Improving Sentences questions, you’ll select choice a (same as the original sentence).
Questions are presented from easiest to hardest.
For the hardest questions, expect the answer to be tricky; what at first looks to be the correct answer may not be.
Identifying Sentence Errors questions don’t test your knowledge of spelling or punctuation, so don’t waste time looking for those types of errors.
Don’t waste time reading choice a in Improving
Sentences questions; it merely repeats the original sentence. Target your studies: based on the analysis of your first practice test, pinpoint the areas you’re weakest in, and spend the most time studying them.
Still not sure whether to use lay or lie, who or whom? Now’s the time to get it straight.
The best sentences are those that use the 3C’s: correct (no grammar or usage errors or lapses in logic), clear (no ambiguity or tangled sentence structure), and concise (no unnecessary wordiness).
Read the choices for Improving Paragraphs questions carefully. Some of them may be “No error,” and others will repeat the same error as the original sentence, but these responses could be a, b, c, d, or e.

– THE MULTIPLE-CHOICE SECTION –

Answer Key

Improving Sentences

11. c. Choice a incorrectly uses the passive voice.
Choice b repeats the error, and also uses the wrong verb tense (yesterday calls for the past tense was, not the present tense is). In choice d, extra words are added, and choice e repeats the passive voice error while unnecessarily dividing the sentence into two sentences.
12. b. In choice a, although does not express the correct relationship between the two clauses.
Although does express contrast, which is the logical relationship here, but it belongs with the first clause rather than the second. In choice b, the subordinate conjunction while clearly and effectively expresses the right relationship. Choice c’s use of however is correct, but it is preceded by a comma instead of a semicolon, creating a run-on sentence. Choice c is unnecessarily wordy. Choice d creates a run-on sentence and does not offer a coordinating or subordinating conjunction to express the contrast between the two clauses.
Choice e repeats the error in a and adds unnecessarily wordy constructions.
13. c. Choices a, b, and d have misplaced modifiers; the rock band is not seated high in the arena.
Choice b also includes the ungrammatical phrase being seated. Choice e is wordier than c, which is more direct and logical in structure.
14. b. Coordination, parallel structure, and wordiness are the problems here. Choices a, c, and d use incorrect conjunctions (whereas, but, and in contrast), and they lack parallel structure. In all of them, the elements of the second plant description (type and location) do not match the first. Choice e is wordy and lacks parallel structure.
15. e. Choices a, c, and d are unnecessarily wordy.
Choice b, while also suffering from wordiness, is a sentence fragment.

Identifying Sentence Errors

1. d. The verb was feeling should be in the simple past tense (felt) to maintain consistency with fretted and worried.
2. b. The adjective fewer should be used with the noun people. Less is used for singular nouns that represent a quantity or degree (less salt, less time), while fewer is used to modify plural nouns or things that can be counted (fewer bagels, fewer minutes).
3. a. The verb tenses in this sentence are not consistent. In order to maintain consistency, the present tense realizes should be changed to the past tense realized.
4. b. The verb have finished expresses the action of the noun one. Therefore, it should take the singular form has finished.
5. e. There is no error in this sentence. If you chose d, recall that the pronoun everyone is singular.
The pronoun following this antecedent must agree with it (in this case, the singular pronouns his or her).
6. d. Can’t and hardly are both negatives. When used together, they cancel each other’s meaning. To correct the sentence, either drop hardly, or change can’t to can.
7. b. Careful is an adjective. In this sentence, it incorrectly modifies the verb handle. The correct word is the adverb carefully.
8. a. This is an idiom error; the correct preposition to use after contrast is with.
9. d. The verb to creep is irregular; its past tense form is crept.
10. e. There is no error in this sentence. If you chose a, recall that well is an adverb, and it correctly modifies the verb doing. Good is an adjective, which modifies nouns.

52

– THE MULTIPLE-CHOICE SECTION –

increase is over the decade, which is incorrect.
Choice d repeats the semicolon error and is unnecessarily wordy. Choice e is a run-on sentence, is wordy, and has awkward sentence structure. 21. a. There are two problems with the original sentence, and both are corrected in choice a. The helping verb did (in did hide) is unnecessary; it’s clearer and more concise to say the man hid the corpse. Another instance of wordiness is the phrase corpse that was dismembered.
Recall that that phrases can easily be turned into adjectives: dismembered corpse. Choice b corrects the that phrase, but adds another unnecessary helping verb, did invite. In choice c, neither original problem is corrected.
Choice d uses the informal right near where, and adds confusion—corpses don’t hide themselves. Choice e is wordier than the original sentence.
22. c. These sentences involve two actions: Poe creates mood (by describing a setting), and builds suspense (how he does this is explained in the next sentence). Choice c uses the conjunction and to link them. Choice a seems to link the action with the conjunction not only, but it leaves out the second part of the conjunction pair, but also. It is also a run-on sentence. Choice b includes both actions, but improperly uses the passive voice. In choice d, the conjunction pair not only . . . but also is used. However, it unnecessarily repeats the title of the story, which was just mentioned in the previous sentence. Choice e is wrong because it incorrectly links the description of setting to the creation of suspense. In context, that doesn’t make sense.
23. e. There is nothing wrong with these sentences.
They present critical information in the correct order, and would become a run-on

16. a. This is the most clear and concise version.
Choice b has a misplaced modifier—many people are not the fringe treatment. It is also wordy. The second clause in choice c is untrue. Choice d is unnecessarily wordy, and choice e is a sentence fragment.
17. b. The problem with choice a is proper coordination/subordination. Although does not express the right kind of contrast; free verse has no organized structure while sonnets do— they exist simultaneously. Choice c makes the same mistake with likewise, which expresses similarity. Choice d uses the wrong subordinator (since); unlike would correct it. Choice e omits the subordinating or coordinating word and is a run-on sentence.
18. c. Choice a incorrectly uses the semicolon.
Choice b corrects the semicolon error, but is unnecessarily wordy. Choice c also corrects the semicolon error, and most concisely and clearly expresses the idea. Choice d repeats the semicolon error, has awkward sentence structure, and is wordy. Choice e corrects the semicolon error, but the change in syntax now calls for a semicolon rather than a comma.
19. d. Choice a has a misplaced modifier. The ceremony was held in 1883, not President Arthur.
Choice b retains this error and adds the wordy it was . . . who construction. Choice c is grammatically correct, but not as concise as choice d because it uses the passive voice. Choice e is a sentence fragment; removal of the word which would correct it.
20. b. Choice a incorrectly uses the semicolon and does not clearly indicate what is a significant increase—the percentage of Americans who cook vegetarian meals, or the frequency with which they cook them. Choice b corrects the semicolon error and correctly identifies exactly what the increase is: an increase in numbers. Choice c merely states that the

53

– THE MULTIPLE-CHOICE SECTION –

the action of the story, a reference to this eerie setting makes no sense. Sentence 7 also logically follows sentence 5—it is the example of suspense building mentioned in the previous sentence. Sentence 6 should be after sentence
7. If it appears anywhere else in the paragraph, it causes confusion, because he could refer to
Poe, the young man, or the elderly man.

sentence if they were connected with a comma or other conjunction such as and.
24. d. An effective conclusion recalls assertions made in the introduction without repeating them verbatim. Only choice d reiterates the introduction’s ideas that Poe is a master of language and uses it to evoke emotion from his readers.
25. e. Sentence 4 logically follows sentence 3— without the mention of the specific location of

54

C H A P T E R

3

The Essay
The SAT essay is in many respects unlike any other kind of writing you’ve had to do. While the fundamentals of good writing remain a constant, the approach to this task is radically different. You don’t have time to brainstorm, outline, revise, and edit the way you would if the clock wasn’t ticking. When you have just 25 minutes, each one counts. The scorers know this, so they’re trained to look for “polished rough drafts.”
Does that mean you can forget about spelling, verb tenses, and idioms?
Not exactly. How can you produce a high-scoring essay, while under pressure? The key is preparation.

T

his chapter is designed to help you do just that. You’ll learn how to tackle the two types of prompts, how to budget your time, and how to organize your writing. You’ll get to practice by quickly coming up with thesis statements and introductory “hooks” for a number of prompts. Also included are three essays for you to score and study, one that meets the requirements for the highest score, a six, and two that earn lower scores. Preparation, through study and practice, will help you develop a strategy for approaching the essay.
That strategy will make you more confident when you face the SAT essay, help you use each minute to full advantage, and result in a higher score.

55

The Issue of Space
Some recent test takers report that they were shocked by the small amount of space they were given on their answer sheet to write the essay. Although you can use the test booklet to take notes and organize your thoughts, only what is written on the answer sheet will count. The practice tests in
Chapters 4–6 include the same amount of space you’ll encounter when taking the SAT, so you will be able to walk into the test confident that there will be no surprises.

Strategies for Timed Essays



As we’ve said, you are not expected to turn in a finaldraft essay, but instead you’re creating a “polished rough draft.” A couple of minor errors in grammar, usage, and mechanics will not be weighed against you.
Scorers read the essay to get an overall impression of your writing ability. They look for evidence of critical thinking: How well did you respond to the topic, develop a point of view, and use appropriate examples and evidence to support your position? Is your essay clearly focused, and does it transition smoothly from one point to the next? Do you show evidence of having a varied and intelligent vocabulary? Since readers spend approximately three minutes on each essay, it’s important to hit each of the marks they’re looking for.







Scoring

Two graders, who are high school or college English or writing teachers, will read your essay. They are trained to spend two to three minutes on each essay, and then give it a score of 1–6 (although an essay that is written off-topic gets a zero). Your total score will be in the range of 2–12. If the graders disagree by more than a point, a third reader will be given your essay.
Although readers will give your essay one score based on a holistic response, they look at four areas to determine that score: meaning (content), development
(support), organization (flow of ideas), and language use or mechanics (grammar and punctuation). Scores range from a low of 1 (showing writing incompetence)

Official Directions

As with the multiple-choice questions, it’s important that you study the directions for the essay as part of your preparations. You don’t want to waste time reading them during the test. Official directions are at www.collegeboard.com. Either print them out, or copy them, and then study them. Here are some of the key points stressed in the directions:


Develop your ideas logically (use transition words, and don’t jump randomly from one thought to another).
Support those ideas with appropriate examples and evidence (think concrete and precise—don’t make assertions you can’t back up).
Use clear, concise language, and proofread for spelling, punctuation, and grammar mistakes
(“big words” used to show off and long complicated sentences won’t get you points).
Write only in the space provided (wide margins, skipping lines, large indents, and handwriting that’s too big will waste space).

Choose a point of view to argue, and stick to it
(don’t mention both sides of an argument while flip-flopping back and forth).

56

– THE ESSAY –

Overall, are you observing the standard rules of grammar, punctuation, and spelling? Is your vocabulary sufficient to adequately put forth your ideas? Are your sentences varied?
In the box below are criteria that are more specific for each essay score.

to a maximum of 6 (demonstrating clear and consistent competence). The graders will focus on the strength of your argument. Are you convincing? Do your ideas make sense? Do you have insightful, supported comments on the topic? Do your ideas flow logically? Your writing style will also be judged (but keep in mind that a few stray commas and spelling mistakes will be forgiven).

Scoring
Score of 6

Score of 3

A 6 essay stands out because of its thinking; it has

This essay is inconsistent. While some elements

a strong point of view, which is developed in an

may be adequate, such as demonstration of critical

organized, focused manner. Examples and evi-

thinking skills or use of appropriate examples, oth-

dence are used to support the point of view. There

ers are weak. It may lose focus and provide evi-

may be a few errors in grammar, usage, and/or

dence for some points but not others. There will be

mechanics, but word choice (vocabulary) and struc-

little or no variety in sentence structure, and a num-

ture (varied sentences) show mastery of language

ber of errors in spelling, grammar, and punctuation.

and writing skills.
Score of 2
Score of 5

A 2 essay is inadequate. Its point of view will be

A 5 essay is also effective in the position it takes

weak, poorly developed, and/or insufficiently sup-

and how it develops that position, but it has more

ported. There is a lack of organization and evidence

errors than a 6 essay. It uses examples and evi-

of poor language skills. Errors in mechanics, word

dence skillfully, and presents its argument in an

choice, grammar, and spelling will be serious and

organized way. A strong vocabulary is evident.

plentiful.

Score of 4

Score of 1

An essay in this category is adequate, developing

This essay will be incoherent, with no clear position

a position reasonably well, and supporting it with

taken or supported. It will be disorganized, with

examples and evidence. Its organization and focus

serious errors in vocabulary, sentence structure,

will be weaker than that of a 5 essay. There will be

grammar, and mechanics. These errors will prevent

errors in language, grammar, and mechanics.

the reader from understanding the essay.
Score of 0
No matter how well written, essays that do not address the assignment are scored zero.

57

– THE ESSAY –

Another prompt of this type asks you to choose between two opposing ideas expressed in quotations, as seen below:

Understanding the Prompts
The assignment of the essay is to respond thoughtfully and skillfully to a prompt, which will be one of two types: ■



Better a lie that soothes than a truth that hurts.
—Czechoslovakian proverb

Response to a quote. You are given one or two quotes and asked to evaluate or compare them in response to a question.
Completion of a statement or idea. You are given an incomplete statement and asked to fill in the blank, then use the completed statement as the basis for your essay.

Truth is the only safe ground to stand on.
—Elizabeth Cady Stanton
Assignment: Consider the two contrasting statements above. Choose the quotation that most closely reflects your viewpoint. Write an essay explaining your choice.
To support your view, use an example or examples from history, politics, science and technology, literature, the arts, current events, or your own personal experience and observation.

Responding to Quotes

Here’s an example of this type of prompt:
Good fences make good neighbors.

Again, the first step is to be certain of what each author is trying to say. Once that’s determined, choose the side for which you can immediately think of some good examples or evidence. You don’t truly need to agree with the quotation to write a strong essay, and you won’t have time to change your mind. Choose a side, and get going. Remember that a well-developed point of view is what the readers look for; they have no idea whether you really believe that view.

—Robert Frost
Assignment: In Frost’s poem “Mending Wall,” a man tells his neighbor “good fences make good neighbors,” suggesting that clear boundaries are needed for people to get along with one another. Do you agree or disagree with this idea? Develop your point of view, supporting it with an example (or examples) from literature, the arts, history, current events, politics, science and technology, or personal experience or observation.

Completing a Statement or Idea

The other type of question you may confront will ask you to complete a sentence or idea, such as the one that follows: When faced with a quotation, determine what is being said, and rewrite it succinctly, in your own words, before you begin your essay. For example, in response to the above quotation, you may write “when people have their own space, and feel that space is protected, they get along better with others.” Then, look at the assignment for more details about how to proceed.
Here, the task is to develop a point of view that agrees or disagrees with the quote’s assertion.

People often say it’s better to be safe than sorry. I felt the truth of this statement when . . .
Assignment: Complete the sentence above with an appropriate phrase. Then, write an essay supporting your completed statement.

58

– THE ESSAY –

This question type requires that you think of examples and situations that illustrate the statement.
Readers will be looking for a clear, strongly supported account of an event that led you to appreciate the need to be careful. This question easily lends itself to use personal experience. Or, you could answer using a historical event with which you are very familiar. No matter how you address the question, the point is not just to make claims or assertions, but also to back them up with evidence and examples.





The Art of Persuasion
Both types of essay prompts call for a persuasive essay, one in which you choose an idea and show why it is legitimate or worthy. Your purpose is not to merely explain your point of view, but to convince your reader why it makes sense. In order to persuade effectively, you must base your argument on reasoning and logic.
If you are unsure or undecided in your stance, your writing will be weak and your score will suffer.
However, your opinion is not enough. Like a lawyer before a jury, you must convince your reader with evidence that your opinion is valid. This evidence consists of concrete examples, illustrations, and details. Therefore, the most important strategy for the persuasive essay is to choose the side that has the best, or most, evidence. If you believe in that side, your argument will most likely be even stronger (although, as mentioned above, you don’t have to believe in it to write a good essay).
The essay does not require specific knowledge of literature, history, or current events. However, the topics are broad enough that you will probably be able to use your knowledge from these areas to answer the question. While you can always rely on personal experience, as many high-scoring essays do, it’s a good idea to review areas you’ve studied or are otherwise familiar with to use on test day (don’t try to learn new material for the essay). Here are some ideas for what to review:



Literature: poems, novels, plays, and myths with broad themes that can be applied to a number of topics. Orwell’s Animal Farm, for example, could be used to discuss equality (“some animals are more equal than others”), the class system, or forms of government.
History: events and time periods such as World
War II, the Great Depression, treatment of Native
Americans, and America’s break from British rule. Historical events may be written about from many perspectives, and can be used to make points about a variety of subjects. The Depression, for example, was an economic event that had many factors, including human emotion
(fear).
Science: technology, space exploration, the concept of absolute zero, acid rain, and other environmental issues. For example, you could discuss global warming in terms of man’s disregard for the planet, or in terms of a positive worldwide response that is bringing together many nations.

Whatever the subjects you’ve studied, think in terms of flexibility. How many different ways can you look at an event, an invention, or a work of non-fiction?
What does it mean to people, how do they respond to it, or how has it changed the world (for better or worse)?
Having a few adaptable subjects fresh in your mind may help you respond quickly, specifically, and thoroughly to what will most likely be a very general prompt.

Anatomy of an Essay
Unfortunately, 25 minutes is not enough time to come up with an innovative structure for your essay. You need to address the topic in a clear, well-organized fashion, using examples and details to make your point.
The best way to accomplish those goals is to stick to a traditional format, the five-paragraph essay. Aim for an

59

– THE ESSAY –

are an essential part of a civil society.” The problem with the first sentence is the first three words. “In today’s society” is a clichéd opening, whereas the second sentence makes its point directly, without any overused language.
Once you’ve narrowed down your topic, and have a clear, confident thesis statement, think about how to grab your reader’s attention. Imagine you’re an essay grader reading hundreds or thousands of SAT essays. It’s late and you’re tired. Which of the following first sentences would make you sit up and take notice of the essay?

introduction, three body paragraphs, and a concluding paragraph. By writing within this format, your ideas will be easily available to your reader (the person scoring your essay), and you will have more time to develop and substantiate them.
Introduction: Thesis Statement and Hook

The introduction presents the reader with your topic and point of view. It is more general than the body paragraphs of the essay, which contain the specific examples and evidence that help you substantiate and develop your topic. The goal of the introduction is to make the reader clearly understand your position, without being trite or boring. To accomplish it, you’ll need to write two things: a thesis statement and a hook. A clear thesis statement is one sentence that refers directly to the topic. It gets right to the point, because the real meat of your essay, where you can deliver the greatest impact to the reader, is in the body.
Stating your thesis quickly and clearly means avoiding disclaimers such as “I’m not sure, but . . .” and “This may not be right . . .” Such disclaimers are a waste of time, and will lose points with your reader. No matter how strong your argument becomes in later paragraphs, that initial poor impression will stick. Instead, be confident and direct. A clear thesis statement shows the graders that you understand the assignment and have formulated a relevant response to it. It also sets the stage for a well-developed essay in which specific and interesting examples support it.
But direct doesn’t mean trite. Relying on overused words and phrases to help make your point is the most common way to weaken your introduction.
Compare: “In today’s society, people don’t practice good manners often enough,” with “Good manners

Imagine a world in which plant life is reduced to a few hardy specimens, drought is commonplace, and the world’s coastal regions are under water. OR
The future effects of global warming will be bad. The first sentence is a hook; it is designed to inspire the reader to want to read the rest of the essay.
How can you come up with something so seemingly clever and innovative in a minute or two? It’s not as hard as it looks; what at first glance appears clever and innovative is really the product of a learned method.
Two types of hooks are quick and easy to create. If you study them, and practice writing them from sample prompts, you’ll be able to write a hook for your essay.
One type of hook is used in the first example above: a dramatic scenario, saying, or statistic. A scenario paints a vivid picture with words. A related statistic, proverb, or other saying can boldly introduce your topic, show off your knowledge, and give your writing some heft. Statistics in particular add a tone of

60

– THE ESSAY –

students you attend class with, or the nation with which yours shares a border.

seriousness and importance to your writing; they say
“I’m not the only one who thinks this way—there are studies to back me up.” Compare:

In this introductory paragraph, the three examples that will be explored in the essay are mentioned.
This is a great way to transition the reader from the introduction to the body of the essay. However, it’s not always possible to have those three examples at the ready. What if you have two, but are betting the third will come to you while writing the body? It’s still better to give the reader an idea of the direction you’re headed before jumping into the body of the essay. This technique shows off your thinking skills and your ability to organize your ideas.

Not only is the number of overall incidents of cancer in Americans decreasing, but survival rates are dramatically increasing.
To:
More people survive cancer these days.
The other type of quick hook to study and practice is questioning. Pose a specific, relevant question to your reader that will naturally lead into your topic. In the introductory paragraph below, the hook is in bold.
Example
Is the difference between a good neighbor and a bad neighbor simply that one doesn’t hit baseballs through your windows, and the other does?
It’s not that simple. There are many qualities of a good neighbor, and one of the most important is distance. Having your own space, and having your neighbor respect that space, is the key to a good next-door relationship. This is true whether discussing the person whose lawn abuts yours, the

Common “Hook” Mistakes
1. Using a title as a hook
Titles are typically incomplete sentences, such as The March of Progress or My Soccer Team’s Lesson. A hook is a sentence or couple of complete sentences that draw the reader in. The use of phrases or clauses instead of complete sentences in your essay will lose points.
2. Using an announcement as a hook
Don’t address your reader directly with an announcement or literary road map of where you’re going. “This essay will be about man’s triumph over natural disasters” is not sophisticated enough for the SAT essay.
3. Confusing a thesis statement for a hook
Your thesis statement gets right to the point. As an opening sentence, it’s probably dull. The addition of a hook will ensure a better, more interesting introduction.
61

– THE ESSAY –

It’s been said already that a few small grammar or mechanics errors will be overlooked. However, you probably noticed when reading the scoring rubric that sentence variety is important to your reader. Don’t use too many short, choppy sentences; vary your sentence structure so that your reading is interesting and flows easily. Body

In the body of your essay, you develop and illustrate your ideas on your topic. It is where you add the interesting details and examples that support your thesis and make your essay stand out. The body should be three paragraphs, one for each example or idea.

The Best Way to Achieve a High Score
Readers of your essay are specifically looking for a key element that can make or break your score: is every point you make supported with details, examples, and evidence? Not only will these elements strengthen your argument, but they’ll also make your writing come alive. One way to assure that you’ll include enough supportive information is to write at least one sentence in each paragraph that begins with the words, “For example.”
Compare these paragraphs:
High school seniors should be allowed open campuses, on which they can arrive in time for their first class, leave during free periods, and come back to school for their other classes. There is no reason to treat high school seniors like children by making them stay in school all day when they don’t have classes to attend all day. Seniors can handle the extra responsibility.
High school seniors should be allowed open campuses, on which they can arrive in time for their first class, leave during free periods, and come back to school for their other classes. Seniors are given freedom and responsibility in many other areas of their lives; for example, the ability to drive a car. Seniors are also permitted to vote, and to prepare for their futures through the college admissions process or vocational training.
The first example uses generalizations and unsubstantiated claims (“no reason to treat them . . .” “can handle the extra responsibility”), which add nothing to the argument. The second uses evidence, such as the responsibility of driving and voting, to make the case for open campuses. Writers of high-scoring essays back up what they say with evidence, details, and other types of examples.

62

A Word of Warning about Vocabulary
Although a sophisticated vocabulary will score you points, the use of very obscure or uncommon words won’t. Don’t memorize a list of “big words” and make them fit into your essay. Your vocabulary should flow freely. Substitute common words with more interesting ones, but don’t make your writing sound artificial or self-conscious by overusing difficult or obscure words.

In addition, follow this crucial advice when writing the body:
■ Include only information that pertains to your topic (do not go off on tangents).
■ Illustrate or explain each point with appropriate details. Some essays may call for personal experiences, while others may require historical examples. Don’t simply state that something is true:
Prove it.
■ Organize the body with three paragraphs.
■ Maintain coherence by staying on topic; every sentence should relate to your topic.
■ Use transition words like first, next, and then.
■ Get creative if necessary. Your reader will never know if you really traveled to Bombay, won a hotdog eating contest, or attended science camp. The quality of your writing is what is being tested, not the truthfulness of every detail. If you need to get creative and come up with a strong example or piece of evidence, and you can do so convincingly, go ahead.
■ Take all the time you can to fully develop your ideas. If you stop writing too soon, it may be because you haven’t explained yourself completely, or backed up your assertions with examples. In addition, word choice is important. A sophisticated vocabulary will make your essay stand out from those with a more basic vocabulary. While nothing can take the place of years of reading challenging material and exposing yourself to a variety of texts, study can help. Keep up your reading in the months before the
SAT. When you encounter new words, notice their context and look up their definitions in the dictionary.
Search the Internet with the terms SAT and vocabulary.
You’ll find dozens of websites with lists of words that frequently appear on the SAT. Study those you don’t know, aiming to learn at least five new words a day.
The more words you know, the easier it will be to choose specific, interesting ones rather than general, dull words. Bad, gets the point across, but detrimental, harmful, and injurious could be better choices. As you write your essay, be conscious of the words you select.
Avoid repeating the same words; use a synonym after you use a word twice.

63

Transition Words
These are useful when moving from paragraph to paragraph, or point to point. Transition words help the reader follow your thoughts. after in addition

afterward, after this

moreover

as a result

next though

another

nevertheless

because

on the contrary

consequently

on the other hand

conversely

similarly

despite

simultaneously

finally

subsequently

first, second, third

then

for this reason

therefore

however

yet

it follows that

If you wrote about a problem, try a conclusion that offers a solution. If you have a fitting quotation, use it to conclude your essay. The person quoted doesn’t have to be famous, but the quote should help you make your point. For example, “My third grade teacher put it best . . .” These types of conclusions can leave your reader with a better overall impression of your work
(although be aware that you can’t overcome a weak essay with a clever conclusion).

Conclusion

Your concluding paragraph can simply restate your thesis and the points you made in the body of your essay, but remember to reword them to keep the conclusion fresh. Don’t repeat your introduction, or use phrases such as “I wrote about,” or “This essay was about.”
If you have time, end with something more interesting. A speculative conclusion refers to a future possibility or prediction, such as “perhaps years from now . . .”

Great Question
“What if I come up with a great new idea when writing my conclusion?”
Answer:
In order to use the idea, you must be able to revise your thesis statement to include it, or at least hint at it. You don’t want to turn in an essay that shows you didn’t come up with anything interesting until the final paragraph. A revision of your introduction shows that you can organize and integrate information effectively.
64

– THE ESSAY –

such as Acing the SAT 2006 (LearningExpress, 2006) or
11 Practice Tests for the New SAT (Princeton Review,
2004).

Conclusion Checklist

Do not contradict anything you said earlier in the essay.

Be clear and concise.

Do not introduce new information.

Maintain the tone you used in the rest of your essay.

Do not repeat your introduction.

Do not use clichéd sayings or phrases (“You can’t judge a book by its cover,” “In conclusion,” “As I stated above”).

Do not apologize for anything (especially lack of time).

Planning Your Essay
Planning takes about five minutes. In that time, you need to accomplish three things. It may sound like a lot, but don’t panic: With practice, you’ll be able to complete this task easily and on time. Begin with an initial interpretation of the prompt (putting it in your own words), and choosing of a point of view or side to argue. Once you’ve made a choice, stick to it. There’s no time to scrap your plans and start again. Here’s an example: Budgeting Your Time

You must accomplish three distinct writing tasks in 25 minutes: planning, writing, and revising. The writing stage will be the longest, and the revising stage will be the shortest. What’s the mistake most test takers make?
Jumping into the writing stage with inadequate, or no prewriting. It is essential that you take some time before you begin writing your essay to think about your prompt, brainstorm ideas, and plan a rough organizational strategy. Here’s an idea of how long to spend on each step of the writing process:

Better a lie that soothes than a truth that hurts.
—Czechoslovakian proverb
Truth is the only safe ground to stand on.
—Elizabeth Cady Stanton
Assignment: Consider the two contrasting statements above. Choose the quotation that most closely reflects your viewpoint. Write an essay explaining your choice.
To support your view, use an example or examples from history, politics, science and technology, literature, the arts, current events, or your own personal experience and observation.

Planning = 4–6 minutes
Drafting = 14–16 minutes
Proofreading (Editing) = 3–5 minutes
Total: 25 minutes
The actual time you spend on each step may vary, but only slightly. Finding out how to budget your time while writing the essay is one of the most important things you’ll learn when practicing. In Chapters 4, 5, and 6, you’ll be given an essay prompt and space to write an essay. Don’t begin without setting a timer. Get used to planning the essay in four minutes, and know what it feels like to try editing in just two. If you still haven’t gotten the hang of timing your writing after the third practice essay, get more prompts from resources

Telling the truth can sometimes be painful, not so much for the teller, but for the listener.
This student has taken a side by choosing the
Czechoslovakian proverb, paraphrased the quote, and begun to think her idea through (“truth can be more painful for listener than teller”).
If you’re given a question in the prompt, a quick way to interpret it and at the same time formulate a thesis statement is to recast the question as a pronouncement.

65

– THE ESSAY –

“Is losing a key to success?” becomes “Losing can be an incredible learning experience—one that teaches the tools needed to become a success.” “Do you agree with
Einstein that genius is 1% inspiration and 99% perspiration?” becomes “Although hard work is often essential, I disagree with Einstein that it’s 99% of what it takes to be a genius; plenty of people become successful without working hard.”
The second stage of planning is brainstorming, or gathering ideas. There are dozens of effective brainstorming strategies, including listing, clustering, webbing, and freewriting. In this section, we’ll review the two that adapt best to the time constraints of the SAT essay. After your review, select the one you feel most comfortable with and use that strategy every time you practice (and, of course, during the test). Remember that knowing exactly what you will do when you begin the exam will not only save time, but will take some of the pressure off, too.

Example
Although hard work is often essential, I disagree with Einstein that it’s 99% of what it takes to be a genius; plenty of people become successful without working hard.

Listing

Clustering

Perhaps the easiest form of brainstorming is listing. Jot down ideas in response to the prompt on the scratch paper in the test booklet. Don’t worry about creating complete sentences—keep your ideas short, limiting them to words or phrases. You may even want to abbreviate certain words to save time. After you’ve listed about a dozen ideas, link those that go together by drawing lines between them, and eliminate those that either veer off the topic or are redundant. Choose the three ideas that will be easiest to develop (ones for which you’ve already come up with examples of and evidence for).

Clustering involves jotting down ideas as they come to you and organizing them visually at the same time.
Start a cluster by writing your topic in the center of the scrap paper. Then, write ideas around this topic as they come to you. Quickly put these ideas in circles and attach the circles to the topic by drawing lines. Then, look at your ideas, and expand on them. When you’re finished, you’ll probably find that there are some ideas that generated much material, and others that were dead ends. Choose the three ideas with the most circles around them for your essay.

Examples of people who succeed without much effort— Inherit family business or wealth (personal example—Uncle Lee)
Get lucky by having the right idea at the right time (Amazon.com—Jeff Bezos, among wealthiest never showed profit)
Get into college because family members are alumni (Kennedys, Bushes)
Athletes need natural ability—not just about working hard—two swimmers with same training regimen—one goes to Olympics, other doesn’t make cut

66

– THE ESSAY –

use words and actions to show who you really are; push yourself past what you think youíre capable of

5 minutes of writing each day; at least
30 minutes of reading each day personal philosophy

discipline

person who most influenced me: English teacher reading choices not afraid to assign tough material; learned life lessons from assigned reading

Organizing

1. Some people come into our lives and teach us some of life’s most important lessons, while others are there to guide us through the day-today decisions and trivialities. My grandmother did both, influencing my life in many ways, both big and small.
2. Matriarch of family
■ gave advice on dating, money, problems, etc.
■ came to family’s aid (cousin Joe’s jail time)
3. Hard worker—inspired to reach our goals
■ worked full time—made money for kid’s college
■ amazing single parent—kept household, kids’ lives together
4. Independent—own voice
■ stood up for her beliefs
■ friends—all races
■ didn’t allow ethnic jokes/put-downs in her house 5. Conclusion

The third and final stage of planning is organizing.
Because there is a standard formula for high-scoring
SAT essays, organizing is much easier than it is for other types of writing. You simply take your brainstorming notes and thesis statement and arrange them into five paragraphs. Although you may be tempted to skip this stage, resist the temptation. Your rough outline will be your roadmap that keeps you from wandering off-topic while you’re writing.
Write your outline as a five-point list:
1. Introduction, including thesis statement and hook if you’ve written it already
2. Example one, with details, evidence
3. Example two, with details, evidence
4. Example three, with details, evidence
5. Conclusion that restates thesis
Here is an example of how a student constructed a simple outline based on her brainstorming.
Question: An influential person is one who leaves a footprint in the sand of our soul. To me, the most influential person I can think of is . . .

67

– THE ESSAY –

you addressed the topic. Don’t risk alienating or offending him or her by using words and a tone that are too formal or too casual. Avoid controversy; religion, politics, and race relations are all examples of topics that have the potential to offend. Because you don’t know the personality of your scorer, steer clear of anything that might upset him or her. You are trying to convince your reader that you can write well, and that what you are saying is reasonable and intelligent. If you alienate, confuse, or offend, your essay score will probably suffer.
In addition, your reader can’t give you a score if he or she can’t figure out what you’ve written. Unless your cursive is very easy to read, print your essay. The importance of legibility can’t be overstated.

Drafting Your Essay
Using your outline as a guide, write your essay, using paragraphs to separate your major points. For each paragraph, write a topic sentence that clearly and succinctly explains the point you are making. Do not go off on tangents, but adhere to your plan. If you come up with another strong major point, use it, but don’t freewrite or ramble. Avoid unnecessary words and phrases, including clichés. The literary equivalent of
“blah blah blah” may fill up lines, but it won’t score you points (see the box below on essay length).
Keep your reader in mind. This person will give you a score based on how well you write, and how well

A Word about Length
You may have noticed when reading the scoring rubric that length was not mentioned, either as an attribute of a high-scoring essay, or a detriment to a low-scoring one. However, a recent study of essays used to train scorers indicated that length is indeed considered. The College Board weighed in on the controversy, noting that longer essays are typically better developed, and better meet the other scoring criteria.
What can you take away from the dispute? If you follow the advice in this chapter, writing five paragraphs that include thesis or topic sentences, examples and evidence, transitions, and a solid conclusion, you should fill, or be close to filling, your answer sheet. In other words, don’t let the controversy change the way you approach the essay. Aiming for a specific length takes time—something that’s in short supply during the test. Instead, follow the plan, and your essay will not only be “long enough,” it will be well developed, organized, and otherwise well written.

68

Grammar Pitfalls
The three most common grammatical errors students make on the SAT essay involve confusing words
(they’re, there, their), agreement (singular nouns with singular verbs, plural nouns with plural verbs), runons, and sentence fragments. These issues are explained in Chapter 2. It’s worth taking another look at them to make sure you understand each one. In particular, study the list of confused and misused words.
They’re the kind of error that can really detract from the quality of your essay, and call your language skills into question.

Essay Writing Workshop

Proofreading

You should have about five minutes left to reread your essay. Check for the following do’s and don’ts:

















In this section, we’ll put it all together. Three essays of varying quality will be presented, and you’ll score them using the SAT essay rubric. Then, we’ll examine in detail what worked, what didn’t, and what score each essay would receive. In addition, you’ll have the opportunity to practice writing quick thesis statements and hooks to a number of sample prompts. These exercises are designed to help you put into practice what you learned in this chapter, and prepare you for writing your own essays in the next three chapters.

Do begin with an interesting hook and strong thesis statement.
Do provide details, examples, and supporting evidence in each paragraph.
Do use paragraph breaks to help the reader see your main points (one point per paragraph).
Do transition smoothly from one idea to the next.
Do check your vocabulary (substitute more descriptive words and synonyms where possible).
Do end strongly with a conclusion that restates your thesis.
Don’t forget to look for and correct mistakes in grammar, spelling, and punctuation (see the box below). Don’t miss the opportunity to complete thoughts and phrases that could leave your reader guessing.
Don’t leave in rambling thoughts that are off the topic. Essay Scoring

1. Prompt:
An environmental problem facing our world today is
.
Assignment: Finish the sentence, and use your completed statement as the basis for an essay.
Global warming which means that it is getting warmer all over the globe, is a serious environmental problem. It is bad for the environment, nature, animals, and humans as well. Global warming causes a lot of glaciers to melt which then causes more floods and makes the ocean warmer which could hurt certain kinds of fish. Global warming also leads to more fires in general and increases the rate of cancer in humans, especially skin cancer.

69

– THE ESSAY –

failure is one of the most important factors involved in achieving success. He notes that, “each failure leads us closer to deeper knowledge, to greater creativity in understanding old data, to new lines of inquiry.” But what if, as in the example above, countless failures end with a poor result? It is not failures themselves that lead to success, but rather a combination of natural ability, persistence, and even luck. When I started competitive swimming, at age seven, I had some natural ability. Swimming came easily to me. When shown the correct techniques for strokes, turns, and starts, I was able to employ them much quicker than many of my teammates. In fact, within a few months, I was swimming faster than some kids who were on the team for a few years. They had “failed” many times in the meets they swam in, but it didn’t seem to help them understand the techniques (“old data”) or to come up with better strategies (“new lines of inquiry”). I
“failed” just a few times that first year, but my times were better. My natural ability helped me to achieve more in comparison with their numerous failures. Persistence has also been a factor in my success. For the past eight years, I have attended practice at least three days a week, with a short break between each of two seasons. I swim at least
300 days a year. This persistence has allowed me to improve both technique and speed. In comparison, those who don’t continue to practice frequently and find ways to swim better and faster don’t make the times I do. On my old team, we practiced for an hour and a half, three days a week, forty weeks a year. On my new team, practice is five days a week for two hours, and we have just four weeks off a year.
This new practice schedule has helped me to take seconds off every time, and my new team as a whole performs better than the old one.
Luck is also a factor in success. I once won a regional meet because my competitor, who was one

In order to stop global warming, we should study the greenhouse effect. Because we use too much oil and gas and pollute the air on a regular basis, hot air can’t escape the atmosphere. We need to use less oil and gas so the hot air can get out. People don’t need to drive trucks and SUVs all the time because they use more gas and cause more air pollution. We also don’t have to use air conditioning all the time. People need to remember that minivans and air conditioning are luxuries not neccessities.
If everyone agreed to change their habits, it would help the environment a lot. So, we should find out what needs to be done to solve this serious environmental problem and do whatever it takes.
Score:
2. Prompt:
The principle is this: each failure leads us closer to deeper knowledge, to greater creativity in understanding old data, to new lines of inquiry.
Thomas Edison experienced 10,000 failures before he succeeded in perfecting the light bulb.
When a friend of his remarked that 10,000 failures was a lot, Edison replied, “I didn’t fail 10,000 times, I successfully eliminated 10,000 materials and combinations that didn’t work.”
—Myles Brand,
“Taking the Measure of Your Success”
Assignment: What is your view on the idea that it takes failure to achieve success? Plan and write an essay in which you develop your point of view on this issue.
Support your position with reasoning and examples taken from your reading, studies, experience, or observations. Imagine attempting a task dozens of times, failing each time to accomplish it. No matter how hard you try, you cannot improve. Myles Brand wrongly suggests, in Taking the Measure of Your Success, that

70

– THE ESSAY –

feel like their hired help or something. Contrarily, too little money can make him or her feel like their help isn’t worth anything to his or her parents. So finding the right amount is important.
In conclusion, giving children an allowance for doing household chores is a good idea. Children learn to work for their money and save what they earn.

hundredth of a second ahead, made an error and was disqualified. Another time, a competitor was sick on the day of the meet and didn’t swim his best time (which would have beaten me). My successes at those meets involved, at least in part, luck.
So failure is just one part of success. The more important factors are natural ability, persistence, and luck.

Score:
Score:
3. Prompt:
Parents should give their children a weekly allowance because

Practice Thesis Statements

Recall that a thesis statement directly refers to and takes a stand on the topic. It should be no longer than one sentence. A confident tone and clear, concise language are also hallmarks of great thesis statements.

.

Assignment: Finish the sentence, and use your completed statement as the basis for an essay

General Directions

Parents should give their children a weekly allowance because it gets them to work around the house. My parents paid me, and my brothers and sister. I never liked doing chores, but getting an allowance each week (if I did my chores) made it not so bad. In fact, sometimes I did extra (like reorganizing the pantry) to get some extra money for something I really wanted. I think having my allowance depend on my doing chores made me understand what it’s like to work.
In the “real world,” you don’t get paid if you don’t do your work. That’s how it was in our house.
I also learned that it’s hard work to keep a house going, I learned to appreciate all the hard work my mom and dad use to do. In addition, I learned how to save money. I would set aside my allowance to save up for something I wanted, like a new CD player or outfit.
In my opinion, parents should give an allowance for doing chores, but it shouldn’t be too much. Children should know that they need to help no matter what. Too much money I think would make him or her

Write a thesis statement for each of these three quotes.
Possible answers are found at the end of the chapter, on page 74.
1. Character cannot be developed in ease and quiet. Only through experience of trial and suffering can the soul be strengthened, ambition inspired, and success achieved.
—Helen Keller
Assignment: Is the character developed, and ambition and success achieved, only through adversity? Take a position on this topic, and write an essay that explains and supports your point of view.

2. Education is a method whereby one acquires a higher grade of prejudices.
—Lawrence J. Peter

71

– THE ESSAY –

Assignment: Is there value to an argument, or are differences better managed in another way?

Assignment: Education is predominantly spoken of in terms of broadening students’ perspectives, opening them up to a wider world beyond their own. Is this always the case? Do you agree with Peter that education makes one more discriminating, in effect narrowing one’s scope and lowering one’s tolerance?

2. During periods of relaxation after concentrated intellectual activity, the intuitive mind seems to take over and can produce the sudden clarifying insights which give so much joy and delight.
—Fritjof Capra

3. History is the version of past events that people have decided to agree upon.
—Napoleon Bonaparte

Assignment: Our society rewards hard work. Hours on the job have increased as technology makes it easier to work from home, while commuting, and even while on vacation. Is all this work good for us? What are the benefits of down time that we could be missing?

History illumines reality, vitalizes memory, provides guidance in daily life and brings us tidings of antiquity.
—Cicero
Assignment: Consider the two contrasting statements above. Choose the quotation that most closely reflects your viewpoint. Write an essay that explains your choice. To support your view, use examples that may include your own personal experience and observation.

3. Many people blame their failures on not
“being in the right place at the right time.” That idea assumes that those who succeed do so passively, because of luck. Ellen Metcalf says, however, that it is an active process: “You have to recognize when the right place and the right time fuse and take advantage of that opportunity. There are plenty of opportunities out there. You can’t sit back and wait.”

Practice Hooks

A hook is an opening line or lines that provides the interest needed to draw the reader into the essay. For the following three assignments, write either a questioning hook, or one that presents a dramatic scenario, statistic, or quote. Possible answers are found at the end of the chapter, on page 74.

Assignment: Do you agree with Metcalf that being in the right place at the right time is not enough? Is success simply opportunity, or is it what we do with opportunity? 1. Contentious discourse is a part of our everyday lives. People argue about topics such as politics, religion, moral values, often causing more harm than good. As Supreme Court
Justice Louis Brandeis said, “Behind every argument is someone’s ignorance.”

72

– THE ESSAY –

Commit to Memor y

Answer Key

For your review, here are the most important points made in Chapter 3:






















Essay Scoring

1. In this essay, the writer says global warming causes many problems, has many causes, and can be stopped. Supportive examples for these ideas are located in both the introductory paragraph and one body paragraph. Sentence structure is varied, evidence and examples are plentiful, and the grammar mistakes don’t detract significantly from the points the author makes. As is, the essay would probably receive a score of 4. How could it become a six? The biggest problem with the essay is organization.
In order to improve her score, this writer would have to spend some time reorganizing her major points and examples. Instead of three paragraphs, there should be five. She has enough information to put one point into each body paragraph. Reiterating the thesis statement and major points, rather than introducing new information, would strengthen the conclusion.
2. This essay succeeds on a number of levels. First, it takes a firm stand on the prompt and adheres to it throughout. Evidence and pertinent examples support the position. Second, it is well organized. The five-paragraph structure allows the writer to explore his three chosen subjects, while maintaining a clear focus. This allows the reader to follow him easily. Third, there are very few grammar, usage, or mechanics errors. And fourth, it is well styled. It begins with a hook, and has a strong, direct thesis statement. The vocabulary and sentence structure are varied, and the conclusion restates the thesis. This is a 6 essay.
3. This essay satisfies the requirements of the writing prompt in an abbreviated manner, giving only brief examples and developing ideas inconsistently. It has a general focus, there is an obvious attempt at organization, and ideas are

Choose a point of view to argue, and stick to it
(don’t mention both sides of an argument while flip-flopping back and forth).
Develop your ideas logically (use transition words, and don’t jump randomly from one thought to another).
Support your ideas with appropriate examples and evidence (think concrete and precise—don’t make assertions you can’t back up).
Use clear, concise language, and proofread for spelling, punctuation, and grammar mistakes
(“big words” used to show off, and long complicated sentences won’t get you points).
Write only in the space provided (wide margins, skipping lines, large indents, and handwriting that’s too big will waste space).
Review areas you’ve studied to get ideas for examples and evidence.
Practice writing thesis statements and hooks that will make your first paragraph strong and memorable. Choose three main points to develop in the three body paragraphs; use topic sentences to introduce the points and provide focus for the paragraphs.
Honesty doesn’t count; your scorers will never know if your uncle is a self-made millionaire, you rode the biggest roller coaster in the world 40 times, or your best friend is a stunt double.
Use your conclusion to reassert the point of your thesis statement without citing it verbatim.
Budget your time as follows: planning = 4–6 minutes, drafting = 14–16 minutes, and proofreading
(editing) = 3–5 minutes.

73

– THE ESSAY –

presented in a logical progression. However, there is an uneven control of mechanics, and sentence structure is not varied. Word choice is not formal enough (made it not so bad, hired help or something, contrarily) and negatively affects the quality of the essay. This is a 3 essay. To improve it, the author would have to elevate the tone, correct errors in mechanics, and work to develop his major points in greater depth.

Practice Hooks

The following are provided as examples; your hooks will differ. As you evaluate your responses, keep in mind that effective hooks are questions, dramatic scenarios, statistics, or quotes designed to entice the reader. They are complete sentences (not titles), and never address the reader directly (I’m going to tell you about . . .). Hooks are also not as direct as thesis statements—they don’t always make clear the direction your essay will take.

Practice Thesis Statements

1. It’s 11:30 P.M.; the late news is over, and I can hear my father again, arguing politics with my mother. They’ve never convinced one another the other side was wrong, or come to any agreement, but that doesn’t stop them from this nightly ritual.
2. Americans work more hours that any other industrialized nation in the world. In fact, since
1990, the average annual work total in the United
States has increased by one week.
3. What do Bill Gates, Debbie Fields, and my cousin
Len have in common? Yes, they all started multimillion-dollar businesses. But more important, they had the same opportunities that countless other people had, and they worked hard to turn those opportunities into success stories.

The following are provided as examples. Your thesis statements will differ. Evaluate your work by asking:





Is it clear and direct?
Does it relate to the topic?
Is it clichéd?
Is it confident or are there disclaimers that diminish its strength?

1. Trial and suffering are not the only ways through which to triumph; many people of character have achieved great success while living lives of ease and comfort.
2. Education is about expanding one’s view of the world; to become truly educated, one must learn about and study different ideas and perspectives without passing judgment.
3. The importance of studying history should not be overestimated; it is not an objective collection of data, but rather a subjective retelling of a few selected events.

74

C H A P T E R

4

Practice Test 1

Section 1
Time: 25 minutes
Essay
Directions: In the essay, you will demonstrate how well you develop and present ideas. Your goal is to clearly use language, firmly take a point of view, and logically advance your argument.
You must use only the space provided, which will be adequate if you pay attention to handwriting size and margins. Avoid leaving extra space, such as through double-spacing or leaving a blank line between paragraphs.
Write legibly so the scorers of your essay can understand what you have written. You may use your test booklet to take notes and organize your thoughts, but only what is written on the answer sheet will be scored.
There are 25 minutes in which to write your essay. Carefully read the prompt and your assignment. Respond only to the assignment—off-topic essays will receive a zero.

75

– PRACTICE TEST 1 –

Success is the ability to go from one failure to another with no loss of enthusiasm.
—Winston Churchill

Assignment: Persistence in the face of failure is not always rewarded by success. Many people struggle for years without achieving their goal(s). Is there a point at which it makes sense to “throw in the towel?” Do you agree or disagree with the familiar saying “if at first you don’t succeed, try, try again”? Plan your response, and then write an essay to explain your viewpoint. Support your position with specific examples and evidence, which may be from personal experience or observations; your reading or studies; or knowledge of subjects such as history, literature, and science.

76

– PRACTICE TEST 1 –

77

– PRACTICE TEST 1 –

78

– PRACTICE TEST 1 –

Section 2
Time: 25 minutes
35 multiple-choice questions

ANSWER SHEET:
1.
2.
3.
4.
5.
6.
7.
8.
9.
10.
11.
12.
13.
14.
15.
16.
17.
18.
19.
20.

a a a a a a a a a a a a a a a a a a a a b b b b b b b b b b b b b b b b b b b b c c c c c c c c c c c c c c c c c c c c d d d d d d d d d d d d d d d d d d d d e e e e e e e e e e e e e e e e e e e e 21.
22.
23.
24.
25.
26.
27.
28.
29.
30.
31.
32.
33.
34.
35.

79

a a a a a a a a a a a a a a a

b b b b b b b b b b b b b b b

c c c c c c c c c c c c c c c

d d d d d d d d d d d d d d d

e e e e e e e e e e e e e e e

– PRACTICE TEST 1 –

General Directions: Choose the best answer to each question and fill in the oval that matches that answer on your answer sheet. Errors must be erased completely.

Directions: Each of the following sentences has four underlined words or phrases. Read each sentence and determine which underlined portion, if any, has an error in grammar, usage, word choice, or idiom (standard expression). If there is no error, select choice e—No error. No sentence has more than one error.

1. If you’re bothered by pesky telemarketers constantly interrupting you, register with the National a b c Do Not Call Registry to add you’re home and/or cell phone number to their list. No error. d e
2. Last year’s runner up and her presented the award together at this year’s well-attended ceremony. a b c d
No error. e 3. During the past decade, we experience many changes in technology that greatly improved the quality a b c of our lives. No error. d e
4. Fermat’s last theorem survived for 350 years as a hypothesis until English mathematician Andrew Wiles, a who devoted most of his career to the problem, proved it in 1994. No error. b c d e
5. From 1947 to 1956, thousands of scrolls and fragments of ancient manuscripts has been found in caves a on the shore of the Dead Sea, including early copies of biblical books in Hebrew and Aramaic. No error. b c d e
6. Historians agree that the combat mission was a failure because the soldiers were inadequate armed. a b c d
No error. e 81

– PRACTICE TEST 1 –

7. Due to increased interest on the subject of motocross racing, our county library now has the a b c d
AMA Pro Racing Series Guide. No error. e 8. Between all of the members of their team, they couldn’t decide who should throw out the first pitch. a b c d
No error. e 9. Ron’s untimely entrance on stage disrupted the scene and caused the actors to flub they’re lines. No error. a b c d e 10. Because we experience temperatures below freezing and having mild summers, we can’t garden with a b c d tropical plants. No error. e 11. Having a membership to a health club has helped me to lose weight, increase my overall fitness level, a b c and introduced me to new friends. No error. d e
12. While only seven of Emily Dickinson’s poems were published in her lifetime, she wrote 1800 over a b c the course of thirty-five years. No error. d e
13. If one is looking to buy a used car, you should check the classified section of the local paper, where many a b c sellers advertise. No error. d e
14. Recent studies show that certain plant species developed by man are very susceptible to some diseases a b c d than native species. No error. e 15. Eliza laid down on her bed to rest while the rest of her family enjoyed the appetizing meal prepared by a b c d her grandmother. No error. e 82

– PRACTICE TEST 1 –

16. The best dish on the menu is Tiramisu, a dessert comprised mainly of mascarpone cheese and ladyfingers, a b c that translate from Italian as “pick me up.” No error. d e
17. Pouring over thousands of applications each spring, the Admissions Committee decides who will be in the a b c next freshman class. No error. d e
18. The more I read about deep sea fishing, the more it makes me want to get out there and try it. No error. a b c d e 19. The difference between you and me is that you get your best work done in the morning while I a b c perform better in the evening. No error. d e

83

– PRACTICE TEST 1 –

22. Awkwardly tall and prone to tripping over her own feet, Grace felt her name was truly a misnomer. a. Awkwardly tall and prone to tripping over her own feet,
b. Because she is awkwardly tall and prone to tripping over her own feet,
c. Awkwardly tall and prone to trip over her own feet, d. Grace is awkwardly tall and prone to tripping over her own feet.
e. Awkwardly tall in height, and prone to tripping over her own two feet.

Directions: In each of the following sentences, part or all of the sentence is underlined. The underlined text may contain an error in sentence construction, grammar, word choice, or punctuation. Choice a repeats the original underlined text. If there is no error in the underlined portion, choose a. If there is an error, select the answer choice that most effectively expresses the meaning of the sentence without any ambiguity or awkwardness. 20. To make garlic roasted potato skins, bake six russet potatoes and roast one small head of garlic; then, squeeze the garlic into a small bowl and mash with two tablespoons of butter, salt, and pepper. a. To make garlic roasted potato skins, bake six russet potatoes and roast one small head of garlic; b. To make garlic roasted potato skins, first you bake six russet potatoes and roast one small head of garlic,
c. To make garlic roasted potato skins, in the oven bake six russet potatoes and roast one small head of garlic,
d. Bake six russet potatoes and roast one small head of garlic to make garlic roasted potato skins; e. Making garlic roasted potato skins, bake six russet potatoes and roast one small head of garlic in the oven for 40 minutes;

23. Kyle was able to surmount the difficulties of an uncooperative staff, an impossible deadline, and a complicated project; in order to present the report to the client.
a. an uncooperative staff, an impossible deadline, and a complicated project; in order to present the report to the client.
b. an uncooperative staff, an impossible deadline, and a complicated project in order to present the report to the client.
c. an uncooperative staff, an impossible deadline, and complicated project in order to present the report to the client.
d. an uncooperative staff, impossible deadline, and a complicated project in order to present the report to the client.
e. a staff that was uncooperative, an impossible deadline, and a project that was complicated in order to present the report to the client.

21. I need to get a handle on my credit-card charging, and my budget will be in serious trouble.
a. charging, and my budget will be in serious trouble. b. charging. My budget will be in serious trouble.
c. charging. My budget has been in serious trouble. d. charging, or my budget will be in serious trouble. e. charging, or my budget was in serious trouble.

84

– PRACTICE TEST 1 –

26. The Netherlands, often wrongly referred to as
Holland, are best known for artists such as Rembrandt and Van Gogh, tulips, windmills, and social tolerance.
a. The Netherlands, often wrongly referred to as
Holland, are best known for artists such as
Rembrandt and Van Gogh, tulips, windmills, and social tolerance.
b. The Netherlands, which are often wrongly referred to as Holland, are best known for artists such as Rembrandt and Van Gogh, tulips, windmills, and social tolerance.
c. The Netherlands, wrongly referred to as Holland, are known for artists such as Rembrandt and Van Gogh, tulips, windmills, and tolerance. d. The Netherlands, often wrongly referred to as
Holland, is best known for artists such as
Rembrandt and Van Gogh, tulips, windmills, and social tolerance.
e. The Netherlands, which are often wrongly referred to as Holland, is best known for artists such as Rembrandt and Van Gogh, tulips, windmills, and social tolerance.

24. Ariana was outstanding as the moderator; she handled the intensely heated debate with great finesse, diplomatically and tactfully keeping the conversation on track.
a. she handled the intensely heated debate with great finesse, diplomatically and tactfully keeping the conversation on track.
b. she handled the intensely heated debate with great finesse; diplomatically and tactfully keeping the conversation on track.
c. she handled the debate with great finesse, keeping the conversation on track.
d. she handled the intensely heated debate with great finesse, and then she diplomatically and tactfully kept the conversation on track.
e. the intensely heated debate was handled with great finesse, and the conversation was diplomatically and tactfully kept on track.
25. When she saw that our rivals were wearing the same one, another track uniform was ordered by our coach.
a. another track uniform was ordered by our coach. b. another track uniform that was different was ordered by our coach.
c. while our coach ordered another track uniform. d. our coach will order another track uniform.
e. our coach ordered another track uniform.

27. Cable television is revolutionary in its capacity to provide services to users, also offering Internet access that replaces slow, unreliable dial-up service. a. also offering Internet access that replaces slow, unreliable dial-up service.
b. while offering Internet access that replaces slow, unreliable dial-up service.
c. while, offering Internet access that replaces slow, unreliable dial-up service.
d. while they offer access to the Internet that replaces dial-up service that is slow and unreliable. e. including offering Internet access that replaces slow, unreliable dial-up service.

85

– PRACTICE TEST 1 –

30. Like the Allies and the Nazis, our team overcame a great power to win the championship.
a. Like the Allies and the Nazis,
b. As with the Allies and the Nazis,
c. Compared to the Allies and the Nazis,
d. Similarly to how the Allies vanquished the
Nazis,
e. Like the Allies vanquished the Nazis,

28. He broke the railing while vacationing on his hotel room balcony, but paid for all of the necessary repairs.
a. He broke the railing while vacationing on his hotel room balcony,
b. He broke the railing while vacationing on his hotel room balcony;
c. While vacationing, he broke the railing on his hotel room balcony,
d. He broke the balcony railing while vacationing on his hotel room,
e. While on vacation, he broke the railing that was on his hotel room balcony,
29. As the personal PC revolution continued, clunky bulky monitors were replaced by sleek spacesaving flat-screen displays.
a. clunky bulky monitors were replaced by sleek space-saving flat-screen displays.
b. monitors, which were clunky and bulky, were replaced by displays that have flat screens and save space.
c. clunky, bulky monitors were replaced by flatscreen displays that are sleek and save space as well. d. sleek, space-saving flat-screen displays replaced clunky, bulky monitors.
e. there was a replacement of the clunky, bulky monitors by sleek ones with flat screens that save space.

86

– PRACTICE TEST 1 –

Directions: Questions 31–35 are based on the following passage, a first draft of an essay about the Cold War. Read the passage and the questions that follow. For each question, choose the answer that will most improve the passage. Some questions ask you to choose the best revision of a particular sentence or pair of sentences. Other questions ask you to consider how to best improve the overall organization of the passage. In each case, the correct answer is the one that most closely conforms to the conventions of formal writing.
(1)The Cold War is one of the most interesting and troubling times in American history. (2)Several important historical events led to the Cold War. (3)First, in 1939, Albert Einstein wrote a letter to President Franklin D.
Roosevelt. (4)In that letter, Einstein told Roosevelt that it was possible to create an atomic weapon, and he asked for funds to aid research and experimentation. (5)Roosevelt agreed, and the government created the Manhattan Project, a massive effort to develop nuclear weapons. (6)Next, the date that will live in infamy: August 6,
1945. (7)The United States dropped an atomic bomb on Hiroshima, Japan. (8)An estimated 150,000 civilians were killed in the attack. (9)President Truman claimed that dropping the bomb was necessary to force Japan to surrender and end World War II. (10)Others argue that we used the bomb to show the Soviet Union that we were the superior world power. (11)Though the United States and the U.S.S.R. were officially allies, tensions between the two countries were high. (12)A deep ideological battle between Communism and Capitalism was already in place.
(13)In 1947, the Truman Doctrine was established, redefining American foreign policy with a “policy of containment” which framed our foreign policy as a battle between “good” and “evil.” (14)This dramatically increased the growing tension between the two countries.

87

– PRACTICE TEST 1 –

34. The revision to sentence 13 (reprinted below) that would most improve the essay is:
(13)In 1947, the Truman Doctrine was established, redefining American foreign policy with a “policy of containment” which framed our foreign policy as a battle between “good” and “evil.”
a. In 1947, American foreign policy was redefined by the Truman Doctrine, which presented our foreign policy as a “policy of containment,” which framed our foreign policy as a battle between “good” and “evil.”
b. In 1947, American foreign policy was redefined by the Truman Doctrine, making our foreign policy a “policy of containment,” and framing our foreign policy as a battle between
“good” and “evil.”
c. American foreign policy was redefined by the
Truman Doctrine in 1947, turning it into a “policy of containment,” which framed our foreign policy as a battle between “good” and “evil.”
d. The Truman Doctrine was established in 1947 as a battle between “good” and “evil” that redefined American foreign policy and presented our foreign policy as a “policy of containment.”
e. The Truman Doctrine redefined American foreign policy in 1947 as one of containment, framing the policy as a battle between “good” and “evil.”

31. What is the most fitting title for the essay?
a. Einstein’s Role in the Truman Doctrine
b. Hiroshima—Battle Between Good and Evil
c. The Cold War in Historical Context
d. The Manhattan Project and the Cold War
e. 1945—The Beginning of the Cold War
32. Which of the following is the most effective combination of sentences 1 and 2 (reprinted below)?
(1)The Cold War is one of the most interesting and troubling times in American history. (2)Several important historical events led to the Cold War.
a. Several important historical events led to the
Cold War, which was one of the most interesting and troubling times in American history.
b. The Cold War was led to by several important historical events, one of the most interesting and troubling times in American history.
c. Several important historical events were the most interesting and troubling times in American history, and led to the Cold War.
d. The Cold War, which was one of the most interesting and troubling times in American history, was led to by several important historical events.
e. One of the most interesting and troubling times in American history was the period of time which we call the Cold War; several important historical events led to the Cold
War.

35. In the context of the passage, which of the following is the most effective revision of sentence
10 (reprinted below)?
(10)Others argue that we used the bomb to show the Soviet Union that we were the superior world power. a. Place a comma after superior.
b. Change the present tense verb argue to the past tense argued.
c. Remove the word that both times it appears.
d. Insert the phrase some might say wrongly after the word argue.
e. Leave the sentence as is.

33. The best paragraphing revision to this essay would be which of the following?
a. Combine paragraphs 1 and 2.
b. Start a third paragraph with sentence 6.
c. Start a third paragraph with sentence 5.
d. Start a third paragraph with sentence 9.
e. No change to existing paragraph structure is necessary. 88

– PRACTICE TEST 1 –

Section 3
Time: 10 minutes
14 multiple-choice questions

ANSWER SHEET:
1.
2.
3.
4.
5.
6.
7.

a a a a a a a

b b b b b b b

c c c c c c c

d d d d d d d

e e e e e e e

8.
9.
10.
11.
12.
13.
14.

89

a a a a a a a

b b b b b b b

c c c c c c c

d d d d d d d

e e e e e e e

– PRACTICE TEST 1 –

3. Ms. Lu allowed her son a great deal of latitude in spending his birthday money, because she believed it should be his decision.
a. because she believed
b. and she believed
c. as a result, she believed
d. in spite of the fact that she believed
e. because she believes

Directions: In each of the following sentences, part or all of the sentence is underlined. The underlined text may contain an error in sentence construction, grammar, word choice, or punctuation. Choice a repeats the original underlined text. If there is no error in the underlined portion, choose a. If there is an error, select the answer choice that most effectively expresses the meaning of the sentence without any ambiguity or awkwardness. 4. As the saying goes better late than never.
a. As the saying goes better late than never.
b. As the saying goes; better late than never.
c. As the saying goes: better late than never.
d. As the saying goes better late than never!
e. As the saying goes, better late than never.

1. The numerous beneficial effects of taking a vacation include stress reduction and the increased ability to concentrate, it’s no wonder that managers recommend that all employees take their full two weeks.
a. concentrate, it’s no wonder
b. concentrate so that it’s no wonder
c. concentrate; it’s no wonder
d. concentrate so it’s no wonder
e. concentrate,

5. Riding through town on the way to her orthodontist appointment, which was last Tuesday.
a. Riding through town on the way to her orthodontist appointment, which was last Tuesday.
b. Riding through town on the way to her orthodontist appointment, that was last Tuesday.
c. She rode through town on the way to her orthodontist appointment, which was last
Tuesday.
d. Last Tuesday, riding through town on the way to her orthodontist appointment.
e. Riding through town last Tuesday on the way to her orthodontist appointment.

2. Domesticated hounds are fatter than 50 years ago; since 1950, the average hound’s weight has increased six pounds.
a. Domesticated hounds are fatter than 50 years ago; b. Domesticated hound’s are fatter than 50 years ago, c. Domesticated hounds, being fatter than 50 years ago;
d. Domesticated hounds are fatter than they were 50 years ago;
e. Being fatter than 50 years ago,

91

– PRACTICE TEST 1 –

8. When the house on the corner burned down, the victims’ lives were reestablished because the entire neighborhood rallied together to help them. a. the victims’ lives were reestablished because the entire neighborhood rallied together to help them.
b. the victims’ lives were reestablished because of the rallying together of the entire neighborhood in order to help them.
c. the entire neighborhood reestablished their lives when they rallied together to help the victims. d. the victims’ lives were reestablished when the neighborhood rallied together to help them.
e. the entire neighborhood rallied together to help the victims reestablish their lives.

6. Held in 1985, Bob Geldof organized the first Live
Aid Concert to benefit famine victims in Africa.
a. Held in 1985, Bob Geldof organized the first
Live Aid Concert to benefit famine victims in
Africa.
b. Bob Geldof organized the first 1985 Live Aid
Concert to benefit famine victims in Africa.
c. Held in 1985, to benefit famine victims in
Africa, Bob Geldof organized the first Live Aid
Concert.
d. Held in 1985, the first Live Aid Concert to benefit famine victims in Africa was organized by Bob Geldof.
e. Bob Geldof organized the first Live Aid Concert, which was held in 1985, to benefit famine victims in Africa.
7. Anarchists are those who believe any form of government is bad, in contrast totalitarians believe there should be one political party that rules a nation.
a. bad, in contrast totalitarians believe
b. bad, to which totalitarians, believing
c. bad, in contrast to which totalitarians believe
d. bad, in contrast to totalitarians, who believe that
e. bad; in reference to totalitarians, which in contrast are

9. Planning my winter vacation, a few hotels had decent rates for the week in February I’ve got off.
a. Planning my winter vacation,
b. While planning my winter vacation,
c. Since I am planning my winter vacation, I found d. Planning my winter vacation, I find
e. While planning my winter vacation, I found that

92

– PRACTICE TEST 1 –

11. The judge dismissed the extraneous evidence, which the jury seemed interested in, because it was not pertinent to the trial, despite the vociferous objections of the prosecutors.
a. The judge dismissed the extraneous evidence, which the jury seemed interested in, because it was not pertinent to the trial
b. The extraneous evidence which interested the jurors was dismissed by the judge because it was not pertinent to the trial
c. The judge dismissed the evidence which the jury seemed interested in that was extraneous because it was not pertinent to the trial
d. The judge dismissed the extraneous, impertinent evidence because the jury seemed interested in it
e. The judge dismissed, which the jury seemed interested in it, the extraneous evidence because it was not pertinent to the trial

10. If one is reading Dostoevsky, you should probably keep a chart of all the characters so you don’t get confused; many of their names are similar.
a. If one is reading Dostoevsky, you should probably keep a chart of all the characters so you don’t get confused; many of their names are similar. b. If you are reading Dostoevsky, you should probably keep a chart of all the characters so you don’t get confused; many of their names are similar.
c. If one is reading Dostoevsky, you should probably keep a chart of all the characters so you don’t get confused; many of their names are similar. d. If you are reading Dostoevsky, you should probably keep a chart which will help you keep from getting confused by all of the characters’ names because many of them are similar. e. When reading Dostoevsky, keep a chart of all the characters so you don’t get confused; many of their names are similar.

12. Once they are established, perennials need to be dug up and divided every few years, to keep them healthy. a. perennials need to be dug up and divided every few years,
b. perennials need to be dug up and divided every few years
c. dig up perennials and divide them every few years d. perennials are dug up and divided every few years, e. digs and divides them every few years

93

– PRACTICE TEST 1 –

14. With a show-stopping display of reds and oranges, it was the most perfect sunset.
a. With a show-stopping display of reds and oranges, it was the most perfect sunset.
b. It had a show-stopping display of reds and oranges, and it was the most perfect sunset.
c. With a show-stopping display of reds and oranges, it was a perfect sunset.
d. Giving us a show-stopping display of reds and oranges, it was the most perfect sunset.
e. With a show-stopping display, of reds and oranges, it was a perfect sunset.

13. My roommate orders take-out dinners from Oriental Wok at least twice a week, and they’re better than the other Chinese restaurant in town.
a. and they’re better than the other Chinese restaurant in town.
b. because the food is better than that of the other Chinese restaurant in town.
c. she knows that they’re better than the other
Chinese restaurant in town.
d. so they’re better than the other Chinese restaurant in town.
e. they’re better than the other Chinese restaurant in town.

94

– PRACTICE TEST 1 –

Answer Key

agement and the specific essay-writing strategies you learned in Chapter 3. It’s also here for you to compare your finished product with the rubric. The more you practice and perform these evaluations, the better you’ll understand exactly what your scorers are looking for.

Section 1: Essay

Use the following rubric to evaluate your writing. This practice essay is included so you can work on time man-

6









5









4









3










Demonstrates outstanding writing skills
Includes a clear and insightful point of view on the question and reflects excellent critical thinking, using strong examples and other evidence to support this point of view
Contains a strong organization and focus, a clear sense of unity, and a skillful flow of ideas
Demonstrates a strong command of language, with varied and appropriate word choice, and meaningful variation in sentence structure
Contains few, if any, errors in grammar, usage, and mechanics
Demonstrates effective writing skills
Includes a clear point of view on the question and reflects strong critical thinking, using good examples and other evidence to support this point of view
Contains strong organization and focus, a sense of unity, and a flow of ideas
Demonstrates a good command of language, with appropriate word choices and variation in sentence structure
Contains few errors in grammar, usage, and mechanics
Demonstrates competent writing skills, but the quality of the writing may be inconsistent
Includes a point of view on the question and reflects competent critical thinking, using sufficient examples to support this point of view
Contains a general organizational plan and focus, with some unity and flow of ideas
Demonstrates a sufficient but inconsistent command of language, with mostly appropriate word choice and some variation in sentence structure
Contains some errors in grammar, usage, and mechanics
Demonstrates inadequate, but not incompetent, writing skills
Includes a point of view on the question, reflecting some critical thinking, but this point of view may be inconsistent or incomplete, and support may be lacking
Contains a limited organizational strategy and focus, with a weak or inconsistent sense of unity and flow of ideas
Demonstrates a developing but weak command of language, with weak or inappropriate vocabulary, little or no variation in sentence structure, and may contain errors in sentence construction
Contains many errors in grammar, usage, and mechanics

95

– PRACTICE TEST 1 –

2









1








0



Demonstrates limited writing skills and may contain serious flaws
Includes a limited or vague point of view on the question and reflects poor critical thinking, using inadequate or irrelevant examples or other support
Displays a weak sense of organization and/or focus, and may lack unity and/or flow of ideas
Demonstrates an inadequate command of language, with limited or incorrect vocabulary, and incorrect or flawed sentence structure
Contains serious errors in grammar, usage, and mechanics that may make the writing difficult to understand Demonstrates incompetence in writing and contains serious flaws
Does not contain a point of view on the question, or provides little or no support for the point of view
Lacks organization and/or focus, unity, and a flow of ideas
Contains serious errors in vocabulary and sentence structure
Contains serious errors in grammar, usage, and/or mechanics that make the writing difficult to understand An essay that does not answer the question, or is blank, receives a zero.

(Adapted from The College Board)
Katie was a natural, and she picked up the new sport quickly. I, on the other hand, couldn’t seem to hold the lacrosse stick comfortably. I caught one out of ten throws, if I was lucky, and my tosses were always way off their mark. I was clumsy and feeling clumsier, and I thought maybe it was time to give it up. But that would create an even wider gulf between
Katie and me. Already she was spending more and more time with the girls who, like her, excelled at sports. I was beginning to be left behind.
Determined to stick it out and save our friendship, I begged my mom to take me to a sporting goods store and buy me an early birthday present: my own lacrosse stick and ball so I could practice at home. Katie was impressed with my stick, but I could tell that she thought it was a waste of money.
She figured I would never get to use that stick in a game. I was hurt by her reaction, and again I felt the distance between us. If I was going to keep Katie as

Here are examples of a couple of essays written on the assignment: It might sound ironic, but sometimes the best way to gain confidence is to keep trying to accomplish something that seems to bring nothing but failure.
In seventh grade, I had a best friend who was an incredible athlete. I was pretty coordinated myself, but because I was so insecure, I never seemed to be any good at sports. I was so afraid of missing the ball that I would be sure to swing and miss, even if it was right over the plate. But Katie was my best friend, and if she joined a team, I did, too. Or at least
I tried. Katie was a starter for the junior varsity field hockey team; I sat on the bench all season. Katie played regularly in JV basketball; I was cut during tryouts. I figured I was headed for a similar fate with lacrosse. But Katie was my best friend, so I signed up anyway.

96

– PRACTICE TEST 1 –

They say that if you don’t succeed, try, try again until you do. When I was in Junior High School, I tried many sports because my best friend did. She was a great athlete; I was not. I sat on the bench all of field hockey season and I got cut during basketball tryouts, too. I stuck with it, though and finally made it on the lacrosse team.
My friend Katie picked up lacrosse right away, but I struggled. Even though she was my best friend
I couldn’t be partners with her during practice.
Because she was so much better than me. I was afraid that if I didn’t learn how to be good at lacrosse, our friendship would be over. She was spending more and more time with her sports friends, and I was feeling more and more left out.
I decided to do something to save our friendship. I went out and bought a lacrosse stick. After practice, I’d come home and practice. I practiced on weekends, too. I tried and tried and tried. Some days I felt like there wasn’t any hope, but I kept trying. Then one day, it happened. I was throwing and catching the ball with Suzie, my new partner. Suddenly, I caught the ball. I caught the next one she threw, too. My throws to her were accurate. From that day on, I got better and better. I had more confidence, too. I ended up playing a lot that season on the JV team and even scored 12 goals. I’m really glad
I kept trying.

a friend, I thought, I simply had to get the hang of this sport. It was my last chance. Somehow, someway, I had to learn how to throw and catch the ball in that net and be respectable on the playing field.
So I practiced, and I practiced, and I practiced some more. I often felt like there was no hope, and I broke two windows in the garage, but I kept at it.
Then, one day, just after the first official game of the season (during which I sat on the bench), something happened. I paired off with Suzie, who had become my partner since Katie had quickly proven to be too good to play with me. That day, when
Suzie sent me her first throw, I caught it. When I threw the ball back to her, I hit her stick dead on. I caught her next throw, and the next. Something was happening. I was getting it. The stick was actually feeling good in my hands. The movements were becoming natural. I was catching and throwing the ball accurately.
I still don’t know what exactly happened that day, but I will always be grateful for it. By the end of the season, I was starting for the JV team. I scored
12 goals that year, and the next year I was playing varsity. This success makes it easy to forget that just a few years ago, I failed at every sport I tried.
But instead of giving in to my insecurities, I kept trying. Finally, my success on the field gave me confidence that I desperately needed.
This essay received a 6. Notice that it was written completely from personal experience—there are no profound examples taken from history, literature, or even current events. The five-paragraph structure, strong point of view that doesn’t waver, logical flow through use of chronological organization, varied sentence structure, and strong conclusion put the essay at the top. It’s a great example of the fact that you don’t need to sound like a college professor to do well on the essay. Stick to what you know, and follow the plan.

This essay received a 4. Organizationally, it is strong. It follows the same chronological sequence as the first essay, giving it a logical structure. It also maintains a strong point of view. However, the ideas are not developed with the same depth. From that day on
I got better and better is an example of an assertion that isn’t supported or explained. There are also enough grammatical errors to bring the score down. Because she was so much better than me is a sentence fragment. In paragraph three, the word practice is used three times and try/tried is used four times. Synonyms would have added variety. In addition, the conclusion is one short

97

– PRACTICE TEST 1 –

14. c. This sentence is setting up a comparison between two types of species. The first type is more (not very) susceptible than the other.
15. a. The correct past tense form of the verb to lie is lay.
16. d. The verb translate is in the plural form, and does not agree with the singular subject dish.
The correct verb is translates.
17. a. The correct word is poring, which means
“looking over closely.” The verb pouring means “causing a stream-like flow.”
18. c. To maintain parallel structure, both phrases after the words the more must be grammatical equivalents. I read about should be matched with I want to.
19. e. No error. If you chose a, recall that you and me functions as the object of the preposition between. The objective case me is therefore correct. 20. a. Choices b and c add unnecessary words (first you and in the oven). Choice d would work if the second half of the sentence weren’t there; as is, it doesn’t combine logically with what follows. In choice e, the participle making is not grammatically correct.
21. d. Choice a uses the wrong conjunction (and) to express the relationship between the two independent clauses. Choice b creates a new sentence whose meaning isn’t clear; it needs the first clause to make sense. Choice c repeats the choice b error, and uses the wrong verb tense
(the past has been instead of the future will be). Choice e uses the correct conjunction, but the wrong verb tense (the past was).
22. a. The phrase because she is in choice b isn’t wrong, but it makes the sentence less concise than choice a. Choice c uses the wrong verb form; the gerund tripping is needed, rather than the infinitive to trip. In choice d, a new sentence is formed unnecessarily. Choice e is wordy. sentence at the end of the last paragraph. The author doesn’t go back to her point about gaining confidence.
Section 2: Multiple Choice

1. d. The wrong word is used here. It should be the possessive pronoun your rather than the contraction of the words you are.
2. b. The pronoun is part of this sentence’s compound subject. It must therefore be the subjective (she) rather than the objective (her).
3. b. The past decade indicates that this sentence requires the past tense verb experienced.
4. e. There are no errors in this sentence.
5. a. This sentence refers to a discovery that happened in the past. The correct verb is were found. 6. d. Inadequate is an adjective used here to modify a verb armed. To correct the error, change it to the adverb inadequately.
7. a. This is an error in prepositional idiom. The correct phrase is interest in rather than interest on.
8. a. Between is used when discussing two people or things; among is used when there are three or more. Among is correct in this sentence.
9. d. The lines are the actors’, so the correct possessive pronoun is their. They’re is a contraction of the words they are.
10. b. The first verb, experience, is in the simple present tense. To maintain consistency, having should be changed to have.
11. d. The health club membership does three things, all of which should be in the present tense to maintain consistency. Introduced should be changed to introduce.
12. e. This sentence is correct.
13. b. This sentence changes pronouns from the impersonal, indefinite one to the personal, definite you. Since changing one is not an option, the pronoun you should be changed.

98

– PRACTICE TEST 1 –

23. b. The problem with choice a is the use of a semicolon where a comma is needed. Choices c, d, and e correct it, but include errors with parallel structure. Choice c drops the article from complicated project and d drops it from impossible deadline. Choice e changes the grammatical composition of uncooperative staff and complicated project, not only making them unlike impossible deadline, but making them wordy as well.
24. a. A semicolon is wrongly used in choice b.
Choice c removes three adverbs that added meaning to the sentence. Choice d is not concise, and choice e incorrectly uses the passive voice. 25. e. Choice a incorrectly uses the passive voice.
Choice b repeats the error and adds the unnecessary phrase that was different. Choice c correctly uses the active voice, but includes the conjunction while, making the sentence illogical. Choice d also corrects the passive voice issue, but uses the wrong verb tense (will order should be ordered).
26. d. Choice a uses the plural verb are with the singular noun the Netherlands. Choice b repeats the error, and unnecessarily adds the word which to the phrase often wrongly referred to as
Holland. Choice c also repeats the error, and deletes most of the adjectives and adverbs.
Choices d and e correct the subject-verb agreement problem, but e includes the wordy which are from choice b.
27. e. Choices a, b, c, and d all use conjunctions
(also and while) that do not express the correct relationship between the phrase and clause. In addition, c has a superfluous comma after while, and d is wordy.
28. c. Choice a has a misplaced modifier; he wasn’t vacationing on his hotel room balcony. Choice b repeats the error, and uses a semicolon instead of the correct comma. Choice d also

29. d.

30. e.

31. c.

32. a.

33. b.

99

has a misplaced modifier; he wasn’t vacationing on his hotel room. Choice e corrects the modifier problem, but is wordy.
Choice a uses the passive voice and is missing commas between the modifying pairs clunky bulky and sleek space-saving. Choice b uses wordy which and that phrases instead of adjectives. Choices c and e correct the comma problem, but are unnecessarily wordy.
The problem with choices a, b, and c is faulty comparison; the author is comparing his team’s win to two historical opponents. He needs to compare his team’s win to another win, as choices d and e do. Choice d, however, uses the wordy and awkward similarly to how instead of the concise like.
All of the other choices are too specific, and some confuse the facts of the essay. The start of the Cold War was not 1945, Hiroshima is not referred to in the essay as the battle between good and evil, and Einstein played no role in the Truman Doctrine.
Choices b and d both use the gramatically incorrect phrase led to by. In choice c, the facts are confused—the most interesting and troubling times in America were not several important historical events. Choice e is unnecessarily wordy. Choice a would make the essay one large paragraph, while choice e would leave it as one large and one small paragraph. Beginning a paragraph with sentence 5 would break up the discussion of the development of the atomic bomb, and starting a new paragraph with sentence 9 would break up the discussion of
Hiroshima. Sentence 6 is the most logical place to begin a new paragraph, because it introduces another one of the events that led to the Cold War.

– PRACTICE TEST 1 –

34. e. Choice e is the only one that eliminates the repetition of the words foreign and policy. It is the clearest and most concise choice.
35. b. The change in verb tense is necessary to maintain consistency. Choice a would include a superfluous comma, while choice c would make the sentence awkward. The phrase suggested in choice d would need commas around it, and choice e would retain the incorrect verb form.

7. d.

8. e.

Section 3: Multiple Choice

1. c. Choice c is the only one that is not a run-on sentence. All of the others use a comma or no punctuation mark to separate two independent clauses.
2. d. Only choice d corrects the faulty comparison—domesticated hounds are fatter than they were 50 years ago, not fatter than 50 years ago.
3. a. Choices b, c, and d use the wrong conjunctions (and, as a result, in spite of the fact) to express the relationship between the two phrases. The first phrase depends on, or is subordinate to, the second, making because the correct word. Choice e also uses the right conjunction, but it incorrectly uses the present tense verb believes.
4. e. As the saying goes is a phrase that introduces the clause better late than never. Introductory words and phrases should always end with a comma. 5. c. Choice c is the only one that’s a complete sentence. All of the other choices are sentence fragments. 6. e. Choices a and c have misplaced modifiers; Bob
Geldof was not held in 1984. Choice b corrects the error but introduces a new one. The meaning of the sentence is lost, because the concert wasn’t the first 1984 one. Choice d also

9. e.

10. e.

11. a.

12. b.

100

corrects the error, but it is not as clear and concise as choice e.
Choice a is a run-on sentence. Choice b uses illogical and ungrammatical sentence structure. Choice c incorrectly uses the wordy phrase to which. Choice e creates a fragment by using a semicolon and is wordy.
The problem is incorrect use of the passive voice. Notice how choice e is more direct and concise than the other four versions. While choice c also uses the active voice, its word order changes the meaning of the sentence.
The problem is a misplaced modifier—the hotels aren’t planning the vacation. Choice e is the clearest way to correctly express the idea of the sentence. Choices b and d include, with a slight variation, the original error, and choice d also uses the wrong verb tense.
Choice e is the most clear and concise.
Choices a and c shift from the impersonal pronoun one to the personal you. Choices b and d correct that error, but retain the wordy phrase you should probably. Choice d also changes the wording and eliminates the semicolon, creating a long and confusing sentence unbroken by punctuation.
Choice b incorrectly uses the passive voice to create an awkward construction. Choice c needs commas to separate the interrupter phrase which the jury seemed interested in.
Choice c is also wordy. Choice d tightens up the language of the sentence, but loses the meaning. In choice e, the interrupter phrase, which modifies the noun evidence, is misplaced. The comma after years is superfluous in choices a and d. Choice d also uses the wrong verb tense. The introductory phrase once they are established refers to perennials, so choice c, which uses the implied subject you (you dig them up and divide them) is confusing. You

– PRACTICE TEST 1 –

aren’t what’s established. Choice e also uses the wrong verb tense; the future progressive tense is needed rather than the simple present.
13. b. Choice b is the only one that corrects the faulty comparison and clears up the ambiguous pronoun. What does they refer to? It’s supposed to refer to the restaurant, but in the original sentence, as well as in choices c, d, and e, it refers to the take-out dinners. Comparing dinners to a restaurant is not the intended meaning.

101

14. c. Choices a, b, and d include a faulty comparison; something can’t be more perfect than something else—it’s either perfect, or it’s not.
Choices c and e correct the error, but choice e has a superfluous comma after display.

C H A P T E R

5

Practice Test 2

Section 1
Time: 25 minutes
Essay
Directions: In the essay, you will demonstrate how well you develop and present ideas. Your goal is to clearly use language, firmly take a point of view, and logically advance your argument.
You must use only the space provided, which will be adequate if you pay attention to handwriting size and margins. Avoid leaving extra space, such as through double-spacing or leaving a blank line between paragraphs.
Write legibly so the scorers of your essay can understand what you have written. You may use your test booklet to take notes and organize your thoughts, but only what is written on the answer sheet will be scored.
There are 25 minutes in which to write your essay. Carefully read the prompt and your assignment. Respond only to the assignment—off-topic essays will receive a zero.

103

– PRACTICE TEST 2 –

Money never made a man happy yet, nor will it.
—Benjamin Franklin
But for money and the need of it, there would not be half the friendship in the world. It is powerful for good if divinely used. Give it plenty of air and it is sweet as the hawthorne; shut it up and it cankers and breeds worms.
—George MacDonald

Assignment: What is your view on the uses of money? Choose the quote you disagree with most strongly, and write an essay explaining your position. Support that position with evidence and examples from literature, the arts, history, current events, politics, science and technology, and/or from personal experiences or observations.

104

– PRACTICE TEST 2 –

105

– PRACTICE TEST 2 –

106

– PRACTICE TEST 2 –

Section 2
Time: 25 minutes
35 multiple-choice questions

ANSWER SHEET:
1.
2.
3.
4.
5.
6.
7.
8.
9.
10.
11.
12.
13.
14.
15.
16.
17.
18.
19.
20.

a a a a a a a a a a a a a a a a a a a a b b b b b b b b b b b b b b b b b b b b c c c c c c c c c c c c c c c c c c c c d d d d d d d d d d d d d d d d d d d d e e e e e e e e e e e e e e e e e e e e 21.
22.
23.
24.
25.
26.
27.
28.
29.
30.
31.
32.
33.
34.
35.

107

a a a a a a a a a a a a a a a

b b b b b b b b b b b b b b b

c c c c c c c c c c c c c c c

d d d d d d d d d d d d d d d

e e e e e e e e e e e e e e e

– PRACTICE TEST 2 –

General Directions: Choose the best answer to each question and fill in the oval that matches that answer on your answer sheet. Errors must be erased completely.

Directions: Each of the following sentences has four underlined words or phrases. Read each sentence and determine which underlined portion, if any, has an error in grammar, usage, word choice, or idiom (standard expression). If there is no error, select choice e—no error. No sentence has more than one error.

1. Although Impressionism is perhaps best known for the work of its male artists, painter Berthe Morisot a was one of its central figures, embraced by Degas, Monet, and Renoir for her loosely brushstroke and b c use of color. No error. d e
2. Television executives heartily embrace Reality TV, which began in the 1940s with Candid Camera, for a b two reasons: It is much cheaper to produce than any other type of programming, and its incredibly c d popular. No error. e 3. Although Dick Dale was best known for his contributions to surf music and earns the moniker “King of a b the Surf Guitar,” he was also referred to as the “Father of Heavy Metal” because of his unique playing style c d and pioneering use of Fender guitars and amplifiers. No error. e 4. The game Monopoly, originally called The Landlord’s Game, was based on the economic theories of a Henry George, who proposed a single federally tax based on land ownership; he believed this tax would b weaken the ability to form monopolies, encourage equal opportunity, and narrow the gap between c rich and poor. No error. d e

109

– PRACTICE TEST 2 –

5. Compact discs (CDs), which may be found in over 25 million American homes, first entered popular a b culture in the 1980s, decades after inventor James Russell decided to create a system that could record, storage, and replay music without ever wearing out. No error. c d e 6. When a plant is pollinated, seeds will form at the base of the flower pod, which is found above the stem a b c d and below the blossom. No error. e 7. Jazz great Pat Metheny has been influenced by many other performers, most notably Brazilian guitarist a b and composer Toninho Horta, legendary American saxophonist Ornette Coleman, and Latin music in c d general. No error. e 8. The top sheet must be placed on the bottom, or fitted, sheet with enough overhang on three sides to be a b c able to tuck it between the boxspring and mattress. No error. d e
9. Former actor John Robert Powers has had a huge affect on our culture; after opening the first modeling a b agency in 1923, he began the practice of selling everything from magazines to food items to c vacation destinations with the help of beautiful people. No error. d e
10. To make cucumber salad, you will need to slice a peeled cucumber as finely as you can, and then a b macerate them in sugar, vinegar, salt, and chopped dill. No error. c d e 11. Keeping your room uncluttered is easy when you make it a habit to spend 10 minutes a day just putting a b c things back where they belonged. No error. d e

110

– PRACTICE TEST 2 –

12. The quadrants of a backgammon board are known as the home boards and outer boards, and these a b boards are separated by each other with a ridge down the center of the board called the bar. No error. c d e 13. “Eat, drink, and be merry,” is a label associated with Greek philosopher Epicurus, but like most catchy a b slogans, they simplify what is actually a rich and complex message. No error. c d e 111

– PRACTICE TEST 2 –

Directions: In each of the following sentences, part or all of the sentence is underlined. The underlined text may contain an error in sentence construction, grammar, word choice, or punctuation. Choice a repeats the original underlined text. If there is no error in the underlined portion, choose a. If there is an error, select the answer choice that most effectively expresses the meaning of the sentence without any ambiguity or awkwardness. 14. Feeling like she was underappreciated, and growing tired of her job which was boring, Celia began to ponder what it would be like to quit.
a. Feeling like she was underappreciated, and growing tired of her job which was boring,
b. Feeling underappreciated and tired of her boring job,
c. Feeling like she was underappreciated, and growing tired of her boring job,
d. Feeling underappreciated, and tired of her boring job;
e. Feeling underappreciated, and growing tired of her job which was boring,
15. A handful of novels by John Irving were adapted for the big screen two have new titles, “The Door in the Floor” was A Widow for One Year and
“Simon Birch” was A Prayer for Owen Meany.
a. A handful of novels by John Irving were adapted for the big screen two have new titles;
b. A handful of novels by John Irving were adapted for the big screen, two have new titles;
c. A handful of novels by John Irving were adapted for the big screen; two have new titles:
d. A handful of novels by John Irving were adapted for the big screen; two have new titles;
e. A handful of novels by John Irving were adapted for the big screen. Two have new titles; 112

16. The CEOs large expense account’s proved she was a liberal spender of the companies money.
a. The CEOs large expense account’s proved she was a liberal spender of the companies money.
b. The CEOs large expense accounts proved she was a liberal spender of the company’s money.
c. The CEO’s large expense accounts proved she was a liberal spender of the companys money.
d. The CEO’s large expense account’s proved she was a liberal spender of the company’s money.
e. The CEO’s large expense accounts proved she was a liberal spender of the company’s money.
17. The itinerant construction crew built large new buildings all over the South, wherever the demand for qualified workers took them.
a. The itinerant construction crew built large new buildings all over the South, wherever the demand for qualified workers took them.
b. The itinerant construction crew built all over the South large new buildings wherever the demand for qualified workers took them.
c. The itinerant construction crew, qualified workers, built large new buildings all over the south, wherever the demand took them.
d. Wherever the demand took them, the itinerant construction crew were qualified workers who built large new buildings all over the
South.
e. All over the south, wherever the demand for qualified workers took them, the itinerant construction crew built large new buildings.

– PRACTICE TEST 2 –

18. Since the average American generates four pounds of trash per day, which adds up to
600,000 tons per day or 210 million tons per year.
a. Since the average American generates four pounds of trash per day, which
b. Since the average American generates four pounds of trash per day, that
c. The average American generates four pounds of trash per day, which
d. Because the average American generates four pounds of trash per day, which
e. The average American generates four pounds of trash per day, and that number
19. A popular style of electronic dance music gets its name from the Warehouse Club in Chicago, for regulars referred to their favorite mixes by DJ
Frankie Knuckles as House Music.
a. for regulars referred to their favorite mixes by
DJ Frankie Knuckles as House Music.
b. where regulars referred to their favorite mixes by DJ Frankie Knuckles as House Music.
c. since regulars referred to their favorite mixes by DJ Frankie Knuckles as House Music.
d. now that regulars referred to their favorite mixes by DJ Frankie Knuckles as House
Music.
e. so regulars referred to their favorite mixes by
DJ Frankie Knuckles as House Music.

20. I headed over to the shoe sale at 9 o’clock in the morning, but was disappointed by the meager selection. a. I headed over to the shoe sale at 9:00 A.M. in the morning, but was disappointed by the meager selection.
b. I headed over to the shoe sale at 9:00 A.M.; and was disappointed by the meager selection.
c. I headed over to the shoe sale at 9:00 a.m. in the morning, but was disappointed by the poor selection.
d. I headed over to the shoe sale at 9:00 A.M., but was disappointed by the meager selection.
e. I went to the shoe sale at 9:00 A.M. in the morning, but was disappointed by the meager selection. 21. The politician’s reelection that he has been campaigning for is being threatened by a faction of union members that is disgruntled that is growing larger by the day.
a. The politician’s reelection that he has been campaigning for is being threatened by a faction of union members that is disgruntled that is growing larger by the day.
b. The campaigning politician’s reelection is being threatened by a growing faction of disgruntled union members.
c. The politician’s reelection that he has been campaigning for is being threatened by a growing faction of union members that is disgruntled. d. The campaigning politician’s reelection is being threatened by a faction of disgruntled union members that is growing larger by the day.
e. The campaigning politician’s reelection is being threatened by a growing faction of union members that is disgruntled.

113

– PRACTICE TEST 2 –

24. To teach an exciting marine biology class, a visit to the aquarium is a must.
a. a visit to the aquarium is a must.
b. one should take the class to the aquarium.
c. a visit to the aquarium should be in the curriculum. d. the class must visit the aquarium.
e. you must write the syllabus to contain a visit to the aquarium.

22. Castling, a defensive chess maneuver and the only one in which two pieces are moved in the same turn, are so named because of the safety afforded to the king through the move.
a. turn, are so named because of the safety afforded to the king through the move.
b. turn; are so named because of the safety afforded to the king through its move.
c. turn, is named that way because of the safety that is afforded to the king because of the move. d. turn, are so named because of the king’s safety afforded through the move.
e. turn, is so named because of the safety afforded to the king through the move.

25. The Four Corners Region in United States geography refers to the place at which the corners of four states, Colorado, New Mexico, Arizona, and
Utah, meet.
a. the place at which the corners of four states,
b. when the corners of four states,
c. the place at which the corners of four states:
d. the place for which the corners of four states,
e. the location that is where the corners of four states, 23. I find it fascinating both the number of Supreme
Court cases that were decided by a 5–4 vote, and that the swing vote is not always cast by the same judge or judges.
a. I find it fascinating both the number of
Supreme Court cases that were decided by a
5–4 vote, and that
b. I find it fascinating that both the number of
Supreme Court cases decided by a 5–4 vote, and that
c. I find it fascinating both such a high number of the Supreme Court cases were decided by a
5–4 vote, and that
d. I am fascinated by the number of Supreme
Court cases decided by a 5–4 vote, and by the fact that
e. I am fascinated by both the number of
Supreme Court cases that were decided by a
5–4 vote, and by the fact that

26. The use of recycled rubber playground surfaces should be considered by every elementary school in the country; not only are they better at preventing injury than traditional surfaces, but they are good for the environment, too.
a. The use of recycled rubber playground surfaces should be considered by every elementary school in the country;
b. The use of recycled rubber playground surfaces is considered by every elementary school in the country;
c. Every elementary school in the country should consider the use of recycled rubber playground surfaces;
d. Every elementary school in the country should consider using recycled rubber playground surfaces; e. The use of recycled rubber playground surfaces are considered by every elementary school in the country;

114

– PRACTICE TEST 2 –

29. Alfred Nobel invented dynamite while developing a safe way to handle nitroglycerin, while
Ascanio Sobrero, the Italian chemist who invented nitroglycerin, did not achieve fortune and fame from his invention.
a. Alfred Nobel invented dynamite while developing a safe way to handle nitroglycerin, while
b. Alfred Nobel invented dynamite while developing a safe way to handle nitroglycerin, but
c. Alfred Nobel invented dynamite, which brought him wealth and international notoriety, while developing a safe way to handle nitroglycerin; d. Alfred Nobel invented dynamite, that makes him wealthy and famous, while developing a safe way to handle nitroglycerin;
e. Alfred Nobel invented dynamite while developing a safe way to handle nitroglycerin, and

27. The yoga instructor waited patiently for her students to find the proper pose, even though she performed it with ease.
a. pose, even though she performed it with ease.
b. pose, because she performed it with ease.
c. pose; even though she performed it with ease.
d. pose, where she performed it with ease.
e. pose, for she was able to perform it with ease.
28. Also known as action painting and the New York school, the 1940s saw the emergence of abstract expressionism, an art movement exemplified by the works of Jackson Pollock and Willem de
Kooning.
a. school, the 1940s saw the emergence of abstract expressionism, an art movement exemplified by the works of Jackson Pollock and Willem de Kooning.
b. school, the 1940s saw the emergence of an art movement, abstract expressionism, that is exemplified by the works of Jackson Pollock and Willem de Kooning.
c. school, the art movement abstract expressionism emerged in the 1940s. It is exemplified by the works of Jackson Pollock and Willem de
Kooning.
d. school, abstract expressionism, exemplified by the works of Jackson Pollock and Willem de
Kooning, is an art movement that saw its emergence in the 1940s.
e. school, the 1940s saw the emergence of an art movement known as abstract expressionism that exemplified by the works of Jackson Pollock and Willem de Kooning.

115

– PRACTICE TEST 2 –

Directions: Questions 30–35 are based on the following passage, a first draft of an essay about the coast of Maine.
Read the passage and the questions that follow. For each question, choose the answer that will most improve the passage. Some questions ask you to choose the best revision of a particular sentence or pair of sentences. Other questions ask you to consider how to best improve the overall organization of the passage. In each case, the correct answer is the one that most closely conforms to the conventions of formal writing.
(1)The coast of the State of Maine is one of the most irregular in the world. (2)It is deeply indented with many coves and harbors, and numerous small islands dot the shoreline. (3)In fact, this area is a prime example of what geologists refer to as a drowned coastline. (4)Draw a straight line from the southernmost city in Maine, Kittery, to the northernmost coastal city, Eastport. (5)This line would measure about 225 miles. (6)Follow the coastline between the same two cities. (7)The distance is more than ten times as far. (8)What is the cause of this irregularity? (9)The ice age, or age of continental glaciation, which occurred thousands of years ago.
(10)There was so much ice over North America that the continent reacted isostatically—it sank. (11)Isostatic is a geology term meaning pertaining to alterations in the height of the earth’s crust relative to the sea.
(12)As glaciers moved over the land, their weight expended enormous force on the mountains. (13)The mountains sank into the sea. (14)The original coastline is now under water. (15)It has been rising isostatically since the ice melted. (16)The Maine coast has risen at least 600 feet because we find ice age beaches 600 feet above the present sea level.

116

– PRACTICE TEST 2 –

32. Which sentence best describes the main idea of the essay?
a. sentence 1
b. sentence 2
c. sentence 9
d. sentence 11
e. sentence 16

30. Which of the following revisions represents the best way to combine sentences 12 and 13?
(12)As glaciers moved over the land, their weight expended enormous force on the mountains.
(13)The mountains sank into the sea.
a. The weight of the glaciers was expending an enormous force on the mountains as it moved over the land, causing the mountains to sink into the sea.
b. As glaciers moved over the land, their weight expended an enormous force on the mountains, causing them to sink into the sea.
c. The weight of the glaciers caused the mountains to sink into the sea because it then expended an enormous force on them as it moved over the land.
d. As they moved over the land, the weight of the glaciers expended an enormous force on the mountains, causing them to sink into the sea.
e. The weight of the glaciers as they moved over the land was expending an enormous force on the mountains which then caused them to sink into the sea.

33. Which of the following sentences would be best inserted before sentence 15 to introduce a new paragraph? (14)The original coastline is now under water.
(15)It has been rising isostatically since the ice melted. (16)The Maine coast has risen at least 600 feet because we find ice age beaches 600 feet above the present sea level.
a. However, coastlines don’t move in just one direction. b. Just like things can sink down, they can also rise up.
c. But as Einstein said, things that go up must come down.
d. In addition, it is also coming back up.
e. Let’s look at another way in which the coastline moves.

31. What is the relationship between sentences 4 and 5?
(4)Draw a straight line from the southernmost city in Maine, Kittery, to the northernmost coastal city,
Eastport. (5)This line would measure about 225 miles. a. Sentence 5 draws a conclusion about sentence 4.
b. Sentence 4 introduces the argument that sentence 5 resolves.
c. Sentence 5 gives an example of the subject of sentence 4.
d. Sentence 5 gives information about sentence 4.
e. Sentence 4 refutes the argument posed by sentence 5.

117

– PRACTICE TEST 2 –

34. In context, what is the best way to combine sentences 8 and 9?
(8)What is the cause of this irregularity? (9)The ice age, or age of continental glaciation, which occurred thousands of years ago.
a. The cause of this irregularity: the thousands of years old ice age, or age of continental glaciation. b. The cause of this irregularity just happens to be the ice age, or age of continental glaciation, which occurred thousands of years ago.
c. The cause of this irregularity was the ice age, or age of continental glaciation, which occurred thousands of years ago.
d. The ice age, or age of continental glaciation, was the cause of this irregularity, which occurred thousands of years ago.
e. Thousands of years ago, the ice age, or age of continental glaciation, caused this irregularity.

118

35. What is the best way to deal with sentence 11?
(11)Isostatic is a geology term meaning pertaining to alterations in the height of the earth’s crust relative to the sea.
a. Add a comma after term.
b. Add quotation marks around the phrase pertaining to . . . sea.
c. Revise it: Isostatic is a term from the science of geology meaning: pertaining to alterations in the height of the earth’s crust relative to the sea.
d. Delete it.
e. Revise it: The term isostatic, taken from geology, is a term meaning pertaining to alterations in the height of the earth’s crust relative to the sea.

– PRACTICE TEST 2 –

Section 3
Time: 10 minutes
14 multiple-choice questions

ANSWER SHEET:
1.
2.
3.
4.
5.
6.
7.

a a a a a a a

b b b b b b b

c c c c c c c

d d d d d d d

e e e e e e e

8.
9.
10.
11.
12.
13.
14.

119

a a a a a a a

b b b b b b b

c c c c c c c

d d d d d d d

e e e e e e e

– PRACTICE TEST 2 –

Directions: In each of the following sentences, part or all of the sentence is underlined. The underlined text may contain an error in sentence construction, grammar, word choice, or punctuation. Choice a repeats the original underlined text. If there is no error in the underlined portion, choose a. If there is an error, select the answer choice that most effectively expresses the meaning of the sentence without any ambiguity or awkwardness. 1. Now that there are computers and word processing software, the art of hand writing letters has become virtually obsolete.
a. Now that there are computers and word processing software, the art of hand writing letters has become virtually obsolete.
b. Computers and word processing software have made the art of hand writing letters virtually obsolete. c. Now that there are computers and word processing software, it’s obsolete to hand write letters any more.
d. Computers and word processing software are here, and they have made the art of hand writing letters virtually obsolete.
e. The art of hand writing letters was virtually obsolete with computers and word processing software. 121

2. Like his other books, Brazilian author Paulo
Coelho’s The Alchemist is a modern-day fairy tale filled with spiritual insights.
a. Like his other books, Brazilian author Paulo
Coelho’s The Alchemist
b. Like his other books, Brazilian author Paulo
Coelho’s The Alchemist,
c. Brazilian author Paulo Coelho’s The Alchemist is like his other books in that it
d. Brazilian author Paulo Coelho’s The Alchemist, it is like his other books—
e. Like his other books, author Paulo Coelho, of
Brazil, The Alchemist
3. There are almost a million members of the Boy
Scout’s of America; its one of the largest organizations of its kind in the country.
a. Boy Scout’s of America; its
b. Boy Scout’s of America, its
c. Boy Scouts of America; its
d. Boy Scouts of America; it’s
e. Boy Scout’s of America; it’s
4. Out of all of the books we read in our book club this year, I liked our discussion of Middlemarch best. a. this year, I liked our discussion of Middlemarch best.
b. this year, I like our discussion of Middlemarch best. c. this year, I liked Middlemarch best.
d. this year, I liked our discussion of the book
Middlemarch the best.
e. this year, I liked the book Middlemarch best.

– PRACTICE TEST 2 –

5. Although she was a successful professional and her inability to negotiate the car deal made her feel inadequate.
a. Although she was a successful professional and b. Although she was a successful professional, and c. Although she was a successful professional still
d. She was a successful professional and
e. Although she was a successful professional,
6. During the 1980s, the cost of attending college rose three times faster than the median household income.
a. During the 1980s, the cost of attending college rose three times faster than the median household income.
b. During the 1980s, the cost of attending college has risen three times faster than the median household income.
c. During the 1980s, the cost of attending college rose three times faster than the medium household income.
d. During the 1980s the cost of attending college rose three times faster than the median household income.
e. During the 1980’s, the cost of attending college rose three times faster than the median household income.

122

7. After years of surfing lessons and countless hours of practice, the largest wave in the contest was ridden successfully by my best friend.
a. the largest wave in the contest was ridden successfully by my best friend.
b. the largest wave in the contest was rided successfully by my best friend.
c. the largest wave in the contest, my best friend rode successfully.
d. my best friend rode the largest wave in the contest successful.
e. my best friend successfully rode the largest wave in the contest.
8. The goal of any company is to have its product name become ubiquitous—constantly at the very forefront of the mind of every consumer.
a. constantly at the very forefront of the mind of every consumer.
b. constantly at the very forefront of the consumer’s mind.
c. constantly at the forefront of the consumer’s mind. d. constantly at the forefront of consumers minds. e. constantly at the forefront of every consumers mind. – PRACTICE TEST 2 –

9. The San Diego Zoo is operated by the not-forprofit organization The Zoological Society of San
Diego, which was founded in 1916.
a. The San Diego Zoo is operated by the not-forprofit organization The Zoological Society of
San Diego, which was founded in 1916.
b. The San Diego Zoo, which was founded in
1916, is operated by the not-for-profit organization The Zoological Society of San Diego.
c. The Zoological Society of San Diego, which was founded in 1916, is the not-for-profit organization that operates the San Diego Zoo.
d. The Zoological Society of San Diego is a notfor-profit organization that operates the San
Diego Zoo, which was founded in 1916.
e. The San Diego Zoo, founded in 1916, is operated by The Zoological Society of San Diego, a not-for-profit organization.
10. Sean has visited all of the large amusement parks on the east coast; his favorite is the Viper, the fastest roller coaster in the United States.
a. Sean has visited all of the large amusement parks on the east coast;
b. While Sean has visited all of the large amusement parks on the east coast,
c. Since Sean has visited all of the large amusement parks on the east coast, he can say that
d. Sean has ridden roller coasters at all of the large amusement parks on the east coast;
e. Sean has been on every roller coaster on the east coast and

11. Being obstinate as usual, his brothers simply could not get him to change his mind.
a. Being obstinate as usual,
b. He was being obstinate as usual, so
c. Being as obstinate as he usually was,
d. Being his usual obstinate self,
e. He is being obstinate as usual, so
12. It was supposed to be written for the general public, and so the report was so esoteric, only those with inside knowledge of government working could understand it.
a. and so the report was so esoteric,
b. because the report was so esoteric,
c. but then the report was so esoteric,
d. but since the report was so esoteric,
e. and the report was so esoteric,
13. The pain medication Kristy received after surgery offered relief, however, the overwhelming feeling of lethargy was an unexpected side effect.
a. relief, however, the overwhelming feeling of lethargy was an unexpected side effect.
b. relief; however, the overwhelming feeling of lethargy was an unexpected side affect.
c. relief, because the overwhelming feeling of lethargy was an unexpected side effect.
d. relief, since the overwhelming feeling of lethargy was an unexpected side effect.
e. relief; however, the overwhelming feeling of lethargy was an unexpected side effect.

123

– PRACTICE TEST 2 –

14. Being knowledgeable and affable, students will feel more comfortable approaching their teachers with questions or problems.
a. Being knowledgeable and affable, students will feel more comfortable approaching their teachers with questions or problems.
b. Being knowledgeable and affable, teachers will feel more comfortable approaching their students with questions or problems.
c. When teachers are knowledgeable and affable, students will feel more comfortable approaching them with questions or problems.
d. Teachers being knowledgeable and affable will make students feel more comfortable about approaching them with their questions or problems. e. When teachers are knowledgeable and affable, students will feel more comfortable with questions or problems that they can approach them with.

124

– PRACTICE TEST 2 –

Answer Key
Section 1: Essay

Use the following rubric to evaluate your writing. This practice essay is included so you can work on time man-

6









5









4









3










agement and the specific essay-writing strategies you learned in Chapter 3. It’s also here for you to compare your finished product with the rubric. The more you practice and perform these evaluations, the better you’ll understand exactly what your scorers are looking for.

Demonstrates outstanding writing skills
Includes a clear and insightful point of view on the question and reflects excellent critical thinking, using strong examples and other evidence to support this point of view
Contains a strong organization and focus, a clear sense of unity, and a skillful flow of ideas
Demonstrates a strong command of language, with varied and appropriate word choice, and meaningful variation in sentence structure
Contains few, if any, errors in grammar, usage, and mechanics
Demonstrates effective writing skills
Includes a clear point of view on the question and reflects strong critical thinking, using good examples and other evidence to support this point of view
Contains strong organization and focus, a sense of unity, and a flow of ideas
Demonstrates a good command of language, with appropriate word choices and variation in sentence structure
Contains few errors in grammar, usage, and mechanics
Demonstrates competent writing skills, but the quality of the writing may be inconsistent
Includes a point of view on the question and reflects competent critical thinking, using sufficient examples to support this point of view
Contains a general organizational plan and focus, with some unity and flow of ideas
Demonstrates a sufficient but inconsistent command of language, with mostly appropriate word choice and some variation in sentence structure
Contains some errors in grammar, usage, and mechanics
Demonstrates inadequate, but not incompetent, writing skills
Includes a point of view on the question, reflecting some critical thinking, but this point of view may be inconsistent or incomplete, and support may be lacking
Contains a limited organizational strategy and focus, with a weak or inconsistent sense of unity and flow of ideas
Demonstrates a developing but weak command of language, with weak or inappropriate vocabulary, little or no variation in sentence structure, and may contain errors in sentence construction
Contains many errors in grammar, usage, and mechanics

125

– PRACTICE TEST 2 –

2









1








0



Demonstrates limited writing skills and may contain serious flaws
Includes a limited or vague point of view on the question and reflects poor critical thinking, using inadequate or irrelevant examples or other support
Displays a weak sense of organization and/or focus, and may lack unity and/or flow of ideas
Demonstrates an inadequate command of language, with limited or incorrect vocabulary, and incorrect or flawed sentence structure
Contains serious errors in grammar, usage, and mechanics that may make the writing difficult to understand Demonstrates incompetence in writing and contains serious flaws
Does not contain a point of view on the question, or provides little or no support for the point of view
Lacks organization and/or focus, unity, and a flow of ideas
Contains serious errors in vocabulary and sentence structure
Contains serious errors in grammar, usage, and/or mechanics that make the writing difficult to understand An essay that does not answer the question, or is blank, receives a zero.

(Adapted from The College Board)

Here are examples of a couple of essays written on the assignment: Benjamin Franklin is one of the greatest figures in
American history, but he wasn’t right about everything. His claim that “money never made a man happy, nor will it. There is nothing in its nature to produce happiness” ignores the fact that money can provide one thing that is essential to happiness: good health.
While money can do nothing to change our genetic makeup and our physiological disposition to illness and disease, it can give us access to better healthcare throughout our lives. This begins with prenatal care and childhood vaccinations. In impoverished third-world countries, infant mortality rates are three, four, even ten times higher than in the
United States, and as many as one in four women still die in childbirth because they do not have

126

access to modern medical care. Sadly, people who are too poor to afford vaccinations and routine healthcare for their children watch hopelessly as many of those children succumb to illnesses and diseases that are rarely fatal in this country.
Money also enables us to afford better doctors and see specialists throughout our lives. If your child has difficulty hearing, for example, and you have insurance (which costs money) or cash, you can see a hearing specialist and pay for therapy. If you have migraines that make you miserable, you can see a headache specialist and afford medication and treatment. Having money also means being able to afford preventative measures, such as taking vitamins and having regular checkups. It means being able to afford products and services that can enhance our health, such as gym memberships, organic foods, and acupuncture.

– PRACTICE TEST 2 –

Another important thing money can do is enable us to live in a healthy environment. Many of the world’s poorest people live in dirty, dangerous places—unsanitary slums crawling with diseases and health hazards of all sorts. In a particularly poor area of the Bronx, for example, children had an abnormally high rate of asthma because of a medical waste treatment plant that was poisoning their air.
Money can also help us be healthy by enabling us to afford proper heating and cooling of our homes.
This includes being able to afford a warm winter coat and the opportunity to cool off at a pool or in the ocean. On a more basic level, it means being able to afford heat in the winter and air conditioning in the summer. During heat waves, victims of heat stroke are often those who are too poor to afford air conditioning in their apartments. In extreme cold, the same is true: people who freeze to death or become gravely ill because of the cold are often those who are unable to afford high heating bills.
Having money may not make people happy, but it goes a long way to keeping them healthy. And as the saying goes, if you don’t have your health, you don’t have anything.
This is a 6 essay. Here are the elements that make it strong: dramatic hook five-paragraph structure body paragraphs each contain a main idea in a topic sentence position is supported with evidence and details ideas are well developed very few grammatical, spelling, and mechanics errors strong conclusion

with this statement. Because money can buy access to good healthcare. In my opinion, good healthcare is essential to happiness. Therefore, money can make you happy by keeping you healthy.
Money in the first place buys good doctors.
With money, you can afford all kinds of things, like tests that check for diseases and special treatments if you find something wrong. If your pregnant you can get good prenatal care and have a good birth and in poor countries lots of women die in childbirth and lots of babies die while their infants.
If you have money you can buy an air conditioner so it’s not too hot in the summer, and you can afford to have heat all winter. You can also stay out of poor areas like slums that are generally just bad places to live. As they say, money can’t buy you love, but I think it can probably buy you good health, if you don’t feel good, it’s hard to be happy.
This essay score a 3. There is a loose organizational structure, and the writer does take a stand, but it is weakened by a number of disclaimers, such as I think it can probably buy and In my opinion. Remember, a direct, confident approach is best. Many points are made for which there is no evidence or other types of support (what is the connection between health and slums, for example?). Errors in grammar and spelling are plentiful, and include a sentence fragment, a runon sentence, and confused words (their/they’re, your/you’re). The conclusion is one sentence at the end of the last paragraph, and while it does generally reiterate the thesis, it is trite, has a weakening disclaimer, and contains a comma splice.
Section 2: Multiple Choice

Benjamin Franklin once said that “Money never made a man happy yet, nor will it. There is nothing in its nature to produce happiness.” I do not agree

127

1. c. The modifier loosely, meant to describe the noun brushstroke, is an adverb. The adjectival form, loose, is needed here.
2. d. The word its is the possessive form of the pronoun it. The correct word is it’s, the contraction of the words it and is.

– PRACTICE TEST 2 –

3. b. The verb earns is in the wrong tense, shifting from past (was . . . known) to present (earns) to past (was . . . referred to). To be consistent and logical, all verbs should be in the past tense (earned).
4. b. Federally is an adverb, but is modifying the noun tax. Since adjectives rather an adverbs modify nouns, it should be in the adjectival form federal.
5. c. This sentence lacks parallel structure. The three items in the list include two verbs (record and replay) and one noun (storage). The noun should be changed to the verb store.
6. e. There is no error in this sentence.
7. d. This sentence lacks parallel structure. The list is intended to be of performers who influenced Metheny, therefore it should not include a type of music. To correct it, a particular Latin musician should be listed.
8. e. There is no error in this sentence.
9. a. Affect is a verb, meaning “to influence.” The correct word is effect, which is a noun referring to result of the influence.
10. d. The problem in this sentence is a lack of agreement with the pronoun and its antecedent. The antecedent is the singular noun cucumber, which must be replaced by a singular pronoun. Instead of them, it should be used here.
11. d. The verbs shift from present tense (is, make) to past tense (belonged). To be consistent, they should all be in the present tense; therefore, belonged should be changed to belong.
12. c. This is an error in prepositional idiom. The correct phrase is separated from.
13. c. The problem with this sentence is pronounantecedent agreement. They refers to just one catchy slogan, “eat, drink, and be merry.”
Therefore, it should be changed to the singular pronoun it and the verb simplifies.

14. b. The problem with the original sentence is wordiness. Like she was is redundant—the idea is conveyed simply with feeling. The phrase which was boring should be turned into an adjective, boring. Choices c and e each correct one of the two wordy problems. Choice d corrects both, but introduces a new error: there is no need for a semicolon after job.
15. c. Choice a has two punctuation mistakes. The first independent clause (A handful of novels by John Irving were adapted for the big screen) should be separated from the rest of the sentence with a semicolon, and the two titles should be listed after a colon. The comma in choice b is not sufficient to set the clause apart, and its semicolon is incorrectly used before a list. Choices d and e properly punctuate the first clause; note that ending the clause with a period and beginning a new sentence is an acceptable solution. However, they repeat the error of improperly punctuating the second part of the sentence.
16. e. Choice a has three punctuation problems:
CEO’s needs an apostrophe to show possession, accounts does not need an apostrophe (it is merely plural), and companies is not plural but singular, and also needs an apostrophe to show possession. The only choice that corrects all three errors is e.
17. a. The four remaining choices break up the sentence by constructing extra phrases that must be surrounded by commas. They are awkward and less clear than the original.
18. c. Choice a is a sentence fragment, missing both a subject and verb. Choices b and d retain the error with some variation. Choice e corrects it, but is wordier that choice c.
19. b. The problem is improper coordination. What is the relationship between the phrases a popular style of electronic dance music gets its name from the Warehouse Club in Chicago and

128

– PRACTICE TEST 2 –

20. d.

21. b.

22. e.

23. d.

regulars referred to their favorite mixes by DJ
Frankie Knuckles as House Music? The conjunctions for, since, now, and so don’t accurately express it. The sentence is about the origin of the name House Music: where does it come from? Only choice b correctly joins the two phrases.
Choice a uses the redundant phrase 9:00 A.M. in the morning. Choices c and e repeat the error. Choice b corrects it, but adds a new error by replacing the comma with a semicolon and replacing the word but with and.
This creates a dependent clause (and was disappointed at the meager selection) that stands alone after the semicolon.
There are three instances of unnecessary, wordy that phrases in choice a: that he has been campaigning for, that is disgruntled, and that is growing larger by the day. All of them should be turned into adjectives: campaigning, disgruntled, and growing. Only choice b corrects all three.
The problem with choice a is subject-verb agreement. Castling, the subject, is a singular noun that must take the singular form of the verb to be (is, not are). Choices b and d include the same error. Choices c and e correct it, but choice e is the most clear and concise.
There are two problems with choice a. The construction I find it fascinating both the is non-idiomatic, non-standard written English.
In addition, the two elements of the sentence are not parallel: the number of Supreme Court cases should be the grammatical equivalent of that the swing vote. Choice c repeats the idiom error, and b repeats the error of parallelism
(note that even though it includes the word that, the first element is illogical and still does not match the second). Choices d and e use the correct phrase I am fascinated by, but d is more concise.

24. b. Choice a has a misplaced modifier. To teach an exciting marine biology class is illogically describing the visit, rather than the person(s) who wants to teach the exciting class. Choice c repeats the error, and choice d uses the modifier to incorrectly describe the class. In choice e, the modifier could work to describe you, but the sentence is unnecessarily wordy.
25. a. Choice b uses the adverb when, which indicates time. Choice c inserts a colon to introduce the names of the four states; this would be correct only if the sentence ended with the list of four. In choice d for which does not make grammatical sense. Choice e is wordier than choice a.
26. d. Choice a’s use of the passive voice is wordy and awkward. Choices b and e have the same error, and also use incorrect verb tenses (is considered and are considered rather than should consider). Choices c and d are in the active voice, but d is more concise; it changes considers the use of to consider using.
27. a. Check the relationship between the two clauses the yoga instructor waited patiently for her students to find the proper pose, and she performed it with ease. There is a contrast between students and teacher. The conjunction in choice b, because, indicates cause and effect. Choice c’s conjunction, even though, does indicate contrast, but the use of the semicolon is incorrect. Choice d is not standard written English; where, often used (incorrectly) in speech, is about place. Choice e, for she was able to, is also incorrect because it does not show the contrast between students and teacher. Choice a’s use of a comma and the conjunction even though correctly joins the two clauses.

129

– PRACTICE TEST 2 –

28. c. Choice a has a misplaced modifier; the 1940s are not also known as action painting and the
New York school. Choices b and e repeat this error. Choice d fixes the modifier problem, but is less clear and concise than choice c.
Note that c breaks up the material into two sentences. 29. c. Choice a contains a faulty comparison. The first clause tells about why the invention happened, and the second tells only the result (or lack thereof) of the invention. Choices b and e repeat the error. In choice d, information is added that corrects the comparison, but it is in the wrong verb tense. The rest of the sentence is in the past tense (invented, did not), so makes should be made.
30. b. Choices a and d use pronouns in a confusing way. In a, it moved incorrectly refers to the weight of the glaciers. It makes more sense to say the glaciers moved, not their weight moved.
In choice d, the modifier as they moved over the land also incorrectly describes the weight of the glaciers, and not the glaciers themselves.
Choices c and e are wordy, run-on sentences.
31. d. There is no argument posed by either sentence, so choices b and e are incorrect. There is also no conclusion drawn, or example given.
The distance specified in sentence 5 is information describing the line mentioned in sentence 4.
32. a. Only sentence 1 is general enough to be a main idea. All of the other sentences listed are too specific.
33. a. Recall that when you are asked to add a sentence, there is a poor transition in the passage that needs improvement. In this case, sentence
15 skips to an entirely new idea. While all of the choices acknowledge the new idea in light of the old, only one does it with an appropri-

ate conjunction (however), and tone consistent with the rest of the passage (not overly informal). 34. c. The sentences must not only be combined smoothly, without confusion, but must also transition well from the previous sentence.
Sentence 7 is: The distance is more than ten times as far. Choices d and e do not make the transition. Choice a includes the awkward phrase thousands of years old ice age, and choice b uses the informal just happens to be.
35. d. This sentence is unnecessary. The meaning of the term may be gleaned from the context of the passage; it is awkward and intrusive to include the definition. The suggested punctuation changes are not needed, and both revisions are wordier than the original.
Section 3: Multiple Choice

130

1. b. Wordiness is the problem with most of the choices for this sentence. Choices a and c use the unnecessary phrase now that there are.
Choice d varies the error with are here and they have made. Choice e obscures the meaning of the sentence by using the past tense was virtually obsolete.
2. a. Choice b adds a superfluous comma after
Alchemist. Choice c is grammatically sound but wordy. Choices d and e are confusing. The pronoun it in choice d is unnecessary, and choice e creates a misplaced modifier that sounds as if the author is one of his other books. 3. d. There are two punctuation errors in this sentence. Boy Scouts is a plural noun, not a possessive one, so it does not need an apostrophe.
The word after the semicolon should be the contraction of the words it and is, spelled it’s.
Only choice d corrects both of these errors.

– PRACTICE TEST 2 –

4. c. Choices a, b, and d have faulty comparisons; they compare books to discussions. Only choices c and e correct the error, but since e uses the unnecessary words the book, c is the most concise.
5. e. Choice a is a sentence fragment. Adding a comma in choice b does not correct the error.
The word still in choice c. conveys the correct meaning and resolves the fragment issue, but the lack of punctuation turns it into a run-on sentence. In choice d, the deletion of the word although changes the meaning of the sentence. Choice e demonstrates that the addition of a comma and removal of the word and correct the sentence.
6. a. Choice b uses the wrong verb tense (the present participle has risen instead of the simple past rose). In choice c, the correct word median is changed to medium. Choice d eliminates the comma after the introductory phrase during the 1980s. Choice e incorrectly uses an apostrophe in 1980s.
7. e. The problem with choice a is improper use of the passive voice. This sentence is about action, and it benefits from the active voice.
Choice b also uses the passive voice, and adds a verb tense error; to ride is an irregular verb whose simple past tense is rode, and past participle is ridden. Choice c is illogical, and forms a misplaced modifier with the addition of a comma after contest. The wave didn’t take lessons and practice. Choice d is in the active tense, but changes the adverb successfully, which modifies the verb rode, to an adjective modifying the noun contest.
8. c. Choice a is unnecessarily wordy; the word very and the construction mind of every consumer could be eliminated or tightened. Choice b corrects the mind of every consumer problem,

9. a.

10. d.

11. b.

12. d.

131

but does not remove very. Choice d incorrectly spells consumers without the possessive apostrophe. Choice e repeats that error, and introduces the unnecessary word every.
Choices b, d, and e rearrange the sentence, placing the phrase which was founded in 1916 so that it incorrectly modifies the San Diego
Zoo instead of the Zoological Society. Choice c is correct, but not as clear and concise as choice a.
The problem with this sentence is faulty comparison—comparing amusement parks with roller coasters. Choices b and c repeat the error. Choice e might be true, but that information was not part of the original sentence.
The elimination of the semicolon also turns it into a run-on sentence. Only choice d eliminates the original error without introducing a new one.
Choice a has a misplaced modifier. Being obstinate as usual does not refer to the brothers who are trying to change his mind, but rather to the one whose mind they are trying to change. With some variations, choices c and d repeat the error. Choice e corrects it, but changes the verb tense to present when the past is required (his brothers could not get).
Only choice b is correct.
Improper coordination of ideas is the problem with choices a, b, c, and e. What is the relationship between the clauses It was supposed to be written for the general public and the report was so esoteric? Choices a and e incorrectly indicate addition. Choice b indicates cause and effect. Choices c and d both correctly use but, which shows there is a contrast between the ideas, however while but then is okay in spoken English, it is not standard written
English.

– PRACTICE TEST 2 –

13. e. Choices a, c, and d are run-on sentences. The correct punctuation mark for separating independent clauses is either a semicolon or period, not a comma. In addition, c and d use conjunctions (because, since) that distort the meaning of the sentence. Choice b corrects the run-on sentence, but adds the word affect instead of effect, which is an error.

14. c. Choices a and b have misplaced modifiers.
Being knowledgeable and affable refers to teachers, not students. Choice d corrects the error, but the sentence is awkward and wordy
(the words about and there are unnecessary).
Choice e also corrects the modifier problem, but the word order confuses the meaning of the sentence. Students won’t feel more comfortable with questions and problems, they’ll feel more comfortable approaching their teachers. 132

C H A P T E R

6

Practice Test 3

Section 1
Time: 25 minutes
Essay
Directions: In the essay, you will demonstrate how well you develop and present ideas. Your goal is to clearly use language, firmly take a point of view, and logically advance your argument.
You must use only the space provided, which will be adequate if you pay attention to handwriting size and margins. Avoid leaving extra space, such as through double-spacing or leaving a blank line between paragraphs.
Write legibly so the scorers of your essay can understand what you have written. You may use your test booklet to take notes and organize your thoughts, but only what is written on the answer sheet will be scored.
There are 25 minutes in which to write your essay. Carefully read the prompt and your assignment. Respond only to the assignment—off-topic essays will receive a zero.

133

– PRACTICE TEST 3 –

The photograph or picture that moved me the most is
.

Assignment: Visual images have the power to inspire thought, evoke emotion, create mood, and even make political statements. Complete the statement, and write an essay that explains your choice of image. You may choose any image, including a family photograph, famous work of art, drawing or painting done by a friend, or even a book illustration. Support your choice by using appropriate examples and details.

134

– PRACTICE TEST 3 –

135

– PRACTICE TEST 3 –

136

– PRACTICE TEST 3 –

Section 2
Time: 25 minutes
35 multiple-choice questions

ANSWER SHEET:
1.
2.
3.
4.
5.
6.
7.
8.
9.
10.
11.
12.
13.
14.
15.
16.
17.
18.
19.
20.

a a a a a a a a a a a a a a a a a a a a b b b b b b b b b b b b b b b b b b b b c c c c c c c c c c c c c c c c c c c c d d d d d d d d d d d d d d d d d d d d e e e e e e e e e e e e e e e e e e e e 21.
22.
23.
24.
25.
26.
27.
28.
29.
30.
31.
32.
33.
34.
35.

137

a a a a a a a a a a a a a a a

b b b b b b b b b b b b b b b

c c c c c c c c c c c c c c c

d d d d d d d d d d d d d d d

e e e e e e e e e e e e e e e

– PRACTICE TEST 3 –

General Directions: Choose the best answer to each question and fill in the oval that matches that answer on your answer sheet. Errors must be erased completely.

Directions: Each of the following sentences has four underlined words or phrases. Read each sentence and determine which underlined portion, if any, has an error in grammar, usage, word choice, or idiom (standard expression). If there is no error, select choice e—No error. No sentence has more than one error.

1. According to recent statistics, approximately 9,000 people die annual from injuries caused by an accident a b c involving a fall, a number only exceed by deaths and injuries resulting from automobile accidents. d No error. e 2. One of the best ways to prepare for a career in journalism is to become an informed citizen by reading a a b variety of newspapers, watching documentaries and televised news programs, and you should read books c d about world leaders, politics, and grassroots movements. No error. e 3. Many travelers use Internet ticket outlet sites and travel agents to find low-priced airline fares, not a b to realize that the airlines themselves often offer the cheapest tickets. No error. c d e 4. The physical and psychological unrest of the working class was explored often in the plays of Arthur Miller, a b for who the subject of the American Dream, and its achievability for ordinary Americans, never got stale. c d
No error. e 5. Although it had its beginnings in Buddhism, mindful meditation is practiced around the world by people a b of many religions, and are shown to be effective for not only stress reduction, but for healing purposes as c d well. No error. e 139

– PRACTICE TEST 3 –

6. It is an enormous commitment, but every morning the members of the team get up at five and head to a b c the pool to practice swimming. No error. d e
7. Many people believe that the end justifies the means, so weather their intentions are good or bad is a b c irrelevant—the result is the only thing that matters. No error. d e
8. The word Gestalt, meaning a configuration that is so unified it can’t be considered in terms of parts but a b c only as a whole, come from a school of psychology developed by Max Wertheimer, Wolfgang Kohler, and d Kurt Koffka in 1912 Germany. No error. e 9. In 1839, Romantic painter and printmaker Louis-Jacques-Mandé Daguerre revealed an invention to the a b
French Academy of Sciences that he called the daguerreotype, which we now know as photography. c d
No error. e 10. The pilgrims whose journey is followed in Geoffrey Chaucer’s masterpiece The Canterbury Tales represent a b a cross section of 14th-century English life, and told tales that include a variety of medieval genres. c d
No error. e 11. Americans get almost one-third of their calories from junk food that offers little nutrition, but plenty of a b c problems, such as obesity, diabetes, and chronic disease. No error. d e
12. Think twice before sending potentially computer-clogging e-mail attachments such as pictures and videos; a b if the recipient is low on disk space, or uses a dial-up service to get their e-mail, he or she won’t appreciate c d the gesture. No error. e 140

– PRACTICE TEST 3 –

13. During the nineteenth century, the sport of fly fishing underwent a period of great change as rod designs a were improved, plaited silk lines entered in production, and Calcutta bamboo became the rod wood of b c choice over the previously popular lancewood. No error. d e
14. The Taj Mahal, a marble monument designed by Shah Jehan as a tributary to his wife, was completed a in 1648, and incorporates many of the traditional themes of Islamic architecture, such as onion-shaped b c d domes and flanking towers. No error. e 15. Many birdwatchers, or birders as they prefer to be called, keep journals called life lists, in which they a b formally note birds they have seen and identified, typically beginning with the feathered creatures c observed in one’s own backyard. No error. d e
16. Vintage handmade marbles, including mica shooters, a sulfide (in which a small figure is imbedded), a b c and latticinio cores, have seen their values skyrocket as collectors place greater demand on the market. d No error. e 17. Among debaters, a straw man is not a bird-chaser used in cornfields, but rather a logical fallacy in which a b one side creates a ridiculous image of the other side and then it is easily attacked it for its ridiculousness. c d
No error. e 18. The purpose of the camp is improving the overall physical condition of each camper so a b his or her performance in any sport is enhanced. No error. c d e 141

– PRACTICE TEST 3 –

Directions: In each of the following sentences, part or all of the sentence is underlined. The underlined text may contain an error in sentence construction, grammar, word choice, or punctuation. Choice a repeats the original underlined text. If there is no error in the underlined portion, choose a. If there is an error, select the answer choice that most effectively expresses the meaning of the sentence without any ambiguity or awkwardness. 19. John’s sports car is faster and hugs the road tighter than Mr. Alberto.
a. John’s sports car is faster and hugs the road tighter than Mr. Alberto.
b. John’s sports car is faster and is a tighter road hugger than Mr. Alberto.
c. John’s sports car is faster and hugs the road tighter like Mr. Alberto’s.
d. John’s sports car is able to go faster and hugs the road tighter than Mr. Alberto.
e. John’s sports car is faster and hugs the road tighter than Mr. Alberto’s.
20. Diatomaceous earth is made of the crushed, fossilized skeletons of aquatic organisms, and is used primarily as protection for stored grain in many parts of the world.
a. organisms, and is used primarily as protection for stored grain in many parts of the world.
b. organisms, so it is used primarily as protection for stored grain in many parts of the world.
c. organisms, but is used primarily as protection for stored grain in many parts of the world.
d. organisms: and it is used primarily as protection for stored grain in many parts of the world. e. organisms, when is used primarily as protection for stored grain in many parts of the world. 21. The class of 06 had a great time at their prom, they also enjoyed the party the next day at the beach. a. The class of 06 had a great time at their prom, they also enjoyed the party the next day at the beach. b. The class of 06 had a great time at their prom.
They also enjoyed the party the next day at the beach. c. The class of ’06 had a great time at their prom.
They also enjoyed the party the next day at the beach. d. The class of 06 had a great time at their prom; they also enjoyed the party the next day at the beach. e. The class of ’06 had a great time at their prom, they also enjoyed the party the next day at the beach. 22. The American Red Cross offers a program called the Learn to Swim Program that begins with a class called Introduction to Water Skills, then progresses to Fundamental Aquatic Skills.
a. The American Red Cross offers a program called the Learn to Swim Program that begins with a class called
b. The American Red Cross’s Learn to Swim Program begins with a class called
c. The American Red Cross offers a program that is called the Learn to Swim Program that begins d. The American Red Cross is an organization that offers a program that is called the Learn to Swim Program that begins with a class called e. The American Red Cross offers a program called the Learn to Swim Program that begins with a class called

142

– PRACTICE TEST 3 –

23. John McPhee won a Pulitzer Prize in 1999 for his book Annals of the Former World, which is about, among other things, North American geology.
a. John McPhee won a Pulitzer Prize in 1999 for his book Annals of the Former World, which is about, among other things, North American geology.
b. John McPhee’s book about, among other things, North American geology, is called
Annals of the Former World, and it won a
Pulitzer Prize in 1999.
c. John McPhee wrote a book about, among other things, North American geology, Annals of the Former World, which won a Pulitzer
Prize in 1999.
d. In 1999, John McPhee won a Pulitzer Prize for his book Annals of the Former World, which is about, among other things, North American geology. e. A Pulitzer Prize was awarded to John McPhee in 1999 for his book Annals of the Former
World, which is about many things, including
North American geology.
24. Avoid sitting or lying on the ground during an electrical storm because lightning can hit the earth and travel through the ground and the more of your body that is in contact with the ground, the greater the chances you will get injured.
a. ground and the more of your body that is in contact with the ground, the greater the chances you will get injured.
b. ground, the more of your body that is in contact with the ground, the greater the chances you will get injured.
c. ground; the more of your body that is in contact with the ground, the greater the chances you will get injured.
d. ground, and the more of your body in contact with the ground, the greater chance of injury.
e. ground; and the more of your body that is in contact with the ground, the greater are the chances that you will get injured.

25. A massage technique that uses percussion to relieve muscle tension is called tapotement and has four methods: cupping, hacking, pinching, and rapid shaking or vibrating.
a. A massage technique that uses percussion to relieve muscle tension is called tapotement and has four methods:
b. A massage technique called tapotement uses percussion to relieve muscle tension; the four methods are:
c. There are four methods of percussion used to relieve muscle tension in the massage technique called tapotement:
d. Tapotement is a massage technique that uses percussion to relieve muscle tension and that has four methods:
e. Percussion is used to relieve muscle tension in the massage technique called tapotement, and it has four methods:
26. Stamp collectors, or philatelists, differentiate between on-paper and off-paper stamps; offpaper means the stamp was never used, so it was removed from its paper backing through a process of soaking or steaming.
a. so it was removed from its paper backing through a process of soaking or steaming.
b. and it was removed from its paper backing by soaking or steaming.
c. or it was removed from its paper backing by soaking or steaming.
d. so it was removed from its paper backing by a soaking or steaming process.
e. or it was removed from it’s paper backing by soaking or steaming.

143

– PRACTICE TEST 3 –

27. The Beaufort Wind Scale, invented in 1806 by
Admiral Francis Beaufort, uses the numbers
0–12 to describe air movement; one is a light breeze, seven is near gale, and a storm is ten.
a. to describe air movement; one is a light breeze, seven is near gale, and a storm is ten.
b. to describe air movement. One is a light breeze, seven is near gale, and a storm is ten.
c. to describe air movement: one is a light breeze, seven is near gale, and a storm is ten.
d. to describe air movement as follows: one is a light breeze, seven is near gale, and a storm is ten.
e. to describe air movement; one is a light breeze, seven is near gale, and ten is a storm.
28. Milky quartz is typically opaque and whitish in color because it contains microscopic bubbles or fluid-filled cavities that scatter light rather than allowing it to pass through as it would in a clear crystal. a. Milky quartz is typically opaque and whitish in color because it contains microscopic bubbles or fluid-filled cavities that scatter light
b. Milky quartz is typically whitish and opaque because it contains microscopic bubbles or fluid-filled cavities. These bubbles or cavities scatter light
c. Milky quartz is typically opaque and whitish in color. It contains microscopic bubbles or fluid-filled cavities that scatter light
d. Milky quartz is typically whitish and opaque.
It is because it contains microscopic bubbles or fluid-filled cavities that scatter light
e. Milky quartz contains microscopic bubbles or fluid-filled cavities that make it typically opaque and whitish, and that scatter light

29. Boxing Day, also known as St. Stephen’s Day, is a national holiday in many countries that are celebrated on the day after Christmas.
a. is a national holiday in many countries that are celebrated on the day after Christmas.
b. is a national holiday in many countries that is celebrated on the day after Christmas.
c. is a national holiday in many countries that is celebrated on December 26th.
d. is a national holiday celebrated on the day after Christmas in many countries.
e. is a national holiday in many countries, and is celebrated on the day after Christmas.
30. As the stock market crash of 1929 was known as
Black Friday, the 1987 crash saw the largest oneday percentage drop in value in history.
a. As the stock market crash of 1929 was known as Black Friday,
b. The stock market crash of 1929 was known as
Black Friday,
c. The stock market crash of 1929 lost almost half of the market’s value in less than two months, whereas
d. In the stock market crash of 1929, almost half of the market’s value was lost over the course of two months, whereas
e. Because the stock market crash of 1929 was known as Black Friday,

144

– PRACTICE TEST 3 –

Directions: Questions 31–35 are based on the following passage, a first draft of an essay about the history of bicycles. Read the passage and the questions that follow. For each question, choose the answer that will most improve the passage. Some questions ask you to choose the best revision of a particular sentence or pair of sentences. Other questions ask you to consider how to best improve the overall organization of the passage. In each case, the correct answer is the one that most closely conforms to the conventions of formal writing.
(1)Today, bicycles are so common it’s hard to believe they haven’t always been around. (2)But two hundred years ago, bicycles weren’t even existing, and the first bicycle, invented in Germany in 1818, was nothing like our bicycles today—it was made of wood and didn’t even have pedals. (3)Since then, however, numerous innovations and improvements in design have made the bicycle one of the most popular means of recreation and transportation around the world.
(4)In 1849, James Starley, an English inventor, made the front wheel many times larger than the back wheel, put a gear on the pedals to make the bicycle more efficient, and lightened the wheels by using wire spokes.
(5)Although this bicycle was much lighter and less tiring to ride, it was still clumsy, and ridden mostly for entertainment. (6)In 1839, Kirkpatrick Macmillan a Scottish blacksmith dramatically improved upon the original bicycle design, employing tires with iron rims to keep them from getting worn down. (7)But the back wheel was substantially larger than the front wheel.
(8)Another Englishman, H.J. Lawson, invented the “safety bicycle,” which had equal sized wheels that were less prone to toppling over. (9)Lawson also attached a chain to the pedals to drive the rear wheel. (10)As the bicycle improved over time, its popularity grew and it became useful for transportation.

145

– PRACTICE TEST 3 –

31. Which of the following is the best way to revise the underlined portion of sentence 2?
(2)But two hundred years ago, bicycles weren’t even existing, and the first bicycle, invented in Germany in 1818, was nothing like our bicycles today—it was made of wood and didn’t even have pedals.
a. ago, there were no existing bicycles,
b. ago, bicycles weren’t even around,
c. ago bicycles weren’t even existing
d. ago, bicycles didn’t exist,
e. ago bicycles didn’t exist

34. Considering the context of the passage, which of the following sentences is best to insert between sentences 9 and 10?
a. These innovations made it easier to ride.
b. Lawson and his family made the bicycle a better machine.
c. It was Lawson who finally made the bicycle a two-wheeled machine.
d. The English finally perfected the machine they invented over a hundred years earlier.
e. No additional sentence is needed.

32. What is the most logical information, in context, to add before sentence 8?
a. In France at that time, other inventions were being made.
b. The English were quite ready now to really improvement the bicycle.
c. Bicycles began to be used for transportation at this time.
d. It didn’t matter which tire was larger, the front or the back.
e. It wasn’t until 1874 that the first truly modern bicycle appeared on the scene.

35. What is the best title for the passage?
a. From Germany to England: Europe’s Exciting
Inventions
b. Two Hundred Years of Cycling History
c. The Development of the Modern Bicycle
d. The Bicycle: Big Wheels to Small Wheels
e. Recreational Uses of the Bicycle

33. What is the best order of sentences for paragraph 2?
a. 4, 5, 6, 7
b. 6, 7, 5, 4
c. 6, 7, 4, 5
d. 4, 5, 7, 6
e. 4, 6, 7, 5

146

– PRACTICE TEST 3 –

Section 3
Time: 10 minutes
14 multiple-choice questions

ANSWER SHEET:
1.
2.
3.
4.
5.
6.
7.

a a a a a a a

b b b b b b b

c c c c c c c

d d d d d d d

e e e e e e e

8.
9.
10.
11.
12.
13.
14.

147

a a a a a a a

b b b b b b b

c c c c c c c

d d d d d d d

e e e e e e e

– PRACTICE TEST 3 –

Directions: In each of the following sentences, part or all of the sentence is underlined. The underlined text may contain an error in sentence construction, grammar, word choice, or punctuation. Choice a repeats the original underlined text. If there is no error in the underlined portion, choose a. If there is an error, select the answer choice that most effectively expresses the meaning of the sentence without any ambiguity or awkwardness. 1. The American Begonia Society publishes a bimonthly journal called the Begonian, and maintain a virtual greenhouse on their website for those interested in the plant family Begoniacea.
a. Begonian, and maintain a virtual greenhouse on their website
b. Begonian; they also maintain what they call a virtual greenhouse on their website
c. Begonian, and maintains a virtual greenhouse on their website
d. Begonian. The Society maintains a greenhouse that is virtual on their own website
e. Begonian, and on their website a virtual greenhouse is maintained

149

2. The history of ice cream is subject to an erroneous legend that Dolly Madison introduced it to America at a White House reception during her husband’s administration but in fact it was already popular in cities such as Philadelphia and
New York.
a. during her husband’s administration but in fact it was already popular in cities such as
Philadelphia and New York.
b. during her husband’s administration, in fact it was already popular in cities such as Philadelphia and New York.
c. during her husband’s administration; it was already popular in cities such as Philadelphia and New York.
d. during her husband’s administration. In fact it was already popular in cities such as Philadelphia and New York.
e. during her husband’s administration. Therefore it was already popular in cities such as
Philadelphia and New York.
3. The dating of the great Sphinx at Giza became controversial between Egyptologists after John
Anthony West espoused his theory that the erosion on the Sphinx was caused by water, and not blowing sand as was previously believed.
a. The dating of the great Sphinx at Giza became controversial between Egyptologists
b. The dating of the great Sphinx that is in Giza became controversial between Egyptologists
c. The dating of the great Sphinx at Giza became controversial among Egyptologists
d. The dating of the great Sphinx at Giza is becoming controversial among Egyptologists
e. The dating of the great Sphinx at Giza has become controversial among Egyptologists

– PRACTICE TEST 3 –

5. According to the Barbecue Industry Association, three out of four U.S. households own a barbecue grill, more than half of all Americans use their grills year-round, and over 60% of barbecuers were men.
a. According to the Barbecue Industry Association, three out of four U.S. households own a barbecue grill, more than half of all Americans use their grills year-round, and over 60% of barbecuers were men.
b. According to the Barbecue Industry Association, there are three out of four U.S. households that own a barbecue grill, more than half of all Americans use their grills yearround, and over 60% of barbecuers were men.
c. According to the Barbecue Industry Association, there are three out of four U.S. households that own a barbecue grill, more than half of all Americans use their grills yearround, and over 60% of barbecuers are men.
d. The Barbecue Industry Association reports that three out of four U.S. households own a barbecue grill, more than half of all Americans use their grills year-round, and over 60% of barbecuers were men.
e. According to the Barbecue Industry Association, three out of four U.S. households own a barbecue grill, more than half of all Americans use their grills year-round, and over 60% of barbecuers are men.

4. Although more than 95% of earthquakes occur in plate margins (meaning areas in which plates separate, grind past each other, or collide), New
York State has experienced over 400 earthquakes in the past two centuries and it is not near any plate margins.
a. margins (meaning areas in which plates separate, grind past each other, or collide),
b. margins, which are simply areas in which plates separate, or they might grind past each other, or they could collide,
c. margins (“margins” here refers to areas in which plates separate, grind past each other, or collide),
d. margins (meaning those areas in which plates of the earth’s crust separate, grind past each other, or collide),
e. margins. Margins are areas in which plates separate, grind past each other, or collide.

150

– PRACTICE TEST 3 –

8. If you’ve heard that shampoo can cause cancer, bananas may contain a flesh-eating bacteria, or that you could unlock your car door with a cell phone, you’ve read an Internet hoax.
a. shampoo can cause cancer, bananas may contain a flesh-eating bacteria, or that you could unlock your car door with a cell phone,
b. shampoo can cause cancer, you can find flesheating bacteria in bananas, or that you could unlock your car door with a cell phone,
c. cancer can be caused by shampoo, bananas may contain a flesh-eating bacteria, or that you could unlock your car door with a cell phone, d. shampoo can cause cancer, bananas may contain a flesh-eating bacteria, or that you could use your cell phone to unlock your car door,
e. shampoo can cause cancer, bananas may contain a flesh-eating bacteria, or that cell phones can unlock car doors,

6. Jealousy is often defined as the desire to keep something (or someone) that one already has, while envy is the desire to obtain something that someone else has.
a. has, while envy is the desire to obtain something that someone else has.
b. has, so that envy is the desire to obtain something that someone else has.
c. has, while envy is when you desire to obtain something that someone else has.
d. has, and envy is about the desire to obtain something that someone else has.
e. has, because envy is the desire to obtain something that someone else has.
7. Paprika, a powdered form of dried red peppers and mainstay of Hungarian cuisine, became commonly used in the 19th century and is available in varieties ranging from mild to hot.
a. commonly used in the 19th century and is available in varieties ranging from mild to hot.
b. available in varieties ranging from mild to hot after it became commonly used in the 19th century. c. commonly used in the 19th century. Since then, it has become available in varieties ranging from mild to hot.
d. commonly used in the 19th century; it then became available in varieties ranging from mild to hot.
e. commonly used in the 19th century, when it was available in varieties ranging from mild to hot.

9. The Fédération Internationale de Gymnastique
(FIG) began as a group of representatives of the gymnastics associations of Belgium, France, and the Netherlands in 1881, so now includes countries from around the world.
a. in 1881, so now includes countries from around the world.
b. in 1881, but now include countries from around the world.
c. in 1881; now include countries from around the world.
d. in 1881, then later it would include countries from around the world.
e. in 1881, but now includes countries from around the world.

151

– PRACTICE TEST 3 –

10. You can participate in a massive recycling effort just by purchasing carpeting made of polyester rather than nylon fibers, which is made from recycled plastic bottles.
a. fibers, which is made from recycled plastic bottles. b. fibers. These fibers are made from recycled plastic bottles.
c. fibers. They are made from recycled plastic bottles. d. fibers, which are made from plastic bottles that have been recycled.
e. fibers, which makes carpeting out of recycled plastic bottles.
11. One of the longest bridges in the world, the
American Society of Civil Engineers named San
Francisco’s Golden Gate Bridge, completed in
1937, a “Monument of the Millennium.”
a. the American Society of Civil Engineers named San Francisco’s Golden Gate Bridge, completed in 1937, a “Monument of the Millennium.”
b. the American Society of Civil Engineers, completed in 1937, named the Golden Gate Bridge of San Francisco, a “Monument of the Millennium.”
c. San Francisco’s Golden Gate Bridge, a “Monument of the Millennium,” was completed in
1937 by the American Society of Civil
Engineers.
d. the Golden Gate Bridge in San Francisco was completed in 1937 and was named a “Monument of the Millennium” by the American
Society of Civil Engineers.
e. the American Society of Civil Engineers named the 1937 San Francisco bridge the
Golden Gate a “Monument of the Millenium.”

12. Insect pests of farm crops are difficult to control when necessary activities such as tillage; weeding; irrigation; and harvesting increase the pest population and decrease the numbers of its natural enemies. a. tillage; weeding; irrigation; and harvesting increase the pest population and decrease the numbers of its natural enemies.
b. tillage; weeding; irrigation; and harvesting increase the pest population and decrease the numbers of their natural enemies.
c. tillage, weeding, irrigation, and harvesting increase the pest population and decrease the numbers of their natural enemies.
d. tillage, weeding, irrigation, and harvesting increase the pest population and decrease the numbers of its natural enemies.
e. tillage, weeding, irrigation, and harvesting increase the pest population and decrease the number of its natural enemy.
13. Gertrude Stein coined the phrase “the Lost Generation” to describe a group of American expatriate writers living in Paris in the 1920’s and 30’s, including Ernest Hemingway, F. Scott Fitzgerald, and Sherwood Anderson.
a. American expatriate writers living in Paris in the 1920’s and 30’s, including
b. American expatriot writers living in Paris in the 1920’s and 30’s, including
c. American expatriate writers living in Paris in the 1920s and 30s, including
d. American expatriate writers living in Paris in the 1920s and 30s and who included
e. American expatriot writers living in Paris in the 1920s and 30s, including

152

– PRACTICE TEST 3 –

14. Anton Bruckner, considered the Austrian master of the 19th-century symphony, couldn’t support himself completely by being a composer, so he also worked as a church organist, first at St. Florian in Ansfelden and then at Linz Cathedral.
a. Anton Bruckner, considered the Austrian master of the 19th-century symphony, couldn’t support himself completely by being a composer, so he also worked as a church organist, first at St. Florian in Ansfelden and then at
Linz Cathedral.
b. Austrian master of the 19th-century symphony, Anton Bruckner supported himself as a composer by working as a church organist, first at St. Florian in Ansfelden and then at
Linz Cathedral.
c. As the Austrian master of the 19th-century symphony, Anton Bruckner was not only a composer, but also supported himself by working as a church organist, first at St. Florian in Ansfelden and then at Linz Cathedral.
d. Austrian master of the 19th-century symphony, Anton Bruckner had to resort to supporting himself as a composer by working as a church organist, first at St. Florian in Ansfelden and then later at Linz Cathedral.
e. Indisputably the Austrian master of the 19thcentury symphony, Anton Bruckner couldn’t completely support himself as a composer, so he worked as a church organist, first at St. Florian in Ansfelden and then at Linz Cathedral.

153

– PRACTICE TEST 3 –

Answer Key
Section 1: Essay

Use the following rubric to evaluate your writing. This practice essay is included so you can work on time man-

6









5









4








3










agement and the specific essay-writing strategies you learned in Chapter 3. It’s also here for you to compare your finished product with the rubric. The more you practice and perform these evaluations, the better you’ll understand exactly what your scorers are looking for.

Demonstrates outstanding writing skills
Includes a clear and insightful point of view on the question and reflects excellent critical thinking, using strong examples and other evidence to support this point of view
Contains a strong organization and focus, a clear sense of unity, and a skillful flow of ideas
Demonstrates a strong command of language, with varied and appropriate word choice, and meaningful variation in sentence structure
Contains few, if any, errors in grammar, usage, and mechanics
Demonstrates effective writing skills
Includes a clear point of view on the question and reflects strong critical thinking, using good examples and other evidence to support this point of view
Contains strong organization and focus, a sense of unity, and a flow of ideas
Demonstrates a good command of language, with appropriate word choices and variation in sentence structure
Contains few errors in grammar, usage, and mechanics
Demonstrates competent writing skills, but the quality of the writing may be inconsistent
Includes a point of view on the question and reflects competent critical thinking, using sufficient examples to support this point of view
Contains a general organizational plan and focus, with some unity and flow of ideas appropriate word choice and some variation in sentence structure
Contains some errors in grammar, usage, and mechanics
Demonstrates inadequate, but not incompetent, writing skills
Includes a point of view on the question, reflecting some critical thinking, but this point of view may be inconsistent or incomplete, and support may be lacking
Contains a limited organizational strategy and focus, with a weak or inconsistent sense of unity and flow of ideas
Demonstrates a developing but weak command of language, with weak or inappropriate vocabulary, little or no variation in sentence structure, and may contain errors in sentence construction
Contains many errors in grammar, usage, and mechanics

154

– PRACTICE TEST 3 –

2









1








0



Demonstrates limited writing skills and may contain serious flaws
Includes a limited or vague point of view on the question and reflects poor critical thinking, using inadequate or irrelevant examples or other support
Displays a weak sense of organization and/or focus, and may lack unity and/or flow of ideas
Demonstrates an inadequate command of language, with limited or incorrect vocabulary, and incorrect or flawed sentence structure
Contains serious errors in grammar, usage, and mechanics that may make the writing difficult to understand Demonstrates incompetence in writing and contains serious flaws
Does not contain a point of view on the question, or provides little or no support for the point of view
Lacks organization and/or focus, unity, and a flow of ideas
Contains serious errors in vocabulary and sentence structure
Contains serious errors in grammar, usage, and/or mechanics that make the writing difficult to understand An essay that does not answer the question, or is blank, receives a zero.

(Adapted from The College Board)
Here are examples of a couple of essays written on the assignment: You might think a memorable picture would have vivid color, an appealing or inspirational theme, or be something you might want to display and look at every day. That is not the case with the picture that is most memorable to me. Rather, it is a large mural, painted in 1937 by the Spanish artist, Pablo
Picasso, to protest the bombing of a small village in northern Spain.
Surprisingly, there is no vivid red color to show the flowing blood. One must imagine this, for the mural is startlingly gray, black, and white. But there is no avoiding the horror of the images. The figures are not realistically drawn, but are cubist and abstract, and it is apparent that innocent civilians are being slaughtered. A mother screams with her mouth wide open, her head tipped back in heart-

155

rending anguish, as she holds her dead baby. A soldier lies dead on the ground, clutching his broken sword, and three other people are shown in shock and agony. Animals, including a tortured horse and a crying bird, are also portrayed as innocent victims of the massacre.
Some symbols are open to interpretation.
What is the meaning of the bull, which seems simply to be observing, or of the light bulb emitting rays at the top of the mural? Does the bull symbolize brute force, and does the light bulb signify that there is hope? Yet there is no doubt that the distorted, horrible images are intended to shock the viewer. This depiction of human grief is a profound statement of the cruelty and senselessness of war. Limiting the pictures to black and white adds a funereal element to the shocking depiction of the catastrophe. – PRACTICE TEST 3 –

This picture could be for any war it doesn’t matter. In that way it is a universal message. There is not anything in the picture that tells you where it is happening. You don’t know who the people are.
There are wars happening today. People suffer now like in Guernica. You remember it because it makes you upset and you wish there would never be a war.
Then people wouldn’t have to suffer. This picture is memorable because you remember how the people suffered and they probly didn’t do anything.

The memory of the picture cannot be forgotten; it is a metaphor for the senselessness and the horror of war. While it was painted to protest atrocities in a long ago war, it is as relevant today as the recollection of the horrors of September 11th. Perhaps it should be shown to all those who contemplate starting a war. Would it be worth it to have another Guernica?
This essay received a score of 5. While the writing skills are effective, the organization could be improved.
For example, the fact that the painting is black and white is mentioned in the second and third paragraphs, both times noting how the color choice adds to the mood of the painting. Paragraph three has a number of major points; it would be less confusing if each point had its own paragraph.
There is a clear point of view, and the writer has obviously studied not only the painting, but the language of art criticism as well. Examples are well chosen and numerous. Word choice is varied and sophisticated, and there are very few errors in grammar and mechanics. If the essay were better organized, and the writer had followed the five-paragraph form, it could have received a score of 6.

This essay received a 3. Organizationally, it has three paragraphs that each contains a main idea. However, two of them also include the introduction and conclusion. While they don’t detract from or confuse the author’s ideas, there are numerous errors in grammar and spelling. Most sentences are very short, and the lack of variety detracts from the essay. A strong point of view is maintained, but it gets lost in the unsophisticated and overly informal vocabulary.
Section 2: Multiple Choice

The picture I remember is Guernica. It is by Picasso.
It is not realist. That means the shapes don’t look real but you know what they are in real life. It is in black and white. It is not in color like most pictures.
But it really gets to you. It shows people getting killed or who are already killed. The images make it so you won’t forget it.
What this picture does is to make you know that war kills people and it is just awful. A baby is killed and a soldier is killed. A mother is screaming because her baby is dead. It kills people and it kills animals and even if you are not killed you will probly be screaming or crying. There are lots of ways that life gets destroyed by war. The painting shows many of them.

156

1. b. Annual is an adjective, which modifies nouns and other adjectives. What is needed is an adverb to modify the verb die. The correct form is annually.
2. d. This sentence lacks parallel structure. To be correct, you should read should have the same grammar structure as reading and watching.
The sentence should read . . ., and reading.
3. c. The verb to realize is in the infinitive form, when it should be a gerund. To correct it, change it to realizing.
4. c. Who is the object of the preposition for, but it is in the subjective case. Correct it by changing it to the objective form whom.
5. c. The verb are shown does not agree with its subject, mindful mediation, which is singular.
This subject requires a singular verb, is shown.
6. e. There is no error in this sentence.

– PRACTICE TEST 3 –

7. b. This is a word choice error; weather refers to meteorological conditions. The correct word is whether.
8. d. The verb come does not agree with its subject, the word Gestalt, which is singular. It should be in the singular form comes.
9. e. There is no error in this sentence.
10. c. The verb forms in this sentence lack consistency. Represent, is followed, and include are in the present tense, and told is in the past. To correct it, change told to the present tense tell.
11. e. This sentence has no error.
12. d. This sentence contains a shift in pronoun from their to he/she. Since the antecedent is the singular noun recipient, change the plural their to the singular his or her to be consistent.
13. c. This is an error in prepositional idiom. The correct phrase is entered into.
14. a. This is an error in word choice. A tributary is a small stream that feeds into a larger stream or lake. The correct word is tribute.
15. d. The error here is a shift in pronoun usage from the plural they to the singular one. To be consistent, one’s should be their.
16. b. Parallel structure is the error; sulfide should be plural as are shooters and cores.
17. c. This sentence has a problem with parallel structure. To be correct, it is easily attacked must be grammatically structured like creates a ridiculous image. Change it to easily attacks.
18. a. The error in this sentence is the non-idiomatic use of a gerund. The phrase the purpose of the camp should be followed by an infinitive. The correct sentence should read The purpose of the camp is to improve . . . .
19. e. Choice a is a faulty comparison of two unlike items, John’s car and Mr. Alberto (a person).
Choices b and d make the same error. In choice c, the word like is incorrectly used to make the comparison.

20. a. The problem with choices b, c, and e is improper coordination. The conjunctions so, but, and when do not correctly convey the relationship between the two phrases. In choice d, there is a punctuation error; a colon is not used to introduce a phrase that begins with the conjunction and.
21. c. The original sentence contains two errors in punctuation, a missing apostrophe in 06 (to replace the missing numbers 20), and a comma splice (incorrect use of a comma to join two complete sentences). Only choice c eliminates both errors.
22. b. Wordiness is the problem with choice a.
Choices c and d repeat the error with some variations (notice the overuse of the words program, that, and called). Choice e corrects the wordiness, but changes the verb begins to beginning, which ruins the parallelism of the sentence (begins/progresses).
23. a. Choice b is wordy and breaks the sentence into many small phrases. Choices c and d also use short phrases, which are awkward. Choice e is in the passive voice and uses the unnecessarily wordy construction which is about many things, including North American geology.
24. c. The original sentence is a run-on. Choices b and d use a comma to solve the problem, but they still need the coordinating conjunction and, which both drop. The semicolon works in both c and e, but choice e is unnecessarily wordy. 25. c. Choice a is in the passive voice, which makes it awkward. Choice b adds confusion—four methods of what? Choice d has two wordy that phrases, and choice e’s and it has is also wordy.
26. c. Choice a uses the wrong coordinating conjunction. So, which indicates that one idea logically follows the other, does not express the relationship between the stamp was never used and it was removed. The ideas don’t fol-

157

– PRACTICE TEST 3 –

27. e.

28 b.

29. e.

30. d.

31. d.

low one another or occur at the same time.
Choice d repeats the error, and choice b uses and, which is also incorrect. Or is the right conjunction, used in both choices c and e.
Choice e, however, introduces a new error: it’s, the contraction of it is, is incorrect.
The problem with choice a is parallelism; the items in a list must be grammatically equivalent. In this case, the number should be first, followed by the air description. Choices b, c, and d repeat this error. Only choice e has correct parallel structure.
Choice a is a run-on sentence that contains the redundant phrase whitish in color. Choice e, also without punctuation, retains the runon sentence error. Choice c repeats the redundant phrase. Choice d solves the run-on sentence problem, but adds the wordy it is because rather than choice b’s because.
There are two problems with choice a: the plural verb are should be the singular is, and the meaning of the modifier that are celebrated on the day after Christmas is unclear. Choice b corrects the verb, but not the modifier. Choice c also repeats the modifier error. Choice d moves the modifier, but it is still unclear; is the holiday celebrated on another day in some countries? The original sentence is a faulty comparison.
The nickname of one crash is being compared to the size of another. Choices b and e repeat this error. Choice c attempts to correct it, but is confusing: the crash didn’t lose half the market’s value—half of the market’s value was lost in the crash.
The problem with the original sentence is verb form. The phrase two hundred years ago tells us the past tense verb is required, meaning existing is incorrect. Choice b is more informal than the rest of the passage, and the deletion of commas in choices c and e makes the sen-

32. e.

33. c.

34. a.

35. c.

tence awkward to read. Choice a is unclear; the phrase existing bicycles sounds as if there were bicycles prior to that time, but none survived. Choice e mentions a year that fits with the chronology of the rest of the passage. None of the other choices are logical in the context of the passage.
Sentences 4 and 6 introduce inventors, and sentences 5 and 7 give greater details about their inventions. Thus the two pairs (4 and 5,
6 and 7) belong together. Choice c restores chronological order.
The two sentences do not transition smoothly, and another sentence is needed. However, there is no mention of Lawson’s family in the passage (choice b), and the bicycle was already two-wheeled (choice c). Paragraph 1 clearly states that the bicycle was invented in Germany (choice d). Lawson’s wheel change and addition of the chain did make the bicycle easier to ride; choice a works to better link the two sentences.
Choice a doesn’t work because only one European invention is mentioned in the passage. In choice b, the word cycling refers to the sport of riding bicycles. The passage never mentions this sport. Choice d is too specific; although changes in wheel size and configuration are discussed, this title is too narrow to represent the whole passage. Choice e has the same problems as b and d—there is no mention of recreational uses, and although its general use for recreation is brought up, this topic is too specific to work as a title.

Section 3: Multiple Choice

158

1. c. The underlined portion of the sentence contains a mistake in parallelism—the Society publishes and maintains, not maintain. Choice b repeats the original error. Choice d corrects

– PRACTICE TEST 3 –

2. d.

3. c.

4. a.
5. e.

6. a.

7. a.

it, but adds the wordy phrase that is virtual.
Choice e also corrects it, but unnecessarily changes the phrase to the passive voice.
The original is a run-on sentence. Choice b separates the clauses with a comma, creating a comma splice. In choice c, a semicolon is used correctly, however the deletion of the conjunction but confuses the meaning of the sentence.
Choice e correctly uses a period and begins a new sentence, but it introduces a new error by including the conjunction therefore. The information in the second clause it was already popular . . . New York is contradictory to the information in the first clause. Therefore does not correctly express that contradiction.
Choice a incorrectly uses the word between, which refers to two things, instead of among, which refers to three or more. Choice b repeats the error. Choices d and e use the wrong verb forms. The context, which refers to an event in the past, and the rest of the sentence indicate that the simple past tense is required. There is no error in the underlined portion of the sentence.
The problem with choice a is with parallel structure. The first two items in the list (three
. . . own and more than half . . . use) are in the present tense. The last item, over 60% . . . were, is in the past tense. Choices b and d repeat the error. Choice c corrects it, but adds an unnecessarily wordy construction (there are three . . . that). There is no error in the original sentence. The distracters b and e have errors in the coordinating conjunction—so that and because do not express the relationship between the two clauses. Choices c and d ruin the parallel structure of the sentence by changing envy is the desire (it matches jealousy is . . . the desire).
There is no error in this sentence.

8. e. Parallelism is the problem with most of the choices. There are three things listed and they must all have the same grammatical structure: shampoo can cause, bananas may contain, cell phones can unlock.
9. e. Choice a uses the wrong coordinating conjunction. The Fédération (a singular noun) was one thing (a group of representatives from three countries), and/but now it is another (a group that includes many other countries). So implies incorrectly that there is cause and effect. Choice d’s use of then is incorrect for the same reason. Choice b has an appropriate conjunction, but the plural form of the verb, include, doesn’t match the singular noun
Fédération. A verb tense error is also a problem in choice c.
10. b. It’s unclear what the phrase which is made from recycled plastic bottles modifies— carpeting, a singular noun, or fibers, a plural noun. Choice c is also confusing; does they refer to carpeting or fibers? Choice d changes the verb form to correctly match the plural fibers, but is unnecessarily wordy. Choice e is illogical—the fibers do not make carpeting.
Choice b completely clears up the confusion by restating the noun fibers.
11. d. The original sentence has a misplaced modifier—the American Society of Civil Engineers is not one of the longest bridges in the world. To correct the error, Golden Gate Bridge needs to follow the modifier one of the longest bridges in the world. Choices b and e repeat the error. Choice c rearranges the sentence to incorrectly state that the American Society of
Civil Engineers built the bridge.
12. c. Choice a has a simple error: semicolons are only used in a list when one or more items in the list contain a comma. Choice b repeats this error. In choices d and e, the correct plural pronoun their (antecedent—pests) is changed to the singular its.

159

– PRACTICE TEST 3 –

13. c. The original sentence has a punctuation error—plural numbers such as 20s and 30s do not use an apostrophe. Choice e corrects that error, but exchanges the word expatriate (a noun or adjective meaning “voluntarily living in another country”) for expatriot (which is not a word in standard English). Choice b includes both errors, while choice d introduces a new one—and who included is incorrect. There is no need for a coordinating conjunction. 14. b. Compare choice a to choice b, noting how careful editing creates a clear and concise sentence out of a wordy, awkward one. Choices c, d, and e don’t contain any errors in grammar, word choice, or punctuation, but they are not as clear and concise as choice b.

160

Similar Documents

Free Essay

Fitness Guide

...Today’s Workout: __________________________________________________ Notes: __________________________________________________________ ■ ■ NAILED IT BARELY MADE IT REMINDER: Take your “before” photo and enter your stats on the measurement chart located in the Quick-Start Guide or on p. 12 of this journal. • Day 2 “It’s 25 minutes to change your life, so just push through.” Today’s Workout: __________________________________________________ Notes: __________________________________________________________ • Day 3 Notes: __________________________________________________________ BARELY MADE IT ■ ■ NAILED IT BARELY MADE IT “It ain’t easy, but it’s worth it.” Today’s Workout: __________________________________________________ Notes: __________________________________________________________ • Day 5 NAILED IT “Today is about progress, not perfection.” Today’s Workout: __________________________________________________ • Day 4 ■ ■ ■ ■ NAILED IT BARELY MADE IT “First Double Day! Do your best and don’t quit!” Notes: __________________________________________________________ ■ ■ NAILED IT Today’s Workouts: _________________________________________________ BARELY MADE IT Week 1 Weight Waist Chest • ■ ■ NAILED IT BARELY MADE IT STATURDAY Thigh Arm 2 ALPHA CYCLE • Day 1 • Week 2 “Weekend is over, now it’s T-time. Let’s get into it.” Today’s Workout: __________________________________________________ ...

Words: 1340 - Pages: 6

Premium Essay

Cade

...resting, the athlete will regain strength and his/her muscles will have built back up within the first week or so. Light workout would be expected to stay in shape, as well as not totally falling off some sort of eating schedule or plan. The athlete may begin some sort of cross training program to add a variety to training. However cross training should no longer be in the athletes workout plan prior to preseason because it is not necessarily sport specific. Preparatory Period: The preparatory season is the official off season for the athlete. For the first several weeks, the athlete will be in some sort of an endurance phase. Meaning whatever workouts they are participating in, they should aim to do low intensity and high volume. So for instance in the weight room they should be doing low weight with a higher amount of reps. Doing so allows for development of endurance base. Following the endurance phase should come the strength phase. This phase is a time where in the weight room the athlete would begin to add weight and lower the repetitions. This phase probably would not consist of as much cardio as the endurance phase depending on the sport and/or position. The third phase of the preparatory period is the power phase. The power phase is a time where the athlete would start doing heavy weight in the weight room or workouts and low reps, higher intensity workouts to start getting into competition shape. Volume is decreased to allow adequate recovery time. Competition Period: This...

Words: 429 - Pages: 2

Premium Essay

Workout Plan

...constantly did the same workouts, didn’t seem to see any new results. This is why elite trainer Tony Horton created P90X. Horton spent over 20 years in becoming an expert in various areas of fitness. Many call him the “Master of Motivation”, and others that are moved by him include movie stars, professional athletes, and normal people who want amazing bodies. Due to his vast knowledge and skills developed in this concept, he created the number one best-selling home fitness program. P90X is a 90-day home fitness system which is designed to get you in shape. The program includes 12 intense workouts that use resistance and body-weight training, cardio, plyometric, ab work, martial arts and yoga, along with a nutrition plan, fitness guide and workout calendar. The real secret behind P90X is the muscle confusion. The program uses target training phases to allow the body to keep adapting and growing on the workouts. There is a “plateau” that happens whenever a person keeps on doing the same workouts which the body already adapted to and that causes the body not to show any new results. Unlike the same old workout machines used at the gym, the P90X will keep your guessing and it won’t let your body get used to the workouts. When the muscle confusion occurs, your body won’t slow down or stop showing improvements. Some of the workouts in the program include short training cycles which challenge your muscles with intensity and variety at the same time. Also the workouts maximize the fat burning...

Words: 395 - Pages: 2

Free Essay

Workout

...of the new you. Make every second count.” Today’s Workout: __________________________________________________ Notes: __________________________________________________________ • DAY 2 NAILED IT BARELY MADE IT REMINDER: Take your “before” photo and enter your stats on the measurement chart located in the Quick-Start Guide or on p. 12 of this journal. “It’s 25 minutes to change your life, so just push through.” Today’s Workout: __________________________________________________ Notes: __________________________________________________________ NAILED IT BARELY MADE IT • DAY 3 “Today is about progress, not perfection.” Today’s Workout: __________________________________________________ Notes: __________________________________________________________ NAILED IT BARELY MADE IT • DAY 4 “It ain’t easy, but it’s worth it.” Today’s Workout: __________________________________________________ Notes: __________________________________________________________ NAILED IT BARELY MADE IT • DAY 5 “First Double Day! Do your best and don’t quit!” NAILED IT BARELY MADE IT Today’s Workouts: _________________________________________________ Notes: __________________________________________________________ NAILED IT BARELY MADE IT WEEK 1 Weight Chest Waist Arm • STATURDAY Thigh 2 ALPHA CYCLE • DAY 1 • WEEK 2 “Weekend is over, now it’s T-time. Let’s get into it.” Today’s Workout: __________________________________________________...

Words: 1340 - Pages: 6

Premium Essay

Sole F85 Week 5 Assignment 5: Pro Runner Model

...for a treadmill? Below is a review of 10 quality treadmill options than span the spectrum, from the intensity of a competitive athlete workout, to the novice beginner. Sole F85 With the industry’s best warranty, you will immediately notice the solid stability and workmanship of the Sole F85. It has one of the highest user weight limits making it an outstanding selection for big people looking for a sturdy piece of equipment. Stability is only one of the features makes the F85 so appealing. A wide tread platform of 22 inches by 60 inches gives runners...

Words: 960 - Pages: 4

Premium Essay

Writing Center

...Magnificent Writing Center As a college student, essays are inevitable. However, no matter how many times you’ve racked your brain in an attempt to illustrate a well written paper, the process can still prove to be arduous. This is the purpose behind the UCA Writing Center. The UCA Writing Center has been put in place in an attempt to make the essay writing process easier for students. The center is fully equipped with a staff of tutors available to help with any writing issue between the hours of 8:00 am and 4:30 pm, Monday through Friday in the Thompson Center. The center even provides night time hours between 6:00 pm and 8:00 pm every week day, with the exception of Thursday. Thursday night time hours are 7:00 pm to 10:00 pm. During night time hours the center operates in the basement of Main. The center handles an array of issues, and provides unlimited help to students struggling with a writing assignment, or even a speech. Receiving this help is as simple as making an appointment, walking through the door, sending an email, or even picking up the phone. Whether it is as simplistic as comma placement, or as complicated as speech presentation, as long as the UCA Writing Center is in place the student body will always have a helping hand. The Writing Center is an impeccable tool to utilize. The center leaves no room for excuses, as it provides free tutoring services through the entire essay process. Not only will the center provide assistance through the entire writing process...

Words: 538 - Pages: 3

Premium Essay

Paraphrasing

...the original source (Scholarly Writing, 2013). Paraphrased material is usually shorter than the original passage, taking a somewhat broader segment of the source and condensing it slightly. In this paper the following passage will be paraphrased. Original Passage "A good writer is one you can read without breaking a sweat. If you want a workout, you don’t lift a book you lift weights. Yet were brainwashed to believe that the more brilliant the writer, the tougher the going." "The truth is that the reader is always right. Chances are if something you’re reading doesn’t make sense, it’s not your fault it’s the writers. And if something you write doesn’t get your point across, it’s probably not the reader’s fault it’s yours. Too many readers are intimidated and humbled by what they can’t understand and in some cases that are precisely the effect the writer is after. But confusion is not complexity; it’s just confusion. A venerable tradition, dating back to the ancient Greek orators, teaches that if you don’t know what you’re talking about, just ratchet up the level of difficulty and no one will ever know." "Don’t confuse simplicity, though, with simplemindedness. A good writer can express an extremely complicated idea clearly and make the job look effortless. But such simplicity is a difficult thing to achieve because to be clear in your writing you have to be clear in your thinking. This is why the simplest and clearest writing has the greatest power to delight...

Words: 552 - Pages: 3

Premium Essay

Smart Goals

...University of Phoenix Material SMART Goals Part A: Reflect on your results from the Career Interest Profiler Activity and the Career Plan Building Activity: Competencies. Building on your strengths and weaknesses, create five SMART goals to help you with your personal academic and career journey. Resource: University of Phoenix Material: Goal Setting Example: Take a writing workshop in the next 2 to 3 weeks to help me improve my writing skills in order to successfully communicate with others. 1. Keep at least a 3.0 on my report card. 2. Take a math and writing workshop in the next few weeks 3. Keep up with my assignments. 4. 5. Part B: Evaluate your SMART goals according to the SMART criteria. Provide support for your evaluation. | |S |M |A |R |T | |Goal |Is the goal specific? |Is the goal measurable? |Is the goal attainable? |Is the goal realistic? |Is the goal timely? | |Goal 1: |Yes |Yes |Yes |Yes |Yes | |Keeping a 3.0 grade point | | | | ...

Words: 727 - Pages: 3

Premium Essay

Studuent

...the way to do a free writing and desperation writing, which were both helpful to reader writing in future. In short, free writing is doing an unplanned writing without stops and thinks. The main point of free writing was to keep writing, and try to not to make a stop. Also, writer didn’t need to edit the paragraph while in process writing; sometimes, it was even incoherent. And the non-edit function also became the advantage of free writing because most of time people were being stuck in how to write the paragraph right, which would omit the content of the paragraph. Therefore, Dr. Elbow encouraged writer to apply free writing to improve writing. In addition, free writing is considered a workout to the writing; the more you do the more your writing gets shaped. As to desperation writing, it is a method to help people who have had trouble writing; or to save people who were hard to produce a coherent speech or thought. The main point and key of this method is to keep writing as though you were drunk. Keep on doing this until you will find out that there are some usable materials on your paper. Moreover, you can take a piece of paper or pad or card to write down your ideas, feelings, thoughts, perceptions, etc. this would help you build up your writing ability as well. Afterwards, you reorganize the papers or cards and you will receive some new ideas, which should be the return of your work. And it will give you more inspiration. And just keep on writing, make as many as metaphor...

Words: 390 - Pages: 2

Premium Essay

English 11 in Ontario

...English-E11-12 7/27/07 2:24 PM Page 1 Ministry of Education The Ontario Curriculum Grades 11 and 12 English Printed on recycled paper 07-003 ISBN 978-1-4249-4741-6 (Print) ISBN 978-1-4249-4742-3 (PDF) ISBN 978-1-4249-4743-0 (TXT) © Queen’s Printer for Ontario, 2007 2007 REVISED CONTENTS INTRODUCTION 3 Secondary Schools for the Twenty-first Century . . . . . . . . . . . . . . . . . . . . . . . . . . . . . . . . . . . . . . . . . . . The Importance of Literacy, Language, and the English Curriculum . . . . . . . . . . . . . . . . . . . Principles Underlying the English Curriculum . . . . . . . . . . . . . . . . . . . . . . . . . . . . . . . . . . . . . . . . . . . . . . Roles and Responsibilities in English Programs . . . . . . . . . . . . . . . . . . . . . . . . . . . . . . . . . . . . . . . . . . . . THE PROGRAM IN ENGLISH 3 3 4 5 9 Overview of the Program . . . . . . . . . . . . . . . . . . . . . . . . . . . . . . . . . . . . . . . . . . . . . . . . . . . . . . . . . . . . . . . . . . . . . . . . 9 Curriculum Expectations . . . . . . . . . . . . . . . . . . . . . . . . . . . . . . . . . . . . . . . . . . . . . . . . . . . . . . . . . . . . . . . . . . . . . . . 12 Strands in the English Curriculum . . . . . . . . . . . . . . . . . . . . . . . . . . . . . . . . . . . . . . . . . . . . . . . . . . . . . . . . . . . 14 ASSESSMENT AND EVALUATION OF STUDENT ACHIEVEMENT Basic Considerations . . . . . . . . . . . ....

Words: 100005 - Pages: 401

Free Essay

Method

...Part 1: Complete the Research Plan |What is your general topic or area of interest? |My general area of interest is music. | | | | | | | |What is it about your general topic of interest that interests |This topic interests me because music has always been a hobby of mine. In| |you? |fact music is more my passion. | | | | |What questions do you have about the topic that you would like to|How many types of music is there? | |investigate? List them. |Are there any genres of music that today’s music resembles? | | |Why is music used in therapy? | | |Where is music most often played? | |Would any of the questions you...

Words: 640 - Pages: 3

Premium Essay

Media

...A pen (Latin penna, feather) is a writing implement used to apply ink to a surface, usually paper, for writing or drawing. Historically, reed pens, quill pens, and dip pens were used, with a nib dipped in the ink. Ruling pens allow precise adjustment of line width, and still find a few specialized uses, but technical pens such as the Rapidograph are more commonly used. Modern types also include ballpoint, rollerball, fountain, and felt or ceramic tip pens.[1] Contents | The main modern types of pens can be categorized by the kind of writing tip or point: A mark made on paper with a rollerball pen, and the tip of that pen * A ballpoint pen dispenses viscous oil-based ink by rolling a small hard sphere, usually 0.7–1.2 mm and made of brass, steel or tungsten carbide.[2] The ink dries almost immediately on contact with paper. The ballpoint pen is usually reliable and inexpensive.[citation needed] It has replaced the fountain pen as the most common tool for everyday writing. * A fountain pen uses water-based liquid ink delivered through a nib. The ink flows from a reservoir through a "feed" to the nib, then through the nib, due to capillary action and gravity. The nib has no moving parts and delivers ink through a thin slit to the writing surface. A fountain pen reservoir can be refillable or disposable, this disposable type being an ink cartridge. A pen with a refillable reservoir may have a mechanism, such as a piston, to draw ink from a bottle through the nib, or it may...

Words: 1997 - Pages: 8

Premium Essay

Job History

...that, we see how Leeland through his life, tries to find a successful career, but fails consistently. He moves various times from place to place, too seek occupation and good business. But it is hard when you’re a high school dropout, without a career. Leeland have to changes his job constantly, because of his lack of luck, and since he can’t get along white his bosses. He is never able to stay at one job or place for long, which lead to problems in the family. He has a hard time supporting his wife, and their children financially. Throughout the story the author, Annie Proulx manages to show us the miserable live of Leeland Lee, without even adding any emotion to her writing. Meaning that there is no point in the story, when we hear about how Leeland feels, about his miserable life. The writing is in typical Annie Proulx fashion, with short, fragmented sentences There is not a moment in the story, where the action takes a different turn, however we keep on hearing about...

Words: 357 - Pages: 2

Premium Essay

The Duhem-Quine Thesis in Economics: a Reinterpretation

...GUIDELINES FOR WRITING LETTERS OF RECOMMENDATION Letters of recommendation are used to gather additional information about a candidate for employment, graduate study or other opportunity. It should give an overall picture of the candidate's: personal characteristics performance experience strengths capabilities professional promise Preferably, the person writing the letter of recommendation has been in a working or academic relationship with the applicant. It is recommended that the student provide you with a copy of their resume, along with the job description they are applying for. The letter should be about one page in length and generally consist of three parts: opening, body, and closing. Two questions to ask yourself BEFORE accepting a request to write a letter of recommendation 1. Can you speak positive about this individual? If you find yourself making up half truths, or having trouble finding positives, then you need to politely decline serving as a reference, or writing a reference letter. To avoid hurting feelings, simply state you feel you are not familiar enough with their background or work ethic to provide the best recommendation. 2. Can you speak for the applicant on the position they are applying for? If an individual asks you to be a reference, or to provide a reference letter and you do not feel you can provide a well rounded background reference, then you may want to consider either politely declining, or setting up an appointment with that individual...

Words: 990 - Pages: 4

Premium Essay

Essay 1

...An essay is a piece of writing which is often written from an author's personal[->0] point of view[->1]. Essays can consist of a number of elements, including: literary criticism[->2], political manifestos[->3], learned arguments[->4], observations of daily life, recollections, and reflections of the author. The definition of an essay is vague, overlapping with those of an article[->5] and a short story[->6]. Almost all modern essays are written in prose[->7], but works in verse[->8] have been dubbed essays (e.g. Alexander Pope[->9]'s An Essay on Criticism[->10] and An Essay on Man[->11]). While brevity usually defines an essay, voluminous works like John Locke[->12]'s An Essay Concerning Human Understanding[->13] and Thomas Malthus[->14]'s An Essay on the Principle of Population[->15] are counterexamples. In some countries (e.g., the United States and Canada), essays have become a major part of formal education[->16]. Secondary students are taught structured essay formats to improve their writing skills, and admission essays[->17] are often used by universities[->18] in selecting applicants and, in the humanities and social sciences, as a way of assessing the performance of students during final exams. The concept of an "essay" has been extended to other mediums beyond writing. A film essay is a movie that often incorporates documentary film making styles and which focuses more on the evolution of a theme or an idea. A photographic essay[->19] is an attempt to cover a topic...

Words: 521 - Pages: 3